Apuntes Resueltos Todo Calculo 1

129

Transcript of Apuntes Resueltos Todo Calculo 1

Page 1: Apuntes Resueltos Todo Calculo 1

Universidad de Santiago de Chile.

Problemas Resueltos y Propuestos Cálculo 1 MBI.

(Primera versión.)

Cristián Burgos G.

16 de septiembre de 2013

Page 2: Apuntes Resueltos Todo Calculo 1

2

Page 3: Apuntes Resueltos Todo Calculo 1

Índice general

1. Problemas Resueltos. 5

1.1. Números Reales . . . . . . . . . . . . . . . . . . . . . . . . . . . . . . . . . . . . . . . . . . . . . . . . . . . 51.2. Funciones . . . . . . . . . . . . . . . . . . . . . . . . . . . . . . . . . . . . . . . . . . . . . . . . . . . . . . . 151.3. Sucesiones. . . . . . . . . . . . . . . . . . . . . . . . . . . . . . . . . . . . . . . . . . . . . . . . . . . . . . . 211.4. Límite y Contuinidad . . . . . . . . . . . . . . . . . . . . . . . . . . . . . . . . . . . . . . . . . . . . . . . . 311.5. Derivadas . . . . . . . . . . . . . . . . . . . . . . . . . . . . . . . . . . . . . . . . . . . . . . . . . . . . . . . 461.6. Integrales: Métodos de Integración. . . . . . . . . . . . . . . . . . . . . . . . . . . . . . . . . . . . . . . . . . 90

2. Problemas Propuestos. 99

2.1. Inecuaciones . . . . . . . . . . . . . . . . . . . . . . . . . . . . . . . . . . . . . . . . . . . . . . . . . . . . . . 992.2. Funciones . . . . . . . . . . . . . . . . . . . . . . . . . . . . . . . . . . . . . . . . . . . . . . . . . . . . . . . 1012.3. Sucesiones: . . . . . . . . . . . . . . . . . . . . . . . . . . . . . . . . . . . . . . . . . . . . . . . . . . . . . . 1042.4. Límite y Continuidad . . . . . . . . . . . . . . . . . . . . . . . . . . . . . . . . . . . . . . . . . . . . . . . . 1072.5. Derivadas . . . . . . . . . . . . . . . . . . . . . . . . . . . . . . . . . . . . . . . . . . . . . . . . . . . . . . . 1132.6. Integrales . . . . . . . . . . . . . . . . . . . . . . . . . . . . . . . . . . . . . . . . . . . . . . . . . . . . . . . 126

3

Page 4: Apuntes Resueltos Todo Calculo 1

4 ÍNDICE GENERAL

Page 5: Apuntes Resueltos Todo Calculo 1

Capítulo 1

Problemas Resueltos.

1.1. Números Reales

1. Resuelva x3 + x2 ≥ x

Solución: Resolviendo, tenemos que

x3 + x2 − x ≥ 0⇒ x(x2 + x− 1) ≥ 0

note que ∆ = 1 + 4 = 5, luego: x = 1±√

52 , entonces: x

(x−

(1+√

52

))(x−

(1−√

52

))≥ 0 analizando los puntos de

cambio de signo tenemos:

]−∞, 1−√

52 ] [ 1−

√5

2 , 0] [0, 1+√

52 ] [ 1+

√5

2 ,∞[

x−(

1+√

52

)- - - +

x - - + +

x−(

1−√

52

)- + + +

P (x) - + - +

Luego, la solución es: S =[ 1−√

52 , 0]

⋃[ 1+√

52 ,∞[

2. Resolver la inecuación3x+ 2

x+ 1≤ 1x+ 1

Solución: Se tiene3x+ 2− 1

x+ 1≤ 0⇒ 3x+ 1

x+ 1≤ 0

multiplicando por el cuadrado de denominador que siempre es positivo: (3x + 1)(x + 1) ≤ 0. Observe que x 6= −1,además. esta expresión se comporta de formá parabólica, es decir, como una función cuadrática, por lo tanto lasolución de esta inecuación es: S =]−∞,−1[∪[− 1

3 ,∞[

3. Resuelva la inecuación √x2 − x− 2 ≤ x

Solución:

tenemos la restricción

x2 − x− 2 ≥ 0

(x− 2)(x+ 1) ≥ 0⇒ x ∈]−∞,−1] ∪ [2,∞[

5

Page 6: Apuntes Resueltos Todo Calculo 1

6 CAPÍTULO 1. PROBLEMAS RESUELTOS.

Sabemos que si x < 0 , no existe una solución real posible para esta inecuación, dado que los radicales no generannúmeros negativos, ahora si x > 0:

x2 − x− 2 ≤ x2

−x− 2 ≤ 0

x ≥ −2

Realizando las intersecciones correspondientes, llegamos a la solución

S = [2,∞[

4.(x2 + 1)

√x− 1

x2 − 7x+ 10< 0

Solución: note que x − 1 ≥ 0 ⇒x ≥ 1 además x2 + 1 > 0, ∀x ∈ R por lo tanto, esta expresión no afecta elsigno de la inecuación lo que hace que se reduzca a:

1

x2 − 7x+ 10< 0,

que es lo mismo que analizar x2− 7x+ 10 < 0 ⇒(x− 5)(x− 2) < 0 ⇒S =]2, 5[ ahora,a la condición para que la raízexista es que x ≥ 1, luego:

Sf =]2, 5[

.

5. Resuelva(x+ 1)

√x2 − 19x+ 18

x5 − x≥ 0

Solución: factorizando complétamente la expresión tenemos:

(x+ 1)√

(x− 18)(x− 1)

x(x2 + 1)(x− 1)(x+ 1)≥ 0⇒

√(x− 18)(x− 1)

x(x2 + 1)(x− 1)≥ 0

En primer lugar, sabemos que el denominador en ningún caso puede ser cero, luego x no puede tomar los valores −1, 0 y 1. En segundo lugar, la expresión x2 + 1 es siempre positiva, por lo tanto no inuye en el signo de la expresión.En tercer lugar, existe una restricción en la raiz, la cual debe tener su argumento siempre positivo o nulo, es decir

(x− 18)(x− 1) ≥ 0⇒ x ∈]−∞, 1] ∪ [18,∞[= R1

entonces, la inecuación se reduce a:1

x(x− 1)≥ 0⇒ x(x− 1) ≥ 0

lo cual tiene como solución:

S1 =]−∞, 0] ∪ [1,∞[

como se deben cumplir tanto la restricción R1 como la solución S1 tenemos que Sf = R1 ∩ S1, luego:

Sf =]−∞, 0] ∪ [18,∞[−−1

6. Resolver

∣∣∣∣x− 1

x− 2

∣∣∣∣ > 1.

Page 7: Apuntes Resueltos Todo Calculo 1

1.1. NÚMEROS REALES 7

Solución: Usando las propiedades del valor absoluto, analizaremos el primer caso:

x− 1

x− 2> 1

x− 1

x− 2− 1 > 0

x− 1− x+ 2

x− 2> 0

1

x− 2> 0

x > 2

En el segundo caso:

x− 1

x− 2< −1

x− 1

x− 2+ 1 < 0

x− 1 + x− 2

x− 2< 0

2x− 3

x− 2< 0

Analizando los puntos de cambio de signo se llega a que: S2 =] 32 , 2[

Uniendo ambos casos tenemos que:

Sf =]3

2, 2[∪]2,∞[

7. Encuentre el conjunto solución de |x+ 1| − 2|x− 3| ≥ x.

Solución: En este tipo de inecuaciones debemos analizar por casos en donde, en el caso de esta inecuación, te-nemos tres intervalos de análisis

Caso 1: Si x < −1, tenemos:

−(x+ 1) + 2(x− 3) ≥ x

−x− 1 + 2x− 6 ≥ x

x− 7 ≥ x

−7 ≥ 0

Esa desigualdad es falsa... por lo tanto no existen valores de ese intervalo que cumplan con la relación.

Caso 2: Si −1 ≤ x < 3, tenemos:

x+ 1 + 2(x− 3) ≥ x

x+ 1 + 2x− 6− x ≥ 0

3x− 5 ≥ 0

x ≥ 5

3

Interseptando la condición del caso 2 con la solución resultante de la inecuación queda: S2 = [ 53 , 3]

Page 8: Apuntes Resueltos Todo Calculo 1

8 CAPÍTULO 1. PROBLEMAS RESUELTOS.

Caso 3: Si x ≥ 3 tenemos:

x+ 1− 2(x− 3) ≥ x

x+ 1− 2x+ 6 ≥ x

−x+ 7 ≥ x

2x ≤ 7

x ≤ 7

2

Procediendo análogamente al caso anterior tenemos: S3 = [3, 72 ]

Ahora,debemos unir las soluciones de los casos ... nos queda nalmente: Sf = [ 53 ,

72 ]

8. Resuelva la inecuación:3x− 4

|4− 3x| − x2≤ 0

Solución:

1º caso: si 4− 3x < 0⇒ x >4

3, entonces:

3x− 4

3x− 4− x2≤ 0⇒ 3x− 4

x2 − 3x+ 4≥ 0

note que ∆ = 9− 16 < 0 , entonces, x2 − 3x+ 4 > 0, ∀x ∈ R, luego

3x− 4 ≥ 0⇒x ≥ 4

3

lo que conduce a la solución:

S1 =

]4

3,∞[

2º caso: si 4− 3x ≥ 0⇒ x ≤ 4

3

3x− 4

4− 3x− x2≤ 0⇒ 3x− 4

x2 + 3x− 4≥ 0

3x− 4

(x+ 4)(x− 1)≥ 0

resolviendo esa inecuación se llega a la solución parcial:

Sp2 =]− 4, 1[∪[

4

3,∞[

lo que con la condición inicial del caso 2 queda:

S2 =]− 4, 1[∪4/3

Uniendo ambas soluciones tenemos la solución nal:

Sf =]− 4, 1[∪[

4

3,∞[

9. Resuelva la inecuación ||3x+ 9| − |x|| ≥ 5

Solución: Usando un teorema fundamental de las desigualdades del valor absoluto tenemos dos alternativas:

Page 9: Apuntes Resueltos Todo Calculo 1

1.1. NÚMEROS REALES 9

a) |3x+ 9| − |x| ≥ 5

b) |3x+ 9| − |x| ≤ −5

Note que |3x+ 9| = 3|x+ 3| , y además, podemos notar lo siguiente, ∀x ∈ R:

|x+ 3| ≥ |x|

Por lo tanto no es dicil notar que

3|x+ 3| ≥ |x+ 3| ≥ |x| ⇒ 3|x+ 3| − |x| ≥ 0

Con ello entonces podemos concluir que

S(b) = ∅

Analizando la inecuación (a) :

|3x+ 9| − |x| ≥ 5⇒ |3x+ 9| ≥ 5 + |x|

Analizando por casos:

1) si x < −3

−3x− 9 ≥ 5− x⇒ x ≤ −7

Realizando las intersecciones correspondientes

S(i) =]−∞,−7]

2) si −3 ≤ x < 0

3x+ 9 ≥ 5− x⇒ x ≥ −1

Procediendo igual que el caso (i) , se llega a s(ii) = [−1, 0[

3) si x ≥ 0

3x+ 9 ≥ 5 + x⇒ x ≥ −2

Luego , S(iii) = [0,∞[Uniendo los tres casos anteriores, se llega a la solución nal

S =]−∞,−7] ∪ [−1,∞[

10.

∣∣∣∣x3 − 4

3|x|

∣∣∣∣ ≤1Solución: Si x < 0 : ∣∣∣∣x3 +

4

3x

∣∣∣∣ ≤ 1

Resolviendo la inecuación:

−1 ≤ x3 + 4

3x ≤ 1

Page 10: Apuntes Resueltos Todo Calculo 1

10 CAPÍTULO 1. PROBLEMAS RESUELTOS.

Resolviendo por casos:

−1 ≤ x

3+

4

3xx

3+

4

3x+ 1 ≥ 0

x2 + 4 + 3x

3x≥ 0

∆ = 9− 16 < 0, por lo tanto esta inecuación será positiva si x > 0

x

3+

4

3x≤ 1

x

3+

4

3x− 1 ≤ 0

x2 + 4− 3x

3x≤ 0

∆ = 9− 16 < 0,por lo tanto esta inecuación tendrá solución si x < 0

Haciendo la intersección de las inecuaciones resueltas anteriomente tenemos que S1 = φSi x > 0; ∣∣∣∣x3 − 4

3x

∣∣∣∣ ≤ 1

−1 ≤ x3 −

43x ≤ 1

Analizando por casos:

−1 ≤ x

3− 4

3xx

3− 4

3x+ 1 ≥ 0

x2 − 4 + 3x

3x≥ 0

(x+ 4)(x− 1)

3x≥ 0

Realizando el análisis de signo correspondiente:

]∞,−4] [−4, 0[ ]0, 1] [1,∞[

x+ 4 − + + +x − − + +

x− 1 − − − +P (x) − + − +

Luego, la solución es: S∗ = [−4, 0[∪[1,∞[

Page 11: Apuntes Resueltos Todo Calculo 1

1.1. NÚMEROS REALES 11

Para la otra parte de la inecuación:

x

3− 4

3x≤ 1

x

3− 4

3x− 1 ≤ 0

x2 − 4− 3x

3x≤ 0

(x− 4)(x+ 1)

3x≤ 0

Procediendo de forma análoga:

]−∞,−1] [−1, 0[ ]0, 4] [4,∞[

x+ 1 − + + +x − − + +

x− 4 − − − +P (x) − + − +

Luego, la solución es: S∗∗ =]−∞,−1]∪]0, 4]

Realizando las intersecciones de ambas soluciones obtenemos la solución para el segundo caso quedando: S2 =S∗ ∩ S∗∗ = [−4,−1] ∪ [1, 4]. Pero como x > 0; tenemos que: S2c = [1, 4]. Finalmente, uniendo ambas soluciones:Sf = [1, 4]

11. Encuentre los x ∈ R que satisfacen con la inecuación√|x− 2|+ 4−

√x− 6 < 2

Solución:

Deben cumplirse simultaneamente las condiciones

|x− 2|+ 4 ≥ 0

x ≥ 6

Observese que si x ≥ 6 la primera condición ya cumple, por ende queda como restricción general x ≥ 6 (verifíquelo),reescribiendo la inecuación original, asegurando que ambos miembros sean positivos√

|x− 2|+ 4 < 2 +√x− 6

Elevando al cuadrado

|x− 2|+ 4 < 4 + 4√x− 6 + x− 6

Además, note que si x ≥ 6 , |x− 2| = x− 2 , entonces, reduciendo la expresión

x− 2 < 4√x− 6 + x− 6

4 < 4√x− 6

1 <√x− 6

Elevando al cuadrado nuevamente

1 < x− 6⇒ x > 7

Interseptando con la condición, nalmente

S =]7,∞[

Page 12: Apuntes Resueltos Todo Calculo 1

12 CAPÍTULO 1. PROBLEMAS RESUELTOS.

12. Considere la funcion f(x) =|x+ 1|+ x2 − x− 1

(x+ 3)√x2 − 2x+ 1

. Entonces para que valores de x ∈ R la funcion g(x) =√f(x)

representa un numero real.

Solución: No es dicil ver que g(x) sera un numero real si su argumento es mayor o igual que 0, luego hayque resolver la inecuacion:

|x+ 1|+ x2 − x− 1

(x+ 3)√x2 − 2x+ 1

≥ 0

Veamos que la inecuacion se puede reescribir como sigue:

|x+ 1|+ x2 − x− 1

(x+ 3)√

(x− 1)2≥ 0

Recordemos que√a2 = |a| y por tanto reescribimos la inecuacion:

|x+ 1|+ x2 − x− 1

(x+ 3)|x− 1|≥ 0

que de igual forma , se analiza por intervalos. Así, se tiene que:

|x+ 1| =

x+ 1 x ≥ −1

−(x+ 1) x < −1

|x− 1| =

x− 1 x ≥ 1

1− x x < 1

Caso 1. Para cuando x < −1.

Para este caso la inecuacion queda:(−x− 1) + x2 − x− 1

(x+ 3)(1− x)≥ 0

; con x 6= −3, 1 ∧ ]−∞,−1[

⇔ x2 − 2x− 2

(x+ 3)(x− 1)≤ 0

; con x 6= −3, 1 ∧ ]−∞,−1[

Factorizando el numerador, la inecuacion queda:

⇔(x−

[1−√

3])(x−

[1 +√

3])

(x+ 3)(x− 1)≤ 0

; con x 6= −3, 1 ∧ ]−∞,−1[

Analogo a lo que se ha hecho antes, se multiplica por un positivo de la forma ((x+ 3)(x− 1))2, se ordena y resulta:

(x− [−3])(x−[1−√

3])(x− 1)(x−

[1 +√

3]) ≤ 0

; con x 6= −3, 1 ∧ ]−∞,−1[

que por el metodo reducido su solucion sera:

S1 =(

]− 3, 1−√

3] ∪ ]1, 1 +√

3])∩ ]−∞,−1[=]− 3,−1[

Page 13: Apuntes Resueltos Todo Calculo 1

1.1. NÚMEROS REALES 13

Caso 2. Para cuando −1 ≤ x < 1

La inecuacion a resolver en este caso sera:

(x+ 1) + x2 − x− 1

(x+ 3)(1− x)≥ 0

⇔ x2

(x+ 3)(1− x)≥ 0

⇔ x2

(x+ 3)(x− 1)≤ 0

; con x 6= −3, 1∧ [−1, 1[ . Pero x2 ≥ 0 para todo x Real, y luego no inuye en la resolucion de la inecuacion. Portanto, esta puede resolverse como:

1

(x+ 3)(x− 1)≤ 0

Como en casos anteriores, se puede resolver multiplicando por un positivo de la forma [(x+ 3)(x− 1)]2,y la inecuacion

queda:(x− (−3))(x− 1) ≤ 0

Que por el metodo reducido, tiene como solucion:

S2 =]− 3, 1[ ∩ [−1, 1[ = [−1, 1[

Caso 3. Para cuando x ≥ 1

La inecuacion se escribe asi:(x+ 1) + x2 − x− 1

(x+ 3)(x− 1)≥ 0

; con x 6= −3, 1∧[1,+∞[

⇔ x2

(x+ 3)(x− 1)≥ 0

⇔ 1

(x+ 3)(x− 1)≥ 0

(x− (−3))(x− 1) ≥ 0

Por el metodo reducido, sigue que la solucion para este caso sera:

S3 = (]−∞,−3[ ∪ ]1,+∞[) ∩ [1,+∞[ =]1,+∞[

FinalmenteSF = S1 ∪ S2 ∪ S3 =]− 3,−1[ ∪ [−1, 1[ ∪ ]1,+∞[ =]− 3, 1[ ∪ ]1,+∞[

13. ¾Para qué valor de c ∈ R la ecuación |c+π|x2+|c|x+(e−c) = 0 tiene dos soluciones en los reales y estas son distintas?

Solución: Para este problema, basta solo recordar que si el discriminante de una ecuacion cuadratica es mayor quecero, entonces la ecuacion tiene dos soluciones (raices) diferentes en los reales. Luego, el problema se convierte ahallar los valores de c ∈ R tal que

|c|2 − 4|c+ π|(e− c) > 0

⇔ |c|2 + 4|c+ π|(c− e) > 0

Pero por la denicion de valor absoluto: |c| =√c2 ⇒ |c|2 = c2.Y por tanto la inecuacion queda:

c2 + 4|c+ π|(c− e) > 0

Page 14: Apuntes Resueltos Todo Calculo 1

14 CAPÍTULO 1. PROBLEMAS RESUELTOS.

Al igual que anteriormente, se analiza por intervalos que dependeran del valor absoluto, esto es:

|c+ π| =

c+ π c ≥ −π−c− π c < −π

Y ahora se analizan dos casos:

Caso 1. Para cuando c ≥ −π, la inecuacion quedará

c2 + 4(c+ π)(c− e) > 0

⇔ c2 + 4(c2 − ec+ πc− πe) > 0

⇔ c2 + 4(c2 + (π − e)c− πe) > 0

⇔ 5c2 + 4(π − e)c− 4πe > 0

Ahora hay que factorizar la ecuacion cuadratica 5c2 + 4(π − e)c− 4πe, en que sus raices sonc1 = − 4

10(π − e)−

√4πe

5+

(4

10(π − e)

)2

c2 = − 4

10(π − e) +

√4πe

5+

(4

10(π − e)

)2

Para sintetizar un poco, se tiene que 5c2 + 4(π − e)c− 4πe = (c− c1)(c− c2) con c1 < c2.

Con lo que la inecuacion quedara:(c− c1)(c− c2) > 0

Y por el metodo reducido, se tiene que la solucion del primer caso sera:

S1 = (]−∞, c1[ ∪ ]c2,+∞[) ∩ [π,+∞[

Caso 2. Para c < −π.c2 + 4(−c− π)(c− e) > 0

⇔ c2 − 4(c+ π)(c− e) > 0

⇔ c2 − 4(c2 + πc− ec− πe) > 0

⇔ c2 − 4(c2 + (π − e)c− πe) > 0

⇔ −3c2 − 4(π − e)c+ 4πe > 0

⇔ 3c2 + 4(π − e)c− 4πe < 0

Se factoriza 3c2 + 4(π − e)c− 4πe y para ello sus raicesc3 = −4

6(π − e)−

√4πe

3+

(4

6(π − e)

)2

c4 = −4

6(π − e) +

√4πe

3+

(4

6(π − e)

)2

la inecuacion quedara:(c− c3)(c− c4) < 0

por metodo reducido la solucion del caso sera:

S2 =]c3, c4[ ∩ ]−∞, π[

la solucion y respuesta a la pregunta sera las c pertenecientes a:

SF = S1 ∪ S2

Page 15: Apuntes Resueltos Todo Calculo 1

1.2. FUNCIONES 15

1.2. Funciones

1. Considere la función f : [0,+∞[→ R denida por f(x) =x+ 1√x2 + 1

a) Determine el signo de f

Solución: Dado que x ∈ [0,+∞[, tenemos que la función es siempre positiva, entonces y > 0 ∀x ∈ Dom(f)

b) Determine Rec(f)

Solución: Despejando x

y =x+ 1√x2 + 1

y2 =(x+ 1)2

x2 + 1

x2y2 + y2 = x2 + 2x+ 1

x2(y2 − 1)− 2x+ (y2 − 1) = 0

Entonces

x =(1− y2)±

√4− 4(y2 − 1)2

2(y2 − 1)

x =(1− y2)± 2

√1− (y2 − 1)2

2(y2 − 1)

Luego, tenemos que y 6= −1, 1 , además

1− (y2 − 1)2 ≥ 0

|y2 − 1| ≤ 1

−1 ≤ y2 − 1 ≤ 1

De donde se desprenden dos inecuaciones, las cuales son

y2 ≥ 0

y2 − 2 ≤ 0

De la seguna inecuación, se obtiene como solución

S2 = [−√

2,√

2]

Como sabemos que y > 0 (relación obtenida en el item anterior) y considerando todas las restricciones, sigueque

Rec(f) =]0,√

2]

2. Dada f(x) =√|x+ 1| − 2|x− 3| − x. , determine su dominio

Solución: Debe cumplirse que

|x+ 1| − 2|x− 3| − x ≥ 0

|x+ 1| − 2|x− 3| ≥ x

En virtud del problema resuelto 7 de la sección 1.1 , se tiene que

Dom(f) = [5

3,

7

2]

Page 16: Apuntes Resueltos Todo Calculo 1

16 CAPÍTULO 1. PROBLEMAS RESUELTOS.

3. Sea f : A→ B tal que f(x) =x− 1

3x+ 2

a) Determine los conjuntos A y B tal que f(x) sea invertible

Solución:

Para que sea invertible (posea inversa) debe cumplirse que la relación en primero lugar sea función, que seainyectiva y además epiyectiva, entonces:

1) Dominio de la función: claramente se puede observar que Dom(f) = R− − 23 , luego A = R− − 2

32) Análisis de inyectividad: Sean a, b ∈ Dom(f) . Demostraremos que si f(a) = f(b)⇒ a = b , entonces

f(a) =a− 1

3a+ 2=

b− 1

3b+ 2= f(b)

(a− 1)(3b+ 2) = (b− 1)(3a+ 2)

3ab+ 2a− 3b− 2 = 3ab+ 2b− 3a− 2

5a = 5b

a = b

Luego, ∀x ∈ A , la función es inyectiva.3) Análisis de epiyectividad: Debemos encontrar un conjunto tal que B = Rec(f), despejando x usando que

f(x) = y , tenemos

y =x− 1

3x+ 2⇔ y(3x+ 2) = x− 1⇔ 3xy + 2y = x− 1

2y + 1 = x(1− 3y)⇔ x =2y + 1

1− 3y

En donde podemos ver que B = Rec(f) = R− 13

b) Determine f−1(x).

Solución:

En virtud del item anterior, se tiene que

f−1(x) =2x+ 1

1− 3x

4. Dada f(x) = x2 − 1 y g(x) =

2x+ 3 x ≤ 0√x 0 < x ≤ 8

x3 x ≥ 8

. Determine g(f(x)).

Solución:

g(f(x)) =

2f(x) + 3 f(x) ≤ 0√f(x) 0 < f(x) ≤ 8

(f(x))3 f(x) ≥ 8

g(f(x)) =

2(x2 − 1) + 3 x2 − 1 ≤ 0 (1)√x2 − 1 0 < x2 − 1 ≤ 8 (2)

(x2 − 1)3 x2 − 1 ≥ 8 (3)

Page 17: Apuntes Resueltos Todo Calculo 1

1.2. FUNCIONES 17

Resolviendo las inecuaciones (1) , (2) y (3) se tiene:

en (1) : x2 − 1 ≤ 0⇔ x ∈ [−1, 1]en (3) : x2 − 1 ≥ 8⇔ x2 − 9 ≥ 0⇔ x ∈]−∞,−3] ∪ [3,∞[en (2) : 0 < x2 − 1 ≤ 8 , aquí tenemos dos inecuaciones:

a) 0 < x2 − 1⇔ x ∈]−∞,−1[∪]1,∞[

b) x2 − 1 ≤ 8⇔ x2 − 9 ≤ 0⇔ x ∈ [−3, 3]De aquí nace la solución general que es la intersección de ambos casos, lo cual será S(2) = [−3,−1[∪]1, 3]

Finalmente

g(f(x)) =

2(x2 − 1) + 3 x ∈ [−1, 1]√x2 − 1 x ∈ [−3,−1[∪]1, 3]

(x2 − 1)3 x ∈]−∞,−3] ∪ [3,∞[

5. Dada f(x) =√

2−√

2x+ 1

a) Determine Dom(f)

Solución:

Debe cumplirse que

2−√

2x+ 1 ≥ 0

Note que debe complirse que x ≥ − 12 , entonces

2 ≥√

2x+ 1

Elevando al cuadrado

4 ≥ 2x+ 1

3 ≥ 2x

x ≤ 3

2

Realizando las intersecciones correspondientes

Dom(f) =

[−1

2,

3

2

]

b) Demuestre que f(x) es estrictamente decreciente

Solución:

Page 18: Apuntes Resueltos Todo Calculo 1

18 CAPÍTULO 1. PROBLEMAS RESUELTOS.

Consideremos a, b ∈ Dom(f) tal que a < b , luego

a < b

2a < 2b

2a+ 1 < 2b+ 1√

2a+ 1 <√

2b+ 1

−√

2a+ 1 > −√

2b+ 1

2−√

2a+ 1 > 2−√

2b+ 1√2−√

2a+ 1 >

√2−√

2b+ 1

f(a) > f(b)

Luego, podemos ver que efectivamente la función es estrictamente decreciente.

6. Sea f [0, 1[→ R tal que f(x) =1√

1− x2, determine Rec(f)

Solución: Despejando x

y =1√

1− x2

y2 =1

1− x2

y2 − x2y2 = 1

x2 =y2 − 1

y2

x =

√y2 − 1

y2

Como x ∈ [0, 1[ , entonces:

0 ≤√

y2−1y2 < 1

0 ≤ y2−1y2 < 1

0 ≤ y2 − 1 < y2

De donde

y2 ≥ 1

Se puede observar de la función original que y > 0 , para cualquier valor de x en el dominio, luego

y ≥ 1

Por lo tanto

Rec(f) = [1,+∞[

7. Denimos f(x) =√

2 tan x1+tan2 x

a) Muestre que es posible escribir f(x) de una manera más simple.

Page 19: Apuntes Resueltos Todo Calculo 1

1.2. FUNCIONES 19

Solución: Tomaremos el argumento de la función y usando propiedades trigonométricas la escribiremos demanera mas simple, entonces

2 tanx

1 + tan2 x=

2 · sin xcos x

1 + sin2 xcos2 x

=2 sin xcos x

cos2 x+sin2 xcos2 x

=2 sinx

cosx· cos2 x

= 2 sinx cosx

= sin(2x)

Luego f(x) =√

sin(2x)

b) Determine Dom(f)

Solución: para el dominio, se debe cumplir que

sin(2x) ≥ 0

Notemos que , en virtud del círculo unitario:

sin(2x) ≥ 0 ⇔ 2x ∈ [0, π] ∪ [2π, 3π] ∪ [4π, 5π] ∪ ....

⇔ x ∈[0,π

2

]∪[π,

2

]∪[2π,

2

]∪ ....

⇔ x ∈⋃k∈Z

[kπ,

(2k + 1)π

2

]De donde se obtiene lo pedido.

8. Considere la función f(x) = 1+sin x1−cos x

a) Determine el dominio de f

Solución: Se debe cumplir que

cosx 6= 1

Esto se cumple si

x 6= 2kπ

k ∈ Z

Luego

Dom(f) = R− 2kπ

b) Determine los ceros y signo de f .

Solución: para los ceros tenemos que

f(x) = 0 ⇔ 1 + sinx = 0

⇔ sinx = −1

⇔ x =

2,

2,

11π

2, ...

⇔ x =

2+ 2kπ

k ∈ Z

Page 20: Apuntes Resueltos Todo Calculo 1

20 CAPÍTULO 1. PROBLEMAS RESUELTOS.

Para el signo, notemos que | sinx| ≤ 1 y que | cosx| ≤ 1 , por lo tanto:

1 + sinx ≥ 0 ∧ 1− cosx ≥ 0

Luego

1 + sinx

1− cosx≥ 0

De donde se deduce que la función f es siempre positiva.

9. Considere la funciones del tipo f(x) = a cos(wx) + b sin(wx)

a) Demuestre que f puede ser escrita como f(x) =√a2 + b2 sin(wx+ φ).

Solución: Dividiendo la expresión por√a2 + b2 , se tiene

f(x)√a2 + b2

=a√

a2 + b2cos(wx) +

b√a2 + b2

sin(wx)

Si nos formamos un triángulo adecuado con ángulo agudo φ , es posible notar que

cosφ =b√

a2 + b2

sinφ =a√

a2 + b2

Reemplazando

f(x)√a2 + b2

= sinφ cos(wx) + cosφ sin(wx)

f(x) =√a2 + b2 sin(wx+ φ).

b) Usando la misma idea expuesta en a), exprese como función sinusoidal las siguientes curvas:

1) f(x) = sinx− cosx

Solución: Usando la misma idea

f(x)√2

=sinx√

2− cosx√

2

De acuerdo a esto, sabemos que simultáneamente

cosφ =1√2

sinφ =1√2

Luego φ =π

4, por lo que

f(x)√2

= cosπ

4sinx− cosx sin

π

4

f(x) =√

2 sin(x− π

4

)

Page 21: Apuntes Resueltos Todo Calculo 1

1.3. SUCESIONES. 21

2) f(x) = 3 sinx cos2 x− sin3 x+ 12 cos(3x)

Solución: Primero, escribiremos la función de una forma más simple, es decir

f(x) = 2 sinx cos2 x+ sinx cos2 x− sin3 x+1

2cos(3x)

= 2 sinx cosx cosx+ sinx(cos2 x− sin2 x) +1

2cos(3x)

= sin(2x) cosx+ sinx cos(2x) +1

2cos(3x)

= sin(3x) +1

2cos(3x)

Entonces, dividiendo por√

54 , se tiene que

f(x)√54

=sin(3x)√

54

+cos(3x)

2√

54

De donde se desprende que

sinφ =1

2√

54

cosφ =1√

54

En donde tanφ = 12 ⇒ φ = arctan

(12

), luego es posible escribir

f(x) =

√5

4sin

(3x+ arctan

(1

2

))

1.3. Sucesiones.

1. Calcule limn→∞

n3 − 3n2 + 5n− 1

(n− 2)2

Solución:

limn→∞

n3 − 3n2 + 5n− 1

(n− 2)2= limn→∞

n3 − 3n2 + 5n− 1

n2 − 4n+ 4= limn→∞

n3(1− 3

n + 5n2 − 1

n3

)n3(

1n −

4n2 + 4

n3

) =∞

2. Calcular limn→∞

P (n)

Q(n), donde P (n) y Q(n) son polinomios del mismo grado.

Solución:

Consideremos que los polinomios podemos escribirlos de la manera siguiente:

P (n) = a0 + a1n+ a2n2 + a3n

3 + ...+ aknk

Q(n) = b0 + b1n+ b2n2 + b3n

3 + ...+ bknk

Luego, debemos resolver

limn→∞

P (n)

Q(n)= limn→∞

a0 + a1n+ a2n2 + a3n

3 + ...+ aknk

b0 + b1n+ b2n2 + b3n3 + ...+ bknk

Page 22: Apuntes Resueltos Todo Calculo 1

22 CAPÍTULO 1. PROBLEMAS RESUELTOS.

Como los polinomios son de grado k , vamos a factorizar de la manera siguiente

limn→∞

nk(a0nk

+ a1nk−1 + a2

nk−2 + a3nk−3 + ...+ ak

)nk(b0nk

+ b1nk−1 + b2

nk−2 + b3nk−3 + ...+ bk

) = limn→∞

a0

nk+

a1

nk−1+

a2

nk−2+

a3

nk−3︸ ︷︷ ︸→0

+...+ ak

b0nk

+b1nk−1

+b2nk−2

+b3nk−3︸ ︷︷ ︸

→0

+...+ bk

=akbk

3. Sean an , bn y cn sucesiones tales que an = bn − n+1n y cn = 2bn

3 . Si limn→∞

an = 2 . Muestre que cn converge

Solución:

Si aplicamos límite a la primera igualdad, tenemos

limn→∞

an = limn→∞

bn − limn→∞

n+ 1

n

2 = limn→∞

bn − 1⇒ limn→∞

bn = 3

Por otra parte cn = 2bn3 , si hacemos un procedimiento similar

limn→∞

cn =2 · lim

n→∞bn

3=

2 · 33

= 2

Dado que el límite existe, podemos armar que cn converge y lo hace a 2.

4. Calcule limn→∞

n∑k=1

2k

n2

7n

Solución:

Podemos escribir el límite de la manera siguiente

limn→∞

n∑k=1

2k

n2

7n

= limn→∞

(1

n2· 2 · n(n+ 1)

2

)7n

= limn→∞

(n+ 1

n

)7n

= limn→∞

(1 +

1

n

)7n

= e7

5. Calcule los siguientes límites:

a) lımn→∞

(1n

)2+(

2n

)2+(

3n

)2+ ...+

(nn

)2n

Solución: Observe que(1

n

)2

+

(2

n

)2

+

(3

n

)2

+ ...+(nn

)2

=1 + 22 + 32 + ...+ n2

n2

=n(n+1)(2n+1)

6

n2

=n(n+ 1)(2n+ 1)

6n2

Page 23: Apuntes Resueltos Todo Calculo 1

1.3. SUCESIONES. 23

Entonces, reemplazando

lımn→∞

(1n

)2+(

2n

)2+(

3n

)2+ ...+

(nn

)2n

= lımn→∞

n(n+1)(2n+1)6n2

n

= lımn→∞

(n+ 1)(2n+ 1)

6n2

=1

3

b) lımn→∞

1 + 5 + 52 + 53 + ...+ 5n−1

5n + n

Solución: No es dicil notar que

1 + 5 + 52 + 53 + ...+ 5n−1 =

n∑k=1

5k−1

=1− 5n

1− 5

Reemplazando

lımn→∞

1 + 5 + 52 + 53 + ...+ 5n−1

5n + n= lım

n→∞

1−5n

1−5

5n + n

= −1

4lımn→∞

1− 5n

5n + n

= −1

4lımn→∞

15n − 1

1 + n5n

=1

4

6. Calcule lımn→∞

yn , donde yn = Pn es el perímetro de un polígono regular de n lados inscrito en una circunferencia de

radio r.

Solución: Considerar el esquema geométrico correspondiente

Figura 1.3.1: Polígono regular inscrito

Del esquema, denimos como b cada uno de los lados del polígono regular, luego su perímetro será:

Pn = n · b

Page 24: Apuntes Resueltos Todo Calculo 1

24 CAPÍTULO 1. PROBLEMAS RESUELTOS.

el lado b, puede ser calculado, considerando que este polígono está formado por n triángulos isósceles de lads igualesr y altura h, considerando que el ángulo del vértice para esos triángulos son iguales, tenemos que, por trigonometría

sin(a

2

)=

b

2r

Sin embargo, en la gura hay n ángulos de magnitud a , donde todos ellos juntos forman los 360° que posee lacircunferencia, entonces

na = 2π

a =2π

n

reemplazando donde corresponda

b = 2r sin(πn

)nalmente

Pn = 2nr sin(πn

)Calculando el límite

lımn→∞

Pn = lımn→∞

2nr sin(πn

)Sea el cambio t = π

n si n→∞ , ⇒ t→ 0 , luego

lımn→∞

2nr sin(πn

)= 2r lım

t→0

π sin t

t= 2πr

7. Sea (an : n ≥ 1) una sucesión denida por an =(2n)!(2θ + 1)2n

(n!)2. Determine todos los valores de θ ∈ R para los

cuales limn→∞

an+1

an< 1

Solución:

limn→∞

an+1

an= limn→∞

(2n+ 2)!(2θ + 1)2n+2

((n+ 1)!)2· (n!)2

(2n)!(2θ + 1)2n

= limn→∞

(2n+ 2)(2n+ 1)(2n)!(2θ + 1)2n(2θ + 1)2(n!)2

(n+ 1)2(n!)2(2n)!(2θ + 1)2n

= limn→∞

(2n+ 2)(2n+ 1)(2θ + 1)2

(n+ 1)2

= (2θ + 1)2 limn→∞

4n2 + 2n+ 4n+ 2

n2 + 2n+ 1= 4(2θ + 1)2

Como limn→∞

an+1

an< 1 , entonces

4(2θ + 1)2 < 1⇔ |2θ + 1| < 1

2⇔ −1

2< 2θ + 1 <

1

2

−3

2< 2θ < −1

2

−3

4< θ < −1

4

Page 25: Apuntes Resueltos Todo Calculo 1

1.3. SUCESIONES. 25

8. Determine los valores de α ∈ R para los cuales limn→∞

n∑k=1

(α2 − 8)k existe, y calcular dicho límite.

Solución:

Recordemos que la serie geométrica tiene la siguiente estructura limn→∞

n∑k=1

ark−1 =a

1− r, este resultado es factible

si |r| < 1 , entonces

limn→∞

n∑k=1

(α2 − 8)k = limn→∞

∑k=1

(α2 − 8)(α2 − 8)k−1 =α2 − 8

1− (α2 − 8)⇔ |α2 − 8| < 1

Tenemos que resolver la inecuación

|α2 − 8| < 1⇔ −1 < α2 − 8 < 1

En donde tenemos dos casos, los cuales son

a) −1 < α2 − 8⇔ 0 < α2 − 7⇔ α ∈]−∞,−√

7[∪]√

7,∞[

b) α2 − 8 < 1⇔ α2 − 9 < 0⇔ α ∈]− 3, 3[

Realizando las intersecciones correspondientes, nalmente se llega a que

α ∈]− 3,−√

7[∪]√

7, 3[

Donde , para los valores de α en ese conjunto, el valor del límite es

L =α2 − 8

1− (α2 − 8)=α2 − 8

9− α2

9. Considere la sucesión (Pn)n∈N denida por P0 > 0 , Pn+1 =bPn

a+ Pndonde a y b son constantes reales positivas

a) Demuestre que si (Pn)n∈N es convergente, los únicos valores posibles de su límite son 0 y b− a

Solución: Como dicen que la sucesión converge entonces

Pn+1 =bPn

a+ Pn

Aplicando limn→∞

()

limn→∞

Pn+1 =b limn→∞

Pn

a+ limn→∞

Pn⇒ L =

bL

a+ L⇒ L(a+ L) = bL

L(a+ L− b) = 0⇒

L1 = 0L2 = b− a

b) Pruebe que si a > b , entonces (Pn)n∈N es decreciente y converge a cero

Solución: Si a > b analizaremos Pn+1 − Pn:

Pn+1 − Pn =bPn

a+ Pn− Pn =

bPn − aPn − P 2n

a+ Pn=Pn(b− a)− P 2

n

a+ Pn

Como a > b, entonces b− a < 0 por lo tanto Pn(b− a) < 0 y −P 2n < 0 además el denominador es positivo por

lo tanto, podemos concluir que:Pn+1 − Pn < 0⇒ Pn+1 < Pn

Donde concluimos que la sucesión es decreciente y en virtud de (a) tenemos que el límite vale cero.

Page 26: Apuntes Resueltos Todo Calculo 1

26 CAPÍTULO 1. PROBLEMAS RESUELTOS.

c) Suponga ahora que a < b y 0 < P0 < b− a1) Pruebe que 0 < Pn < b− a, ∀n ∈ N y que (Pn)n∈N es creciente.

Solución: Si suponemos que 0 < a < b y 0 < P0 < b− a:

Por inducción tenemos que nuestra hipótesis es:

0 < Pn < b− a

Lo que debemos demostrar es0 < Pn+1 < b− a

En efecto, de la hipótesis0 < Pn < b− a

multiplicando por b0 < bPn < b(b− a)

Dividiendo por a+ Pn

0 <bPn

a+ Pn<b(b− a)

a+ PnPor transitividad:

0 <bPn

a+ Pn<b(b− a)

a+ Pn< b− a⇒ 0 < Pn+1 < b− a

Por lo tanto, queda demostrado.

Analizaremos la expresión:Pn+1

Pn, luego:

Pn+1

Pn=

bPn(a+ Pn)Pn

=b

a+ Pn

Como Pn < b− a⇒ a+ Pn < b, entonces

Pn+1

Pn> 1⇒ Pn+1 > Pn

Lo que demuestra que la sucesión es creciente.2) Determine lim

n→∞Pn

Solución: en virtud de la parte (a), el límite vale b− a.

10. Considere xn =

n∑k=0

1

k!y además yn = xn +

1

n · n!

a) Demuestre que ambas sucesiones son monótonas

Solución:

Analizaremos la diferencia xn+1 − xn , luego

xn+1 − xn =

n+1∑k=0

1

k!−

n∑k=0

1

k!

xn+1 − xn =

n∑k=0

1

k!+

1

(n+ 1)!−

n∑k=0

1

k!

xn+1 − xn =1

(n+ 1)!> 0

Page 27: Apuntes Resueltos Todo Calculo 1

1.3. SUCESIONES. 27

Por lo tanto, la sucesión xn es creciente. Por otro lado

yn+1 − yn = xn+1 +1

(n+ 1) · (n+ 1)!−(xn +

1

n · n!

)yn+1 − yn = xn+1 − xn +

1

(n+ 1) · (n+ 1)!− 1

n · n!

yn+1 − yn =1

(n+ 1)!+

1

(n+ 1) · (n+ 1)!− 1

n · n!

yn+1 − yn =1

(n+ 1) · n!+

1

(n+ 1)2 · n!− 1

n · n!

yn+1 − yn =n(n+ 1) + n− (n+ 1)2

n(n+ 1)2n!

yn+1 − yn =n2 + n+ n− n2 − 2n− 1

n(n+ 1)2n!= − 1

n(n+ 1)2n!< 0

Por lo tanto, la sucesión yn es decreciente.

b) Demuestre que ambas sucesiones convergen al mismo límite

Solución:

Debemos analizar la igualdad que relaciona ambas sucesiones, a saber

yn = xn +1

n · n!

yn − xn =1

n · n!

Aplicando límite

limn→∞

yn − limn→∞

xn = limn→∞

1

n · n!

limn→∞

yn − limn→∞

xn = 0⇒ limn→∞

yn = limn→∞

xn

Como los limites son iguales, entonces ambas sucesiones xn e yn convergen al mismo valor.

11. Sea a ∈ [0, 1] y ann∈N denida por a1 = a y an+1 = an(2− an)

a) Demuestre que ∀x ∈ R , x(2− x) < 1

Solución:

(x− 1)2 > 0

x2 − 2x+ 1 > 0

−1 + 2x− x2 < 0

2x− x2 < 1

x(2− x) < 1

b) Use inducción matemática para probar que ∀n ≥ 1 , an ∈]0, 1[ .

Solución:

Page 28: Apuntes Resueltos Todo Calculo 1

28 CAPÍTULO 1. PROBLEMAS RESUELTOS.

Demostraremos que an ∈]0, 1[ , en efecto si n = 1 se tiene que a1 ∈]0, 1[ lo cual es correcto por los datosdel problema. Si n = k (hipótesis)

ak ∈]0, 1[

Si n = k + 1 (tesis)

ak+1 ∈]0, 1[

Demostración: de la hipótesis

0 < ak < 1

0 < a2k < ak

0 > −a2k > −ak

2ak > 2ak − a2k > ak

ak < ak(2− ak) < 2ak

Notemos que , del item (a) , ak(2− ak) < 1 , además, ak ∈]0, 1[ (hipótesis de inducción) , usando transitividad

0 < ak(2− ak) < 1

ak+1 ∈]0, 1[

Así, queda demostrado lo pedido.

c) Demuestre que ann∈N es creciente

Solución:

Analizaremos la diferencia an+1 − an , luego

an+1 − an = an(2− an)− an = an(2− an − 1) = an(1− an)

Como an ∈]0, 1[ , an ∈ R+ y además 1− an > 0 , entonces

an+1 − an = an(1− an) > 0⇒ an+1 − an > 0⇒ an+1 > an

Por lo tanto, la sucesión ann∈N es creciente.

d) Deduzca la existencia de L = limn→∞

an y determine L.

Solución:

Det teorema de las sucesiones monótonas, se tiene que al ser creciente y acotada, entonces la sucesión tienelímite, luego

an+1 = an(2− an)

Aplicando límite

limn→∞

an+1 = limn→∞

an(2− limn→∞

an)

L = L(2− L)

L = 2L− L2

L2 − L = 0

L(L− 1) = 0⇒

L1 = 0

L2 = 1

Page 29: Apuntes Resueltos Todo Calculo 1

1.3. SUCESIONES. 29

Como la sucesión es creciente y acotada superiormente, el límite es el supremo del conjunto, luego se concluyeque el valor del límite es L = 1

12. Calcule limn→∞

(1− n2

n2√n2 + 1

+2− n2

n2√n2 + 2

+ ...+n− n2

n2√n2 + n

)Solución:

Sea an = 1−n2

n2√n2+1

+ 2−n2

n2√n2+2

+ ...+ n−n2

n2√n2+n

, luego, por acotamiento se tiene

1− n2

n2√n2 + n

+2− n2

n2√n2 + n

+ ...+n− n2

n2√n2 + n

≤ an ≤1− n2

n2√n2 + 1

+2− n2

n2√n2 + 1

+ ...+n− n2

n2√n2 + 1

(1 + 2 + ...+ n)− (n2 + n2 + ...+ n2)

n2√n2 + n

≤ an ≤(1 + 2 + ...+ n)− (n2 + n2 + ...+ n2)

n2√n2 + 1

n(n+1)2 − n3

n2√n2 + n

≤ an ≤n(n+1)

2 − n3

n2√n2 + 1

n2 + n− 2n3

2n2√n2 + n

≤ an ≤n2 + n− 2n3

2n2√n2 + 1

Aplicando límite a la desigualdad

limn→∞

n2 + n− 2n3

2n2√n2 + n

≤ limn→∞

an ≤ limn→∞

n2 + n− 2n3

n2√n2 + 1

limn→∞

n3(

1n + 1

n2 − 2)

n3(

2√

1 + 1n

) ≤ limn→∞

an ≤n3(

1n + 1

n2 − 2)

n3(

2√

1 + 1n2

)−1 ≤ lim

n→∞an ≤ −1⇒ lim

n→∞an = −1

13. Calcule limn→∞

n+ sin(nπ2

)2n+ 1

Solución:

Utilizando acotamiento, tenemos que ∣∣∣sin(nπ2

)∣∣∣ ≤ 1

−1 ≤ sin(nπ

2

)≤ 1

n− 1 ≤ n+ sin(nπ

2

)≤ n+ 1

n− 1

2n− 1≤n+ sin

(nπ2

)2n+ 1

≤ n+ 1

2n+ 1

Aplicando límite

limn→∞

n− 1

2n+ 1≤ limn→∞

n+ sin(nπ2

)2n+ 1

≤ limn→∞

n+ 1

2n+ 1

1

2≤ limn→∞

n+ sin(nπ2

)2n+ 1

≤ 1

2⇒ lim

n→∞

n+ sin(nπ2

)2n+ 1

=1

2

14. Calcule limn→∞

an , donde an = [α]+[2α]+[3α]+...+[nα]n2 , con α > 0 y [.] simboliza la parte entera.

Solución: Se sabe que por propiedad de la función parte entera

ψ − 1 < [ψ] ≤ ψ

Page 30: Apuntes Resueltos Todo Calculo 1

30 CAPÍTULO 1. PROBLEMAS RESUELTOS.

Utilizando esto

(α− 1) + (2α− 1) + ...+ (nα− 1) < [α] + [2α] + ...+ [nα] ≤ α+ 2α+ ...+ nα

(α− 1) + (2α− 1) + ...+ (nα− 1)

n2< [α]+[2α]+...+[nα]

n2 ≤ α+ 2α+ ...+ nα

n2

limn→∞

α

(n∑k=1

k

)− n

n2< limn→∞ an ≤ lim

n→∞

α

(n∑k=1

k

)n2

limn→∞

αn(n+1)2 − nn2

< limn→∞ an ≤ limn→∞

αn(n+1)2

n2

limn→∞

αn+ α− 2

2n< limn→∞ an ≤ lim

n→∞

αn(n+ 1)

2n2

α

2< limn→∞ an ≤ α

2

De donde se concluye que

limn→∞

[α] + [2α] + ...+ [nα]

n2=α

2

15. Calcule limn→∞

1

n

[e

1n +

(e

1n

)2

+(e

1n

)3

+ ...+(e

1n

)n], Indicación: [.] denota paréntesis normal.

Solución: Notemos que

1

n

[e

1n +

(e

1n

)2

+(e

1n

)3

+ ...+(e

1n

)n]=

1

n

n∑k=1

(e

1n

)k=

e1n

n

n∑k=1

(e

1n

)k−1

=e

1n

1−(e

1n

)n1− e 1

n

= e1n (1− e) ·

1n

1− e 1n

Entonces

limn→∞

1

n

[e

1n +

(e

1n

)2

+(e

1n

)3

+ ...+(e

1n

)n]= lim

n→∞e

1n (1− e) ·

1n

1− e 1n

Sea el cambio u = 1n si n→∞ , entonces u→ 0 , luego

limu→0

eu(1− e) u

1− eu= (1− e) lim

u→0eu · −u

eu − 1= e− 1

16. Sea (wn)n∈N una sucesión acotada, se dene la sucesión (γn)n∈N mediante

γn =w1 + 2w2 + ...+ nwn

1 + 2 + ...+ n

Demuestre que γn también es acotada.

Solución: Si wn es acotada, enconces existe un M > 0 tal que

|wn| ≤ M

Page 31: Apuntes Resueltos Todo Calculo 1

1.4. LÍMITE Y CONTUINIDAD 31

Entonces

|γn| =

∣∣∣∣∣∣∣∣∣∣

n∑k=1

kwk

n∑k=1

k

∣∣∣∣∣∣∣∣∣∣Ahora |wk| ≤M , entonces

|γn| ≤

∣∣∣∣∣∣∣∣∣∣

n∑k=1

kM

n∑k=1

k

∣∣∣∣∣∣∣∣∣∣≤ =

∣∣∣∣∣∣∣∣∣∣M

n∑k=1

k

n∑k=1

k

∣∣∣∣∣∣∣∣∣∣|γn| ≤ M

Con lo cual se prueba que la sucesión (γn)n∈N es acotada.

1.4. Límite y Contuinidad

1. Calcule los siguientes límites:

a) limx→1

x2 − 2x+ 1

x3 − x

Solución: Si evaluamos directamente nos encontramos con una indeterminación de tipo ( 00 ) , arreglando

tenemos

limx→1

x2 − 2x+ 1

x3 − x= lim

x→1

(x− 1)2

x(x− 1)(x+ 1)

limx→1

x− 1

x(x+ 1)= 0

b) limx→0

√1 + x2 − 1

x

Solución: De manera análoga

limx→0

√1 + x2 − 1

x·√

1 + x2 + 1√1 + x2 + 1

= limx→0

1 + x2 − 1

x(√

1 + x2 + 1)

= limx→0

x2

x(√

1 + x2 + 1) = lim

x→0

x√1 + x2 + 1

= 0

c) limx→1

3√

7 + x3 −√

3 + x2

x− 1

Page 32: Apuntes Resueltos Todo Calculo 1

32 CAPÍTULO 1. PROBLEMAS RESUELTOS.

Solución: arreglando conveniéntemente la expresión mediante la separación de dos radicales diferentes tenemos

limx→1

3√

7 + x3 −√

3 + x2

x− 1= lim

x→1

3√

7 + x3 −√

3 + x2 + 2− 2

x− 1

= limx→1

[3√

7 + x3 − 2

x− 1−√

3 + x2 − 2

x− 1

]= lim

x→1

3√

7 + x3 − 2

x− 1− limx→1

√3 + x2 − 2

x− 1

Racionalizando en forma separada ambos casos tenemos

= limx→1

3√

7 + x3 − 2

x− 1·

3√

(7 + x3)2 + 2 3√

7 + x3 + 43√

(7 + x3)2 + 2 3√

7 + x3 + 4− limx→1

√3 + x2 − 2

x− 1·√

3 + x2 + 2√3 + x2 + 2

= limx→1

x3 − 1

(x− 1)(

3√

(7 + x3)2 + 2 3√

7 + x3 + 4) − lim

x→1

x2 − 1

(x− 1)(√

3 + x2 + 2)

= limx→1

x2 + x+ 13√

(7 + x3)2 + 2 3√

7 + x3 + 4− limx→1

x+ 1√3 + x2 + 2

=3

3 · 4− 2

4= −1

4

d) limx→0

(1 +mx)n − (1 + nx)m

x2

Solución: Usando el Binomio de Newton

limx→0

(1 +mx)n − (1 + nx)m

x2= lim

x→0

n∑k=0

(n

k

)(mx)k −

m∑k=0

(m

k

)(nx)k

x2

= limx→0

n(n−1)2 m2x2 + ...+mnxn −

(m(m−1)

2 n2x2 + ...+ nmxm)

x2

= lim

x→0

[n(n− 1)m2

2+ ..+mnxn−2 −

(m(m− 1)n2

2+ ..+ nmxm−2

)]=

nm(n−m)

2

e) limx→1

xn+1 − (n+ 1)x+ n

(x− 1)2

Page 33: Apuntes Resueltos Todo Calculo 1

1.4. LÍMITE Y CONTUINIDAD 33

Solución: Haciendo el cambio u = x− 1 se puede ver que si x→ 1⇒ u→ 0 , luego

limx→1

xn+1 − (n+ 1)x+ n

(x− 1)2

u=x−1︷︸︸︷= lim

u→0

(u+ 1)n+1 − (n+ 1)(u+ 1) + n

u2

= limu→0

n+1∑k=0

(n+ 1

k

)u(n+1)−k − u(n+ 1)− (n+ 1) + n

u2

= limu→0

(n+1n+1

)+(n+1n

)u+

(n+1n−1

)u2 + ...+ un+1 − u(n+ 1)− (n+ 1) + n

u2

= limu→0

1 + u(n+ 1) + 12n(n+ 1)u2 + ...+ un+1 − u(n+ 1)− (n+ 1) + n

u2

= limu→0

1

u2

(1

2n(n+ 1)u2 + ...+ un+1

)

= limu→0

n(n+ 1)

2+...+ un−1︸ ︷︷ ︸

→0

=

n(n+ 1)

2

f ) limx→0

sin(7x)− sin(5x)

sin(3x)− x

Solución:

limx→0

sin(7x)− sin(5x)

sin(3x)− x= lim

x→0

7 sin(7x)7x − 5 sin(5x)

5x3 sin(3x)

3x − 1

=7− 5

3− 1= 1

g) Calcule limx→0

x sinx

1− cosx

Solución: Arreglando la expresión

limx→0

x sinx

1− cosx= lim

x→0

x sinx · (1 + cosx)

sin2 x

= limx→0

sin xx · (1 + cosx)(

sin xx

)2= 2

h) limx→0

tanx− sinx

x3

Solución:

limx→0

tanx− sinx

x3= lim

x→0

sin xcos x − sinx

x3

= limx→0

sinx− sinx · cosx

x3 cosx

= limx→0

sinx · (1− cosx)

x3 cosx· 1 + cosx

1 + cosx

= limx→0

sin3 x

x3 cosx(1 + cosx)=

1

2

Page 34: Apuntes Resueltos Todo Calculo 1

34 CAPÍTULO 1. PROBLEMAS RESUELTOS.

i) limx→0

√1 + sinx−

√1− tanx

sin(2x)

Solución:

limx→0

√1 + sinx−

√1− tanx

sin(2x)= lim

x→0

√1 + sinx−

√1− tanx

sin(2x)·√

1 + sinx+√

1− tanx√1 + sinx+

√1− tanx

= limx→0

1 + sinx− 1 + tanx

sin(2x)(√

1 + sinx+√

1− tanx)

= limx→0

sinx+ tanx

2 sinx · cosx(√

1 + sinx+√

1− tanx)

= limx→0

sin x cos x+sin xcos x

2 sinx cosx(√

1 + sinx+√

1− tanx)

= limx→0

sinx(cosx+ 1)

2 sinx cos2 x(√

1 + sinx+√

1− tanx)

= limx→0

cosx+ 1

2 cos2 x(√

1 + sinx+√

1− tanx)

=2

2(2)=

1

2

j ) limx→π

3

2 cosx− 1

x− π3

Solución: Realzando el cambio u = x− π3 se tiene que si x→ π

3 ⇒ u→ 0 , luego

limx→π

3

2 cosx− 1

x− π3

u=x−π3︷︸︸︷= lim

u→0

2 cos(u+ π

3

)− 1

u

= limu→0

2(cosu · cos π3 − sinu · sin π

3

)− 1

u

= limu→0

2(

12 cosu−

√3

2 sinu)− 1

u

= limu→0

[cosu− 1

u−√

3 · sinu

u

]= −

√3

k) Calcule limx→2

sin(πx)

(x− 2) cos(πx)

Solución: Haciendo el cambio t = x− 2, si x→ 2 entonces u→ 0 , luego

limx→2

sin(πx)

(x− 2) cos(πx)= lim

t→0

sin(π(t+ 2))

t cos(π(t+ 2))

= limt→0

sin(πt+ 2π)

t cos(πt+ 2π)

= limt→0

π sin(πt)

πt· 1

cos(πt+ 2π)= π

l) limx→π

4

cosx− sinx

ex−π4 − 1

Page 35: Apuntes Resueltos Todo Calculo 1

1.4. LÍMITE Y CONTUINIDAD 35

Solución: Realizando el cambio u = x− π4 , si x→ π

4 ⇒ u→ 0 , luego

limx→π

4

cosx− sinx

ex−π4 − 1

u=x−π4︷︸︸︷= lim

u→0

cos(u+ π

4

)− sin

(u+ π

4

)eu − 1

= limu→0

cosu cos π4 − sinu sin π4 −

(sinu cos π4 + cosu sin π

4

)eu − 1

= limu→0

1u

(√2

2 (cosu− sinu− sinu− cosu))

eu−1u

=

√2

2limu→0

− 2 sinuu

eu−1u

= −√

2

m) limx→0

ex − e−x

ln(1− x)

Solución:

limx→0

ex − e−x

ln(1− x)= lim

x→0

ex − e−x + 1− 1

ln(1− x)

= limx→0

ex−1x − e−x−1

x

ln(1− x)1x

= −2

n) Calcule limx→0

ex2 − 1

ln(1 + x2)

Solución: Sea el cambio x2 = t , si x→ 0 , entonces t→ 0 , luego

limt→0

et − 1

ln(1 + t)= lim

t→0

et−1t

1t ln(1 + t)

= limt→0

et−1t

ln(1 + t)1t

= 1

2. En una circunferencia dada de radio r y centro F , AB es una cuerda y ABCD es un rectángulo inscrito en lacircunferencia. Calcule

limAB→0

Area sector circular AFBArea rectánculo ABCD

Solución: Planteamiento de la gura

Figure 1.4.1: Diagrama del problema

Page 36: Apuntes Resueltos Todo Calculo 1

36 CAPÍTULO 1. PROBLEMAS RESUELTOS.

De la gura, se puede deducir que x = r cos θ2 y que z = r sin θ2 con ello se sabe ademas que el área del rectángulo

AR = 4xz = 4r2 cos θ2 · sinθ2 , además el área del sector circular es AS = r2θ

2 , y si AB → 0⇒ θ → 0 , luego

limAB→0

Area sector circular AFBArea rectánculo ABCD

= limθ→0

r2θ

2· 1

4r2 cos θ2 · sinθ2

=1

4limθ→0

θ2

sin θ2

· secθ

2=

1

4

3. En una circunferencia dada de radio r y de centro F , AB es una cuerda y C es el punto medio del arco AB. AdemásD y E son las intersecciones de la tangente en C con las rectas FA y FB , respectivamente. Calcule:

limAB→0

Área del trapecio ABDE

Área del sector circular AB

Se tiene:

AB = 2r sin

2

)DE = 2r tan

2

)Ahora la altura del trapecio h es:

h = r

(1− cos

2

))Entonces el área del trapecio es:

Atrapecio =1

2(DE +AB)

Atrapecio = r2

(1− cos

2

))(tan

2

)+ sin

2

))Por otro lado el área del sector circular:

As =r2θ

2

Luego si AB → 0 ⇒ θ → 0, entonces tenemos lo siguiente:

limAB→0

Área del trapecio ABDE

Área del sector circular AB= lim

θ→0

1− cos θ2θ2

[tan

2

)+ sin

2

)]= 0

4. Sean θ < β las soluciones de la ecuación x2 − 2ax+ b2 = 0 , donde a > b > 0 . Calcule limb→a

aβ − b2

aθ − b2

Solución: Debemos resolver la ecuación para hallar los valores de θ y β , luego

x2 − 2ax+ b2 = 0 ⇔ x =2a±

√4a2 − 4b2

2

⇔ x = a±√a2 − b2

Page 37: Apuntes Resueltos Todo Calculo 1

1.4. LÍMITE Y CONTUINIDAD 37

Por lo tanto θ = a−

√a2 − b2

β = a+√a2 − b2

Reemplazando

limb→a

aβ − b2

aθ − b2= lim

b→a

a(a+√a2 − b2

)− b2

a(a−√a2 − b2

)− b2

= limb→a

a2 − b2 +√a2 − b2

a2 − b2 −√a2 − b2

= limb→a

√a2 − b2

[√a2 − b2 + 1

]√a2 − b2

[√a2 − b2 − 1

]= lim

b→a

√a2 − b2 + 1√a2 − b2 − 1

= −1

5. Considere la función f(x) =x2e

2x

x− 1

a) Calcule limx→1+

f(x) , limx→1−

f(x) , limx→0+

f(x) , limx→0−

f(x) e interprete sus resultados.

Solución:

limx→1+

x2e2x

x− 1= +∞

limx→1−

x2e2x

x− 1= −∞

Esto indica que estamos en presencia de una asíntota vertical en x = 1 , por otro lado

limx→0+

x2e2x

x− 1

1x=u︷︸︸︷= lim

u→+∞

(eu

u

)21u − 1

=∞

limx→0−

x2e2x

x− 1

1x=u︷︸︸︷= lim

u→−∞

(eu

u

)21u − 1

= 0

Luego , la función es asintótica verticalmmente pero sólo por la derecha.

b) Determine la asíntota oblicua hacia +∞ de f(x)

Solución: La asíntota oblicua de f(x) tiene la forma y = mx+n , dondem = limx→+∞

f(x)

xy n = lim

x→+∞(f(x)−mx),

entonces:

m = limx→+∞

f(x)

x= lim

x→+∞

x2e2x

x(x− 1)

= limx→+∞

xe2x

x− 1= 1

Page 38: Apuntes Resueltos Todo Calculo 1

38 CAPÍTULO 1. PROBLEMAS RESUELTOS.

Por lo tanto

n = limx→+∞

(f(x)−mx) = limx→+∞

(x2e

2x

x− 1− x

)

= limx→+∞

(x2e

2x − x(x− 1)

x− 1

)

= limx→+∞

(x2

(e

2x − 1

x− 1

)+

x

x− 1

)

Sea v = 1x , si x→ +∞⇒ v → 0 , entonces

limx→+∞

(x2

(e

2x − 1

x− 1

)+

x

x− 1

) v= 1x︷︸︸︷

= limv→0

(1

v2

(e2v − 1

1−vv

)+

1v

1−vv

)

= limv→0

(e2v−1v

1− v+

1

1− v

)

= limv→0

[2

1− v

(e2v − 1

2v

)+

1

1− v

]= 3

Luego, n = 3 , por lo tanto, la asíntota oblícua es y = x+ 3

6. Analice la existencia de asíntotas horizontales y verticales de f(x) =x√

x2 − 1

Solución: En primer lugar Dom(f) = x ∈] − ∞,−1[∪]1,+∞[ en particular, se analizará que ocurre en los ex-tremos del intervalo, a saber

limx→−1

f(x) = limx→−1

x√x2 − 1

= −∞

Por lo tanto, la recta x = −1 es asíntota vertical, ahora

limx→1

f(x) = limx→1

x√x2 − 1

= +∞

Por lo tanto, x = 1 es también una asíntota vertical.En el caso de las asíntotas horizontales, debemos analizar que sucede hacia ±∞ luego debemos resolver

limx→±∞

f(x) = limx→±∞

x√x2 − 1

= limx→±∞

x

|x|√

1− 1x2

Aquí tenemos dos casos ; si x→ −∞ , |x| = −x , entonces

limx→−∞

x

|x|√

1− 1x2

= limx→−∞

x

−x√

1− 1x2

= limx→−∞

−1√1− 1

x2

= −1

Page 39: Apuntes Resueltos Todo Calculo 1

1.4. LÍMITE Y CONTUINIDAD 39

Lo cual nos conduce a concluir que si x → −∞ , y = −1 es una asíntota horizontal. Por otro lado, si x → +∞ ,|x| = x, luego

limx→+∞

x

|x|√

1− 1x2

= limx→+∞

x

x√

1− 1x2

= limx→+∞

1√1− 1

x2

= 1

Por lo tanto, si x→ +∞ , y = 1 es asíntota horizontal.

7. Encuentre todas las asíntotas de f(x) =x2 + 2x− 1

x2

Solución: Para la función f(x) =x2 + 2x− 1

x2tenemos que Dom(f) = R− 0. Por lo tanto, x = 0, es candidato a

asíntota vertical. Analizando limx→0

f(x):

limx→0

x2 + 2x− 1

x2= −1

0= −∞

Lo cual muestra que efectivamente x = 0 es asíntota horizontal. Para la asíntota horizontal, debemos analizarlim

x→±∞f(x)

limx→±∞

x2 + 2x− 1

x2= 1

Luego, la recta y = 1 es asíntota horizontal tanto para ∞ como para −∞.

8. Encuentre todas las asíntotas de la función f(x) =2(x− 1)2

√4x2 + 2x+ 1

Solución: Para esta función, el Dom(f) = R ya que el denominador posee un discriminante negativo y porende siempre es positivo, por lo tanto, no hay una restricción del dominio. Eso nos muestra que la función no poseeasíntotas verticales.

Asíntotas horizontales: se ve que el numerador posee un grado mayor que el denominador, por lo tanto, al hacer cre-cer indenidamente x, tenemos que el numerador crece más rápido que el denominador, por lo tanto lim

x→±∞f(x) =∞

y no existen asíntotas horizontales.

Esto nos lleva a preguntarnos si existen asíntotas oblícuas. Para encontrar la pendiente:

m = limx→±∞

f(x)

x= limx→±∞

2x2 − 4x+ 2

x√

4x2 + 2x+ 1= limx→±∞

x2(2− 4

x + 2x2

)x|x|

√4 + 2

x + 1x2

Aquí tenemos dos casos:

Caso 1:

m1 = limx→−∞

x2(2− 4

x + 2x2

)x(−x)

√4 + 2

x + 1x2

= limx→−∞

−(2− 4

x + 2x2

)√4 + 2

x + 1x2

= −1

Para el caso 2:

m2 = limx→∞

x2(2− 4

x + 2x2

)x(x)

√4 + 2

x + 1x2

= limx→∞

(2− 4

x + 2x2

)√4 + 2

x + 1x2

= 1

Luego para calcular n, tenemos:n = lim

x→±∞(f(x)−mx)

Page 40: Apuntes Resueltos Todo Calculo 1

40 CAPÍTULO 1. PROBLEMAS RESUELTOS.

Para el caso 1:

n1 = limx→−∞

(2x2 − 4x+ 2√4x2 + 2x+ 1

+ x

)= limx→−∞

(2x2 − 4x+ 2 + x

√4x2 + 2x+ 1√

4x2 + 2x+ 1

)

= limx→−∞

x2(2− 4

x + 2x2 + 1

x

√4x2 + 2x+ 1

)x2√

4x2 + 2

x3 + 1x4

= limx→−∞

2− 4x + 2

x2 +√

4 + 2x + 1

x2√4x2 + 2

x3 + 1x4

= ∞

Para el caso 2:

n2 = limx→∞

(2x2 − 4x+ 2√4x2 + 2x+ 1

− x)

= limx→∞

((2x2 − 4x+ 2)− x

√4x2 + 2x+ 1√

4x2 + 2x+ 1

)· (2x2 − 4x+ 2) + x

√4x2 + 2x+ 1

(2x2 − 4x+ 2) + x√

4x2 + 2x+ 1

n2 = limx→∞

(4(x− 1)4 − x(4x2 + 2x+ 1)√

4x2 + 2x+ 1((2x2 − 4x+ 2) + x

√4x2 + 2x+ 1

))

= limx→∞

4(x− 1)4 − x(4x2 + 2x+ 1)

(2x2 − 4x+ 2)√

4x2 + 2x+ 1 + x(4x2 + 2x+ 1)= ∞

Por ende, no hay asíntotas oblicuas para f .

9. Demuestre que las rectas y = ± bxa son las asíntotas oblícuas de la hipérbola x2

a2 −y2

b2 = 1

Solución: Recordemos que la recta y = mx+ n es una asíntota oblícua de f cuando se cumple

limx→±∞

(f(x)− (mx+ n)) = 0

Analizando la hipérbola

x2

a2− y2

b2= 1

y =b

a

√x2 − a2

Luego, para x→ +∞

limx→+∞

(b

a

√x2 − a2 − bx

a

)= lim

x→+∞

b

a

(√x2 − a2 − x

)·√x2 − a2 + x√x2 − a2 + x

= limx→+∞

b

a

(−a2

√x2 − a2 + x

)= 0

Si x→ −∞

limx→−∞

(b

a

√x2 − a2 −

(−bxa

))= lim

x→∞

b

a

(√x2 − a2 − x

)= 0

Por lo tanto se demuestra lo pedido.

Page 41: Apuntes Resueltos Todo Calculo 1

1.4. LÍMITE Y CONTUINIDAD 41

10. ¾Para qué valores de a y b la función f(x) =

ax+ 2b, x ≤ 0x2 + 3a− b, 0 < x ≤ 2

3x− 5, x > 2es contínua en todos los reales ?.

Solución: Análisis para el punto x = 0f(0) = 2b

Analizando los límites laterales en torno a ese punto:

limx→0−

(ax+ 2b) = 2b, limx→0+

(x2 + 3a− b) = 3a− b

Para que el límite exista, deben ser iguales, luego:

2b = 3a− b⇒ 3b− 3a = 0

Análisis del punto x = 2f(2) = 4 + 3a− b

Analizando los límites laterales en torno al punto se tiene:

limx→2−

(x2 + 3a− b) = 4 + 3a− b, limx→2+

(3x− 5) = 1

Para que el límite exista, deben ser iguales los laterales, entonces

3a− b = −3

Luego, tenemos el sistema: 3a− b = −33b− 3a = 0

Cuyas soluciones son: a = − 3

2b = − 3

2

11. Encuentre los valores de a y b de modo que la función f(x) =

x2

√1+x2−1

x < 0

ax+ b 0 ≤ x ≤ 2x−√x+2√

4x+1−3x > 2

sea contínua en todo punto

de su dominio.

Solución: En todo su dominio, la función f está bien denida, a excepción de los puntos x = 0 y x = 2, analizandolos límites laterales en torno al cero y al dos :

limx→0−

f(x) = limx→0−

x2

√1 + x2 − 1

Este límite posee una forma indeterminada del tipo ( 00 ) , entonces

limx→0−

x2

√1 + x2 − 1

= limx→0−

x2

√1 + x2 − 1

·√

1 + x2 + 1√1 + x2 + 1

= limx→0−

x2(√

1 + x2 + 1)

1 + x2 − 1= 2

Por otro lado

limx→0+

(ax+ b) = b

Page 42: Apuntes Resueltos Todo Calculo 1

42 CAPÍTULO 1. PROBLEMAS RESUELTOS.

De donde se concluye que b = 2 , en x = 2 se tiene que

limx→2−

(ax+ b) = 2a+ b

Por otro lado

limx→2+

x−√x+ 2√

4x+ 1− 3= lim

x→2+

x−√x+ 2√

4x+ 1− 3· x+

√x+ 2

x+√x+ 2

·√

4x+ 1 + 3√4x+ 1 + 3

= limx→2+

(x2 − x− 2

) (√4x+ 1 + 3

)(4x+ 1− 9)

(x+√x+ 2

)= lim

x→2+

(x− 2)(x+ 1)(√

4x+ 1 + 3)

4(x− 2)(x+√x+ 2

)= lim

x→2+

(x+ 1)(√

4x+ 1 + 3)

4(x+√x+ 2

) =18

16=

9

8

Luego, como los límites laterales deben ser iguales

2a+ b =9

8

2a =9

8− 2

a = − 7

16

12. Dada la función f(x) =

√x2 + 9− 3

x2, x < 0

sin(x)

6x, x > 0

. ¾Existe limx→0

f(x)?

Solución: Por límites laterales en torno al cero:

limx→0−

√x2 + 9− 3

x2·√x2 + 9 + 3√x2 + 9 + 3

= limx→0−

1√x2 + 9 + 3

=1

6

Por otra parte:

limx→0+

sinx

6x=

1

6limx→0+

sinx

x=

1

6

Como los límites laterales son iguales, entonces, existe, y su valor es:

limx→0

f(x) =1

6

13. Determine los valores de c ∈ R para los cuales f(x) es continua en x = 0, donde f(x) =

x3−πx

x+sin(3x) x 6= 0

c x = 0

Solución: Para que la función sea contínua en x = 0 , se debe cumplir que

limx→0

f(x) = f(0) = c

Luego

limx→0

f(x) = limx→0

x3 − πxx+ sin(3x)

= limx→0

x(x2 − π)

x(

1 + sin(3x)x

)= lim

x→0

x2 − π1 + 3·sin(3x)

3x

= −π4

= c

Page 43: Apuntes Resueltos Todo Calculo 1

1.4. LÍMITE Y CONTUINIDAD 43

14. Sea f : Dom(f) ⊂ R→ R , denida por f(x) =

√x+3−

√3x+1√

1−x x < 1

ax+ b 1 ≤ x ≤ πsin(3x)π−x x > π

a) Determine el dominio de f

Solución: Dado que poseemos expresiones con raíces de indice par cuando x < 1 , es necesario que si-multaneamente se cumpla que

x+ 3 ≥ 0 ⇔ x ≥ −3

3x+ 1 ≥ 0 ⇔ x ≥ −1

31− x > 0 ⇔ x < 1

Haciendo las intersecciones corresnpondientes, y notando que para los otros intervalos f está bien denidaconcluimos que Dom(f) =

[− 1

3 ,+∞[

b) Encuentre los valores de a y b para que f sea contínua en x = 1 y x = π

Solución: Analizaremos los límites laterales primero en torno a x = 1 , luego

limx→1−

f(x) = limx→1−

√x+ 3−

√3x+ 1√

1− x·√x+ 3 +

√3x+ 1√

x+ 3 +√

3x+ 1

= limx→1−

2(1− x)√1− x

(√x+ 3 +

√3x+ 1

)= lim

x→1−

2√

1− x√x+ 3 +

√3x+ 1

= 0

Luego

limx→1+

f(x) = limx→1+

(ax+ b)

= a+ b

Como los límites laterales deben ser iguales, se tiene que a+ b = 0. Por otro lado el análisis de f en x = π setiene

limx→π−

f(x) = limx→π−

(ax+ b)

= aπ + b

Y por el lado derecho se tiene

limx→π+

f(x) = limx→π+

sin(3x)

π − x

Realizando el cambio u = π − x si x→ π+ ⇒ u→ 0+ , luego

limx→π+

sin(3x)

π − x

u=π−x︷︸︸︷= lim

u→0+

sin (3(π − u))

u

= limu→0+

sin(3u)

u

= limu→0+

3 · sin(3u)

u= 3

Page 44: Apuntes Resueltos Todo Calculo 1

44 CAPÍTULO 1. PROBLEMAS RESUELTOS.

Por lo tanto, procediendo de la forma que la anterior llegamos a que aπ + b = 3 , con lo cual debemos resolverel sistema

a+ b = 0

aπ + b = 3

Resolviendo el sistema llegamos a que a = 3

π−1

b = − 3π−1

15. Sea f la función denida en R por f(x) =

1−sin2( x2 )

(π−x)2 x 6= π

b x = π. Determine los valores de b para los cuales f es

contínua en x = π

Solución: Debe cumplirse que

f(π) = limx→π

f(x)

Donde f(π) = b , luego

limx→π

1− sin2(x2

)(π − x)2

= limx→π

cos2(x2

)(π − x)2

Sea u = π − x si x→ π ⇒ u→ 0 , entonces

limu→0

cos2(

12 (π − u)

)u2

= limu→0

(cos π2 cos u2 + sin π

2 sin u2

u

)2

= limu→0

( 12 sin u

2u2

)2

=1

4

Luego, por la condición de continuidad, tenemos que f(π) = limx→π

f(x) , por ende

b =1

4

16. Considere una función f : R→ R denida por: f(x) =

sin((1−a)x)

x x < 0

b(x− a)2 0 ≤ x ≤ 1sin(a(x−1))

ln x x > 1

. Determine los valores de a y b

para los cuales, f es contínua en x = 0, 1

Solución: Debemos analizar los dos puntos de discontinuidad, a saber

a) Análisis para x = 0 : Debemos vericar la existencia del limite, para ello los limites laterales son iguales, esdecir:

limx→0−

f(x) = limx→0+

f(x)

Page 45: Apuntes Resueltos Todo Calculo 1

1.4. LÍMITE Y CONTUINIDAD 45

Luego

limx→0−

f(x) = limx→0−

sin((1− a)x)

x

= limx→0−

(1− a) sin((1− a)x)

(1− a)x

= (1− a)

Por otro lado

limx→0+

f(x) = limx→0+

b(x− a)2

= a2b

Como los límites laterales deben ser iguales, se tiene que

1− a = a2b

b) Análisis para x = 1 : Análogamente al caso anterior debe darse que

limx→1−

f(x) = limx→1+

f(x)

Entonces,

limx→1−

f(x) = limx→1−

b(x− a)2

= b(1− a)2

Además,

limx→1+

f(x) = limx→1+

sin(a(x− 1))

lnx

Sea u = x− 1 si x→ 1+ ⇒ u→ 0+ , entonces

limu→0+

sin(au)

ln(1 + u)= lim

u→0+

a sin(au)au

1u ln(1 + u)

= limu→0+

a sin(au)au

ln(1 + u)1u

= a

Como los límites laterales deben ser iguales

b(1− a)2 = a

Luego, debemos resolver el sistema

b(1− a)2 = a

1− a = a2b⇒

b(1− a)2 = a1−aa2 = b

⇒ (1−a)3

a2 = a⇒ 1−3a+3a2−a3 =

a3 ⇒ 2a3 − 3a2 + 3a− 1 = 0. Notese que si consideramos el polinomio p(a) = 2a3 − 3a2 + 3a− 1 , p( 12 ) = 0 ,

entonces

(2a3 − 3a2 + 3a− 1) ÷(a− 1

2

)= 2a2 − 2a+ 2

−(2a3 − a2)

(−2a2 + 3a− 1)

−(−2a2 + a)

(2a− 1)

−(2a− 1)

0

Page 46: Apuntes Resueltos Todo Calculo 1

46 CAPÍTULO 1. PROBLEMAS RESUELTOS.

Entonces

p(a) =

(a− 1

2

)(2a2 − 2a+ 2)

Entonces, p(a) = 0 , si

(a− 1

2

) 6=0︷ ︸︸ ︷(2a2 − 2a+ 2) = 0

Entonces a = 12 . Reemplazando

b =1− 1

214

=1214

= 2

1.5. Derivadas

1. Usando la denición, demuestre que si f(x) =√x+ 1⇒ f ′(x) = 1

2√x+1

Solución: Recordemos que la denición es f ′(x) = limh→0

f(x+ h)− f(x)

h, entonces

f ′(x) = limh→0

√(x+ h) + 1−

√x+ 1

h·√

(x+ h) + 1 +√x+ 1√

(x+ h) + 1 +√x+ 1

= limh→0

x+ h+ 1− x− 1

h(√

(x+ h) + 1 +√x+ 1

)= lim

h→0

1√x+ h+ 1 +

√x+ 1

=1

2√x+ 1

2. Demuestre que si g(x) = f(x+ c)⇒ g′(x) = f ′(x+ c)

Solución: En efecto

g′(x) = limh→0

g(x+ h)− g(x)

h

= limh→0

f((x+ h) + c)− f(x+ c)

h

= f ′(x+ c)

3. Si f es derivable en x = a, encuentre una expresión para limh→0

f(a+ αh)− f(a+ βh)

h

Solución: Sumaremos convenientemente un cero, de tal manera que se obtenga lo siguiente

limh→0

f(a+ αh)− f(a+ βh)

h= lim

h→0

f(a+ αh)− f(a+ βh) + f(a)− f(a)

h

= limh→0

f(a+ αh)− f(a)− (f(a+ βh)− f(a))

h

= limh→0

f(a+ αh)− f(a)

h− limh→0

f(a+ βh)− f(a)

h

= α

[limh→0

f(a+ αh)− f(a)

αh

]− β

[limh→0

f(a+ βh)− f(a)

βh

]= αf ′(a)− βf ′(a) = (α− β)f ′(a)

Page 47: Apuntes Resueltos Todo Calculo 1

1.5. DERIVADAS 47

4. Muestre que la función f(x) =

x sin

(1x

), x 6= 0

0, x = 0es contínua en x = 0 pero no es derivable en x = 0.

Solución: Para analizar la continuidad estudiaremos el limite en cero, es decir

limx→0

x sin

(1

x

)Haciendo el cambio u = 1

x si x→ 0 entonces u→∞, luego:

limu→∞

sinu

u= 0

Como f(0) = 0, entonces f es continua en x = 0.

Para el caso de la derivada en cero se sabe de la denición que:

f ′(0) = limh→0

f(h)− f(0)

h= limh→0

h

hsin

(1

h

)= limh→0

sin

(1

h

)= @

Como el límite no existe eso implica que no tiene derivada en cero por ende no es derivable allí.

5. Sea f una función denida mediante f(x) =

sinx x < π

mx+ b x ≥ π. Determine los valores de m y b tales que f sea

derivable en x = π

Solución: Una de las condiciones necesarias para que f sea derivable en x = π es que f sea contínua en esemis mo punto, por lo tanto, se procederá a realizar un análisis de la continuidad de la función. Analizando loslímites laterales se tiene

limx→π−

f(x) = limx→π−

sinx

= 0

Por otra parte:

limx→π+

f(x) = limx→π+

(mx+ b)

= mπ + b

Como los límites deben ser iguales

mπ + b = 0

Para que la función sea derivable en x = π , debe de existir limh→0

f(π + h)− f(π)

h= f ′(π), previamente, f(π) = mπ+b

, entonces, las derivadas laterales son

f ′−(π) = limh→0−

sin(π + h)− sinπ

h

= limh→0−

sinπ cosh+ cosπ sinh

h

= cosπ limh→0−

sinh

h= −1

Page 48: Apuntes Resueltos Todo Calculo 1

48 CAPÍTULO 1. PROBLEMAS RESUELTOS.

Por otro lado

f ′+(π) = limh→0+

m(π + h) + b−mπ − bh

= limh→0+

mπ +mh+ b−mπ − bh

= limh→0+

mh

h= m

Como las derivadas laterales deben ser iguales

m = −1

Reemplazando este valor en la ecuación obtenida de la continuidad

−π + b = 0

b = −π

6. Sea f la funcion denida mediante f(x) =

x5 + x2 + 6 x < 0

a(cos(3x) + 3 sinx) + b(x+ 1) x ≥ 0. Determine los valores de a y

b tales que f sea derivable en x = 0

Solución: Realizando primero un análisis de la continuidad, se tiene que

limx→0−

f(x) = limx→0−

(x5 + x2 + 6)

= 6

Por otro lado

limx→0+

f(x) = limx→0+

[a(cos(3x) + 3 sinx) + b(x+ 1)]

= a+ b

Como los laterales deben ser iguales

a+ b = 6

Ahora, se analizará la exixtencia de f ′(0) = limh→0

f(0 + h)− f(0)

h= limh→0

f(h)− f(0)

h. Por derivadas laterales

f ′−(0) = limh→0−

h5 + h2 + 6− 6

h

= limh→0−

h(h4 + h)

h

= limh→0−

(h4 + h)

= 0

Por otro lado

f ′+(0) = limh→0+

a(cos(3h) + 3 sinh) + b(h+ 1)− a− bh

= limh→0+

−a(1− cos(3h)) + 3a · sinh+ bh+ b− bh

= limh→0+

(−3a(1− cos(3h))

3h+

3a · sinhh

+ b

)= 3a+ b

Page 49: Apuntes Resueltos Todo Calculo 1

1.5. DERIVADAS 49

Como las derivadas laterales deben ser iguales

3a+ b = 0

De las ecuaciones obtenidas de continuidad y derivadas laterales tenemos el sistema

a+ b = 6

3a+ b = 0⇔

b = 6− a3a+ 6− a = 0

a = −3

b = 9

7. Considere la función f denida como f(x) =

2x2

x+a x < a

2x− ex−a x ≥ a

a) Calcule los límites laterales de f cuando x→ a± (en términos de a ) y muestre que f resulta contínua si y solosi a = 1

Solución: Determinando los límites laterales, se tiene que

limx→a−

f(x) = limx→a−

2x2

x+ a

=2a2

2a= a

Por otra parte

limx→a+

f(x) = limx→a+

(2x− ex−a

)= 2a− 1

Dado que los límites laterales deben ser iguales

a = 2a− 1

a = 1

Por ende, si a = 1 la función es contínua.

b) Considere que a = 1, determine f ′(1) y establezca la ecuación de la tangente a la curva en el punto (1, f(1))

Solución: Para determinar f ′(1) debemos usar la denición. Recordemos que la función a analizar es

f(x) =

2x2

x+1 x < 1

2x− ex−1 x ≥ 1

Page 50: Apuntes Resueltos Todo Calculo 1

50 CAPÍTULO 1. PROBLEMAS RESUELTOS.

Luego:

f ′−(1) = limh→0−

f(1 + h)− f(1)

h

= limh→0−

2(1+h)2

1+h+1 − 1

h

= limh→0−

2(h2 + 2h+ 1)− h− 2

h(h+ 2)

= limh→0−

2h2 + 4h+ 2− h− 2

h(h+ 2)

= limh→0−

2h2 + 3h

h(h+ 2)

= limh→0−

2h+ 3

h+ 2

=3

2

Por otro lado,

f ′+(1) = limh→0+

f(1 + h)− f(1)

h

= limh→0+

2(1 + h)− eh − 1

h

= limh→0+

2 + 2h− eh − 1

h

= limh→0+

(2− eh − 1

h

)= 1

Conclusión: como los límites laterales son distintos, podemos concluir que en x = 1 la función no es derivable,por lo tanto, no es posible calcular la tangente en ese punto.

8. Sea f la función denida mediante f(x) =

1−cos x

x x < 0

ax+ b 0 ≤ x < 1

x2 + 2 1 ≤ x ≤ 3x3−3x2

2x2−18 x > 3

a) Determine los valores de a y b de modo que la función f sea contínua en x = 0 y x = 1.

Solución: Primero realizaremos el análisis para x = 0 ,

limx→0−

f(x) = limx→0−

1− cosx

x= 0

Análogamente,

limx→0+

f(x) = limx→0+

(ax+ b)

= b

Entonces

b = 0

Page 51: Apuntes Resueltos Todo Calculo 1

1.5. DERIVADAS 51

Para x = 1 ,

limx→1−

f(x) = limx→1−

(ax+ b)

= 2a+ b

Por otro lado,

limx→1+

f(x) = limx→1+

(x2 + 2)

= 3

Entonces

2a+ b = 3

a =3

2

Redeniendo f

f(x) =

1−cos x

x x < 032x 0 ≤ x < 1

x2 + 2 1 ≤ x ≤ 3x3−3x2

2x2−18 x > 3

b) Determine la derivada de f en los puntos x = 3 x = 0 y x = 4.

Solución: Para x = 0, usando la denición, se tiene que

f ′−(0) = lımh→0−

f(h)− f(0)

h

= lımh→0−

1−cos(h)h − 0

h

= lımh→0−

1− cos(h)

h2· 1 + cos(h)

1 + cos(h)

= lımh→0−

sin2(h)

h2· 1

1 + cos(h)

=1

2

Por otro lado

f ′+(0) = lımh→0+

f(h)− f(0)

h

= lımh→0+

32h− 0

h

=3

2

Page 52: Apuntes Resueltos Todo Calculo 1

52 CAPÍTULO 1. PROBLEMAS RESUELTOS.

Por lo quef ′(0) no existe. Para x = 3, hacemos un análisis similar, usando la denición

f ′−(3) = lımh→0−

f(3 + h)− f(3)

h

= lımh→0−

(3 + h)2 + 2− 11

h

= lımh→0−

9 + 6h+ h2 − 11

h

= lımh→0−

h(6 + h)

h= 6

Por otro lado

f ′+(3) = lımh→0+

(3+h)3−3(3+h)2

2(3+h)2−18 − 32

h

= lımh→0+

(3+h)2((3+h)−3)2((3+h)−3)((3+h)+3) −

32

h

= lımh→0+

(3 + h)2 − 3((3 + h) + 3)

2h((3 + h) + 3))

= lımh→0+

9 + 6h+ h2 − 9− 3h− 9

2h(6 + h)

= lımh→0+

h2 + 3h− 9

2h(6 + h)= −∞

Como el límite no existe, no existe f ′(3) , para el caso en que x = 4 , la función está bien denida en x ∈]3,+∞[,entonces, basta con derivarla con las reglas convencionales , luego

f ′(x) =(3x2 − 6x)(2x2 − 18)− (x3 − 3x2)4x

(2x2 − 18)2

f ′(4) =(64− 24)(32− 18)− (64− 48)16

(32− 18)2

=40 · 14− 16 · 16

142

=76

49

c) Determine la asíntota oblícua de f hacia +∞ .

Solución:

m = lımx→∞

f(x)

x

= lımx→∞

x3−3x2

2x2−18

x

= lımx→∞

x3 − 3x2

2x3 − 18x

=1

2

Page 53: Apuntes Resueltos Todo Calculo 1

1.5. DERIVADAS 53

Luego

n = lımn→∞

(f(x)−mx)

= lımx→∞

(x3 − 3x2

2x2 − 18− 1

2x

)=

1

2lımx→∞

(x3 − 3x2 − x3 + 9x

x2 − 9

)= −3

2

Por lo tanto, la asíntota oblicua buscada es

y =x

2− 3

2

9. Calcule la derivada de:

a) f(x) = 2√x+ x2

Solución: Aplicando las técnicas de derivación:

f ′(x) = 2 · 1

2√x

+ 2x

=1√x

+ 2x

b) f(x) = lnx · sinx+ tan xx2

Solución:

f ′(x) =sinx

x+ lnx · cosx+

x2 sec2 x− 2x tanx

x4

=sinx

x+ lnx · cosx+

x sec2 x− 2 tanx

x3

c) f(x) =(√x+ 3√x) cos x

ex tan x

Solución:

f ′(x) =ddx ((√x+ 3√x) cosx) · ex tanx− (

√x+ 3√x) cosx · ddx (ex tanx)

e2x tan2 x

=

((1

2√x

+ 2

33√x2

)cosx− (

√x+ 3√x) sinx

)ex tanx− (

√x+ 3√x) cosx ·

(ex tanx+ ex sec2 x

)e2x tan2 x

=

((1

2√x

+ 2

33√x2

)cosx− (

√x+ 3√x) sinx

)ex tanx− cosx · ex (

√x+ 3√x)(tanx+ sec2 x

)e2x tan2 x

d) f(x) =√

2xx2+1

Page 54: Apuntes Resueltos Todo Calculo 1

54 CAPÍTULO 1. PROBLEMAS RESUELTOS.

Solución: Aplicando la regla de la cadena

f ′(x) =1

2√

2xx2+1

· ddx

(2x

x2 + 1

)

=1

2√

2xx2+1

· 2(x2 + 1)− 2x · 2x(x2 + 1)2

=1√2xx2+1

· 1− x2

(x2 + 1)2

=1− x2

√2x · (x2 + 1)

32

e) f(x) = ln(

etan x√2x3+5

)Solución:

f(x) = ln etan x − ln√

2x3 + 5

= tanx− 1

2ln(2x3 + 5)

Derivando

f ′(x) = sec2 x− 6x2

2(2x3 + 5)

= sec2 x− 3x2

(2x3 + 5)

f ) f(x) = 3

√3√x+ 1−sin x

cos(ln x)

Solución:

f ′(x) =2

33

√(3√x+ 1−sin x

cos(ln x)

)2· ddx

(3√x+

1− sinx

cos(lnx)

)

=2

33

√(3√x+ 1−sin x

cos(ln x)

)2·

3

2√x

+− cosx · cos(lnx) + (1− sinx) ·

(sin(ln x)

x

)cos2(lnx)

=

2

33

√(3√x+ 1−sin x

cos(ln x)

)2·(

2

3√x

+(1− sinx) sin(lnx)− x cosx · cos(lnx)

x · cos2(lnx)

)

g) f(x) = arctan(√

ln(1 + tan(sin(3x2))))

Solución:

f ′(x) =1

1 + ln(1 + tan(sin(3x2)))· 1

2√

ln(1 + tan(sin(3x2)))· 1

1 + tan(sin(3x2))· sec2(sin(3x2)) · cos(3x2) · 6x

=3x · sec2(sin(3x2)) · cos(3x2)

(1 + ln(1 + tan(sin(3x2))))√

ln(1 + tan(sin(3x2))) · (1 + tan(sin(3x2)))

Page 55: Apuntes Resueltos Todo Calculo 1

1.5. DERIVADAS 55

h) Sea w(φ) = cos(

1√φ

exp(θφ2))

+ φθ ln (θ − φ) . Calcule dw

Solución:

dw

dφ=

d

(cos

(1√φ

exp(θφ2)))

+d

θln (θ − φ)

)= − sin

(1√φ

exp(θφ2))· ddφ

(1√φ

exp(θφ2))

+1

θln(θ − φ)− φ

θ(θ − φ)

= − sin

(1√φ

exp(θφ2))·

(− 1

2√φ3

exp(θφ2)

+2θφ√φ

exp(θφ2))

+1

θln(θ − φ)− φ

θ(θ − φ)

= sin

(1√φ

exp(θφ2))· exp

(θφ2)·

(1

2√φ3− 2θφ√

φ

)+

(θ − φ) ln(θ − φ)− φθ(θ − φ)

i) Se tiene la función w = w(t) donde w = ln√√

u− eu y u(t) = sin(5t) . Determined2w

dt2en términos de t y

usando explícitamente la regla de la cadena

Solución: Tenemos la relación

w = ln

√√u− eu

w =1

2ln(√u− eu)

Derivando

dw

dt=

1

2

[1√

u− eu

(1

2√u

du

dt− eu du

dt

)]dw

dt=

1

2· dudt· 1√

u− eu

(1

2√u− eu

)Derivando nuevamente

d2w

dt2=

1

2

[d2u

dt21√

u− eu

(1

2√u− eu

)−(du

dt

)21

(√u− eu)2

(1

2√u− eu

)+du

dt· 1√

u− eu

(−1

4√u3

du

dt− eu du

dt

)]

=1

2

[d2u

dt21√

u− eu

(1

2√u− eu

)−(du

dt

)21

(√u− eu)2

(1

2√u− eu

)−(du

dt

)2

· 1√u− eu

(1

4√u3

+ eu)]

Donde u(t) = sin(5t) ,du

dt= 5 cos(5t) y

d2u

dt2= −25 sin(5t) , reemplazando se obtiene lo pedido. (e deja a cargo

del lector el reemplazo nal)

10. Determine las siguientes derivadas implícitas

a) y + xy3 = 2 +√xy

Solución: Derivando, teniendo en cuenta que y = y(x)

y′ + y3 + 3xy2y′ =1

2√xy· (1 + xy′)

y′ + 3xy2y′ − xy′

2√xy

=1

2√xy− y3

y′ =

12√xy − y

3

1 + 3xy2 +√x

2√y

Page 56: Apuntes Resueltos Todo Calculo 1

56 CAPÍTULO 1. PROBLEMAS RESUELTOS.

b) sin(x2 + y)− 2xy3 = arctan(y − x) + 2

Solución: Derivando la igualdad

cos(x2 + y) · (2x+ y′)− 2(y3 + 3xy2y′) =1

1 + (y − x)2· (y′ − 1)

2x cos(x2 + y) + y′ cos(x2 + y)− 2y3 − 6xy2y′ =y′

1 + (y − x)2− 1

1 + (y − x)2

y′[cos(x2 + y)− 6xy2 − 1

1 + (y − x)2

]= 2y3 − 1

1 + (y − x)2− 2x cos(x2 + y)

y′ =2y3 − 1

1+(y−x)2 − 2x cos(x2 + y)

cos(x2 + y)− 6xy2 − 11+(y−x)2

11. Considere las funciones del tipo w(x) = uv , donde u = u(x) y v = v(x)

a) Encuentre una expresión para calculardw

dx

Solución: Aplicando ln(·) a la igualdad

w = uvln(·)︷︸︸︷⇒ lnw = v · lnuddx (·)︷︸︸︷⇒ 1

w

dw

dx=dv

dx· lnu+ v · 1

u· dudx

⇒ dw

dx= w

(dv

dx· lnu+

v

u· dudx

)⇒ dw

dx= uv

(dv

dx· lnu+

v

u· dudx

)b) Utilice la expresión obtenida en (a) para determinar la derivada de f(x) =

(3x2 +

√x+ ex cosx

)arctan x

Solución: Se tienen dos opciones, las cuales son, ubicar u y v o bien, aplicar un procedimiento similar alanterior, utilizando lo anterior se tiene

u = 3x2 +√x+ ex cosx ⇒ du

dx= 6x+

1

2√x

+ ex(cosx− sinx)

v = arctanx ⇒ dv

dx=

1

1 + x2

Reemplazando en la expresión obtenida anteriormente

f ′(x) =(3x2 +

√x+ ex cosx

)arctan x·(

11+x2 · ln

(3x2 +

√x+ ex cosx

)+ arctan x

3x2+√x+ex cos

(6x+ 1

2√x

+ ex(cosx− sinx)))

12. Use derivación logarítmica para calculardy

dxen los siguientes casos:

a) y = xx2

Solución: Aplicando ln() :

ln y = x2 lnx

1

yy′ = 2x lnx+ x

y′ = xx2

(2x lnx+ x)

Page 57: Apuntes Resueltos Todo Calculo 1

1.5. DERIVADAS 57

b) y = ln(1−sinh x)(1 + tanx)

Solución: Procediendo de manera análoga:

ln y = (1− sinhx) ln(ln(1 + tanx))

1

yy′ = − coshx · ln(ln(1 + tanx)) +

(1− sinhx) sec2 x

(1 + tanx) ln(1 + tanx)

y′ = ln(1−sinh x)(1 + tanx) ·[− coshx · ln(ln(1 + tanx)) +

(1− sinhx) sec2 x

(1 + tanx) ln(1 + tanx)

]

c) y =

√1 + coshx

1− exp(x2)

Solución: De manera similar:

ln y = ln

(√1 + coshx

1− exp(x2)

)

ln y =1

2ln

(1 + coshx

1− exp(x2)

)ln y =

1

2

[ln (1 + coshx)− ln

(1− exp(x2)

)]1

yy′ =

1

2

[sinhx

1 + coshx+

2x exp(x2)

1− exp(x2)

]y′ =

1

2

√1 + coshx

1− exp(x2)·[

sinhx

1 + coshx+

2x exp(x2)

1− exp(x2)

]13. Para oscilaciones de amplitud pequeña, podemos modelar sin problemas la relación entre el periodo T y la longitud

L de un péndulo simple mediante la ecuación:

T = 2π

√L

g

Donde g es la gravedad. Si el péndulo es de metal, su longitud variará con la temperatura u en una razón que escasi proporcional a L, es decir:

dL

du= kL

Suponiendo este caso, demuestre que

T ′(u) =kT

2.

Solución: Se tiene que

T = 2π

√L

g⇒ T ′ =

π√Lg

· 1

g· dLdu

Pero sabemos quedL

du= kL, reemplazando:

T ′ =π√Lg

· 1

g· kL⇒ T ′ = kπ

√L

g

PeroT

2= π

√L

g, entonces:

T ′ =kT

2

Page 58: Apuntes Resueltos Todo Calculo 1

58 CAPÍTULO 1. PROBLEMAS RESUELTOS.

14. Encuentre los valores de M , tales que la función y = Mx2e2x sea solución ded2y

dx2− 4

dy

dx+ 4y = 6e2x

Solución: Si y es solución de la ecuación diferencial, entonces es posible reemplazar y en ella , entonces

y = Mx2e2x

dy

dx= M(2xe2x + 2x2e2x)

d2y

dx2= M

(2(e2x + 2xe2x) + 2(2xe2x + 2x2e2x)

)= 2Me2x

(1 + 4x+ 2x2

)Reemplazando

d2y

dx2− 4

dy

dx+ 4y = 2Me2x(1 + 4x+ 2x2)− 4Me2x(2x+ 2x2) + 4Mx2e2x

= Me2x(2 + 8x+ 4x2 − 8x− 8x2 + 4x2)

= 2Me2x

Comod2y

dx2− 4

dy

dx+ 4y = 6e2x

2Me2x = 6e2x

M = 3

15. Considere la ecuaciónd2u

dx2+ x2u = 0 (∗)

a) Pruebe que con la sustitución u = y√x la ecuación (∗) se transforma en x2 d

2y

dx2+ x

dy

dx+ y

(x4 − 1

4

)= 0 (∗∗)

Solución: Si u = y√x , entonces

du

dx= x

12dy

dx+

1

2yx−

12

d2u

dx2=

1

2x−

12dy

dx+ x

12d2y

dx2+

1

2

(dy

dx· x− 1

2 − 1

2yx−

32

)Reemplazando en (∗)

d2u

dx2+ x2u =

1

2x−

12dy

dx+ x

12d2y

dx2+

1

2

(dy

dx· x− 1

2 − 1

2yx−

32

)+ x2 · (yx 1

2 ) = 0

=1

2x−

12dy

dx+ x

12d2y

dx2+

1

2

dy

dx· x− 1

2 − 1

4yx−

32 + yx

52 = 0

= x12d2y

dx2+ x−

12dy

dx+ y

(x

52 − 1

4x−

32

)= 0

Multiplicando por x32

x2 d2y

dx2+ x

dy

dx+ y

(x4 − 1

4

)= 0

b) Pruebe además que esta última sustitución con la sustitución w =x2

2transforman a la ecuación (∗∗) a

w2 d2y

dw2+ w

dy

dw+ y

(w2 − 1

16

)= 0

Page 59: Apuntes Resueltos Todo Calculo 1

1.5. DERIVADAS 59

Solución: Se debe entender que y = f(w(x)) , luego

dy

dx=

dy

dw· dwdx

Dondedw

dx= x , entonces

dy

dx= x

dy

dwd2y

dx2=

dy

dw+ x

d2y

dw2· dwdx

=dy

dw+ x2 d

2y

dw2

Reemplazando en (∗∗)

x2 d2y

dx2+ x

dy

dx+ y

(x4 − 1

4

)= x2

(dy

dw+ x2 d

2y

dw2

)+ x2 dy

dw+ y

(x4 − 1

4

)= 0

= x4 d2y

dw2+ 2x2 dy

dw+ y

(x4 − 1

4

)= 0

Luego, como x2 = 2w , se tiene que

4w2 d2y

dw2+ 4w

dy

dw+ y

(4w2 − 1

4

)= 0

w2 d2y

dw2+ w

dy

dw+ y

(w2 − 1

16

)= 0

16. Para sistemas mecánicos vibratorios simples, existe una expresión que dene la dependencia de la posición en términosdel tiempo para un sistema físico en que una partícula se encuentra unida a un resorte, esta expresión es

x(t) = x0 cos

(√k

mt

)

Donde k y m son la constante elástica del resorte, y la masa de la partícula respectivamente. Demuestre que

d2x

dt2+k

mx = 0

Solución: Derivando

dx

dt= −x0

√k

msin

(√k

mt

)d2x

dt2= −x0 ·

k

mcos

(√k

mt

)

Reemplazando en la ecuación:

d2x

dt2+k

mx = −x0 ·

k

mcos

(√k

mt

)+ x0

k

mcos

(√k

mt

)d2x

dt2+k

mx = 0

Page 60: Apuntes Resueltos Todo Calculo 1

60 CAPÍTULO 1. PROBLEMAS RESUELTOS.

17. Sean f, g : R→ Rderivables tales que f(0) = 0 , g(0) = 1 y además, f ′(x) = g(x) ; g′(x) = f(x). Demuestre queh(x) = (f(x))2 − (g(x))2 es constante y determine su valor.

Solución: Si derivamos h(x) se tiene

h′(x) = 2f(x)f ′(x)− 2g(x)g′(x)

= 2 (f(x)f ′(x)− g(x)g′(x))

De acuerdo a los datos den enunciado,

h′(x) = 2 (f(x)g(x)− g(x)f(x))

= 0

Entonces, como h′(x) = 0, la función h(x) es constante. En particular si x = 0

h(0) = f(0)− g(0)

= −1

Luego , por lo demostrado anteriormente, se tiene que

h(x) = −1

18. Considere la parábola f(x) = x2 + x

a) Determine la ecuación de la recta tangente y normal a la curva en un punto x0 ∈ Dom(f)

Solución: Sabemos que la ecuación de la recta tangente en cualquier punto se escribe como

y − y0 = f ′(x0) · (x− x0)

Donde y0 = f(x0). Para el caso de la recta normal, se tiene

y − y0 = − 1

f ′(x0)· (x− x0)

Para el caso de la recta tangente, tenemos que

y0 = f(x0)

= x20 + x0

Además

f ′(x) = 2x+ 1

f ′(x0) = 2x0 + 1

Reemplazando

LT : y − (x20 + x0) = (2x0 + 1)(x− x0)

y = (2x0 + 1)(x− x0) + x20 + x0

= 2x0x− 2x20 + x− x0 + x2

0 + x0

y = x(2x0 + 1)− x20

Page 61: Apuntes Resueltos Todo Calculo 1

1.5. DERIVADAS 61

Por otro lado

LN : y − (x20 + x0) = − 1

(2x0 + 1)· (x− x0)

y = − x

2x0 + 1+

x0

2x0 + 1+ x2

0 + x0

= − x

2x0 + 1+x0 + (2x0 + 1)(x2

0 + x0)

2x0 + 1

= − x

2x0 + 1+x0 + 2x3

0 + 2x20 + x2

0 + x0

2x0 + 1

y = − x

2x0 + 1+

2x30 + 3x2

0 + 2x0

2x0 + 1

b) Determine el valor de x0 de la tangente determinada en (a) tal que la recta pase por el punto A(2,−3)

Solución: Evaluando este punto en la ecuación de la tangente, se tiene

−3 = 2(2x0 + 1)− x20

−3 = 2x0 + 2− x20

x20 − 2x0 − 5 = 0

Esta ecuación tiene como soluciones

x0 =2±√

4 + 20

2

x0 =2± 2

√6

2

x0 = 1±√

6

19. Determine los valores de la constante b , tal que las tangentes a la curva de ecuación f(x) = b2x3 + bx2 + 3x+ 9 enlos puntos de abscisas x = 1 y x = 2 sean paralelas.

Solución: para que las tangentes en esos puntos sean paraleleas, deben tener la misma pendiente, derivando:

f ′(x) = 3b2x2 + 2bx+ 3

f ′(1) = 3b2 + 2b+ 3

f ′(2) = 12b2 + 4b+ 3

Igualando

3b2 + 2b+ 3 = 12b2 + 4b+ 3

9b2 + 2b = 0

b(9b+ 2) = 0

Ecuación cuyas soluciones son b1 = 0

b2 = − 29

20. Se ha trazado una tangente a la curva f(x) = x3 , cuya pendiente es 3 y pasa por el punto (0,−2) . Hallar el puntode tangencia.

Page 62: Apuntes Resueltos Todo Calculo 1

62 CAPÍTULO 1. PROBLEMAS RESUELTOS.

Solución: Sabemos que la ecuación de recta tangente tiene la forma y = mx + n , donde m es la pendiente yn es su intercepto. Con la pendiente y el punto podemos obtener esta ecuación, es decir

y = mx+ n

y = 3x+ n

Como el punto (0,−2) pertenece a la recta, se tiene que

−2 = 3 · 0 + n

n = −2

Por lo tanto

yT = 3x− 2

Como en el punto de tangencia (x0, y0) , se tiene que dfdx (x0, y0) = 3 , se tiene que

f ′(x0) = 3x20

3 = 3x20

Entonces

x01 = 1

x02 = −1

Una condición necesaria para que la recta encontrada anteriormente sea tangente a la curva, es que en este puntox0 posea la misma imagen, luego

f(1) = 1

yT (1) = 1

Para el otro punto

f(−1) = −1

yT (−1) = 3 · (−1)− 1 = −4

En consecuencia, el punto de tangencia es (1, 1).

21. Sea h(x) = (f(x))3 , donde f es una función derivable tal que f(0) = − 1

2 y f ′(0) = 83 . Encuentre la exuación de la

recta tangente al gráco de h en x = 0.

Solución: Para derivar h , es necesario el uso de la regla de la cadena, por lo tanto

h′(x) = 3 (f(x))2f ′(x)

h′(0) = 3 (f(0))2f ′(0)

= 3 · 1

4· 8

3h′(0) = 2

Luego, h(0) =(− 1

2

)3= − 1

8 . Reemplazando en la ecuación de la tangente

y +1

8= 2 (x− 0)

y = 2x− 1

8

Page 63: Apuntes Resueltos Todo Calculo 1

1.5. DERIVADAS 63

22. Demuestre que la ecuación de la normal a la circunferencia x2 + y2 = a2 en cualquier punto P (x0, y0) , pasa por elorigen

Solución: Determinemos la pendiente de la normal a la circunferencia, derivando implícitamente:

2x+ 2yy′ = 0

y′ = −xy

Para cualquier punto P (x0, y0) , se tiene

mN = − 1(−x0

y0

)=

y0

x0

Luego, la ecuación de la normal es

y − y0 = mN (x− x0)

y − y0 =y0

x0(x− x0)

y =y0

x0· x− y0 + y0

y =y0

x0· x

Si el punto (0, 0) pertenece a la recta, entonces pasa por el origen, luego

0 =y0

x0· 0

0 = 0

Por lo tanto, pasa por el origen.

23. Establezca que la ecuación de la tangente a cualquier elipse del tipo x2

a2 + y2

b2 = 1 , en cualquier punto P (x0, y0) deella, esta dada por xx0

a2 + yy0b2 = 1

Solución: Derivando implícitamente

2x

a2+ 2

yy′

b2= 0

yy′

b2= − x

a2

y′ = − b2x

a2y

Para cualquiero punto (x0, y0) , se tiene

mT = − b2x0

a2y0

Page 64: Apuntes Resueltos Todo Calculo 1

64 CAPÍTULO 1. PROBLEMAS RESUELTOS.

Luego

y − y0 = − b2x0

a2y0(x− x0)

y − y0 = −b2xx0

a2y0+b2x2

0

a2y0

a2yy0 − a2y20 = −b2xx0 + b2x2

0

a2yy0 + b2xx0 = a2y20 + b2x2

0

xx0

a2+yy0

b2=

(x0

a

)2

+(y0

b2

)2

xx0

a2+yy0

b2= 1

24. Considere la función denida por f(x) = a2 ln

(a+√a2−x2

a−√a2−x2

)−√a2 − x2 , con a > 0 . La tangente trazada a la curva

denida por y = f(x) en el punto P (x0, y0) , corta al eje OY en un punto T . Demuestre que la longitud PT esconstante, es decir, independiente del punto P (x0, y0).

Solución: La función f puede escribirse como

f(x) =a

2

(ln(a+

√a2 − x2

)− ln

(a−

√a2 − x2

))−√a2 − x2

Derivando, utilizando la regla de la cadena

f ′(x) =a

2

(−x(

a+√a2 − x2

)√a2 − x2

− x(a−√a2 − x2

)√a2 − x2

)+

x√a2 − x2

=a

2

(−x(a−√a2 − x2

)− x

(a+√a2 − x2

)(a−√a2 − x2

) (a+√a2 − x2

)√a2 − x2

)+

x√a2 − x2

=a

2

(−

x[a−√a2 − x2 + a+

√a2 − x2

](a−√a2 − x2

) (a+√a2 − x2

)√a2 − x2

)+

x√a2 − x2

= −a2

(2ax(

a−√a2 − x2

) (a+√a2 − x2

)√a2 − x2

)+

x√a2 − x2

=−a2x(

a−√a2 − x2

) (a+√a2 − x2

)√a2 − x2

+x√

a2 − x2

=−a2x

(a2 − a2 + x2)√a2 − x2

+x√

a2 − x2

=x√

a2 − x2− a2

x√a2 − x2

=x2 − a2

x√a2 − x2

= −√a2 − x2

x

Luego, la pendiente en el punto P (x0, y0) es mT = −√a2−x2

0

x0y la ecuación de la tangente es

y − y0 = −√a2 − x2

0

x0(x− x0)

Page 65: Apuntes Resueltos Todo Calculo 1

1.5. DERIVADAS 65

En particular. el punto T se produce cuando x = 0 , dado que es una intersección en el eje Y , luego

T : y − y0 =√a2 − x2

0

y = y0 +√a2 − x2

0

Por lo tanto el punto T(

0, y0 +√a2 − x2

0

), para hallar la longitud PT , utilizamos la expresión que determina la

distancia entre dos puntos, luego

d(P, T ) =

√(y0 − y0 −

√a2 − x2

0

)2

+ (x0 − 0)2

=√a2 − x2

0 + x20

d(P, T ) = a

Luego, con este resultado, hemos demostrado que la longitud PT es constante.

25. Sea g derivable en R tal que g(0) = 2 y g′(x) = 1√1+x2

. Determine la ecuación de la recta tangente a f(x) = g(sinx)en x = 0

Solución: Debemos encontrar la pendiente de la recta y luego aplicar la ecuación de la recta punto pendiente,entonces

mT = f ′(x)|x=0 = cosx · g′(sinx)|x=0

= cos(0) · 1√1 + sin2 0

= 1

Ahora, si x = 0⇒ f(0) = g(sin(0)) = g(0) = 2 , por lo tanto el punto de tangencia es (0,2) . Aplicando la ecuacióndela tangente tenemos

y − y0 = mT (x− x0)

y − 2 = x

y = x+ 2

26. Sean f(x) y g(x) dos funciones derivables con f(x) 6= 0 , considere la función h(x) = f(x) · sin(g(x)). Pruebe que enlos puntos donde las grácas de h(x) y f(x) se tocan, ellas tienen la misma tangente.

Solución: las curvas f(x) y h(x) se tocarán si existe un punto x = x0 tal que

f(x0) = f(x0) sin(g(x0))

1 = sin(g(x0))

g(x0) =π

2+ 2kπ , k ∈ Z

Para efectos de cálculo, sólo consideraremos el caso en que k = 0 , es decir, cuando g(x0) =π

2.

En x = x0 punto debe ocurrir que h′(x0) = f ′(x0) luego

h′(x0) = f ′(x0) sin(g(x0)) + f(x0) cos(g(x0))g′(x0)

= f ′(x0) sin(π

2

)+ f(x0) cos

(π2

)g′(x0)

= f ′(x0)

Es decir, como f ′(x0) = h′(x0) en el punto donde se tocan, poseen la misma tangente.

Page 66: Apuntes Resueltos Todo Calculo 1

66 CAPÍTULO 1. PROBLEMAS RESUELTOS.

27. ¾Cuál es la condición que se debe cumplir para que la curva y = x3 + ax+ b sea tangente al eje X ?

Solución: Primero derivamos la función

y′ = 3x2 + ax

Para que la curva sea tangente al eje X debe cumplirse que y = 0 y que además y′ = 0 dado que ese punto detangencia es un cero de la función y además la tangente es paralela al eje X siendo esta colineal. Entonces se tiene

y′ = 0 ⇔ x(3x+ a) = 0

x1 = 0

x2 = −a3

Para la primera raíz, vemos que al evaluarla en la función original se tiene que y(0) = b , para que cumpla con lacondicion de tangencia al eje X en ese punto el valor de b debe ser cero.Para el segundo punto tenemos que y(−a3 ) = −a

3

27 −a2

9 + b = 0 , con lo cual se llega a que b = a2

9

(1 + a

3

)28. Analice completamente y graque f(x) =

(x+ 1)2

x2 + 1

Solución: Para esta función Dom(f) = R el único cero que posee esta función es x = −1, note que esta fun-ción es positiva para todo x en su dominio

Para las asíntotas:

Verticales: no tiene como consecuencia de su dominio

Horizontales: analizamos el

limx→±∞

(x+ 1)2

x2 + 1= 1

, por lo tanto,

y = 1

es asíntota horizontal. Dado que existe asíntota horizontal... no existe asíntota oblicua.

Análisis de la primera derivada:

f ′(x) =2(x+ 1)(x2 + 1)− (x+ 1)22x

(x2 + 1)2

f ′(x) =2(1− x2)

(x2 + 1)2

Puntos críticos:

x = 1x = −1

Análisis del crecimiento y decrecimiento de la función:

f(x) es creciente si: x ∈]− 1, 1[

f(x) es decreciente si x ∈]−∞,−1[∪]1,∞[

Page 67: Apuntes Resueltos Todo Calculo 1

1.5. DERIVADAS 67

Análisis de la segunda derivada:

f ′′(x) = 2

[−2x(x2 + 1)2 − (1− x2)(x2 + 1)4x

(x2 + 1)4

]f ′′(x) = 4x

[−x2 − 1− 2 + 2x2

(x2 + 1)3

]f ′′(x) =

4x(x2 − 3)

(x2 + 1)3

Puntos de Inexión: f ′′(x) = 0 si y solo si x(x −√

3)(x +√

3) = 0, lo cual nos conduce a tres puntos de

inexión:

x = 0

x =√

3

x = −√

3

Análisis de la curvatura: realizamos el análisis de la segunda derivada

f ′′(x) < 0 si

4x(x2 − 3)

(x2 + 1)3< 0

x ∈ ]−∞,−√

3[∪]0,√

3[

f ′′(x) > 0 si

4x(x2 − 3)

(x2 + 1)3> 0

x ∈ ]−√

3, 0[∪]√

3,∞[

Con toda esta información podemos gracar la función

Figure 1.5.1: Graco de f(x) = (x−1)2

x2+1

29. Estudie completamente la función f(x) = ln(1 + sinx)

Solución: Dominio: Sabemos que se debe cumplir

1 + sinx > 0

Sin embargo, sabemos que:1 + sinx ≥ 0

Page 68: Apuntes Resueltos Todo Calculo 1

68 CAPÍTULO 1. PROBLEMAS RESUELTOS.

Por lo tanto el único caso que no cumple con esta restricción es precisamente:

1 + sinx = 0⇒ sinx = −1⇒ x =3π

2+ 2kπ ; k ∈ Z

Por lo tanto su dominio es

Dom(f) = R−

2+ 2kπ

Analizando la paridad tenemos que la función sinx es de periodo 2π, por lo tanto la función f también es de periodo2π. Esto nos lleva a realizar todo el análisis de la función en el intervalo I = [0, 2π]. Para este caso, el unico puntoque no pertenece al dominio de la función en I es

x =3π

2

Ceros: para que f(x) = 0 se debe cumplir que:

ln(1 + sinx) = 0⇒ 1 + sinx = 1⇒ sinx = 0

x = 0, π, 2π

Signo: tenemos que analizar:

(a) Si f(x) < 0:ln(1 + sinx) < 0⇒ 0 < 1 + sinx < 1

−1 < sinx < 0

Para este caso veremos que el primer miembro de la desigualdad es válido casi siempre, a excepción de cuandosinx = −1, para el otro miembro de la desigualdad:

sinx < 0⇒ x ∈]π, 2π[

(b) Si f(x) > 0:ln(1 + sinx) > 0⇒ 1 + sinx > 1⇒ sinx > 0

x ∈]0, π[

Asintotas verticales: Se tiene como candidato a asíntota vertical el punto x = 3π2 para el intervalo de análisis.

Calculamos limx→ 3π

2

ln(1 + sinx) = −∞ , por lo tanto la recta x =3π

2es asíntota vertical.

Asíntotas horizontales: Calculamos limx→±∞

ln(1 + sinx) = @, por lo tanto, no tiene asíntotas horizontales.

1º Derivada: Tenemos:f ′(x) =

cosx

1 + sinx

Puntos críticos: en este caso cosx = 0 y esto se da si x =π

2.

Crecimiento y decrecimiento: Como el denominador es siempre positivo o nulo, el signo sólo depende de cosx,luego:

f(x) es creciente si:

cosx > 0⇒ x ∈]0,π

2[∪]

2, 2π[

f(x) es decreciente si:

cosx < 0⇒ x ∈]π

2,

2[

Page 69: Apuntes Resueltos Todo Calculo 1

1.5. DERIVADAS 69

Con ello podemos concluir que el punto x =π

2es un máximo.

Análisis de la 2º derivada: derivando la función por segunda vez queda:

f ′′(x) =− sinx(1 + sinx)− cosx cosx

(1 + sinx)2

f ′′(x) =− sinx− sin2 x− cos2 x

(1 + sinx)2

f ′′(x) =−(1 + sinx)

(1 + sinx)2

f ′′(x) = − 1

1 + sinx

Dado que f ′′(x) 6= 0 , ∀x ∈ R no posee puntos de inexión, y además, f ′′(x) < 0 ∀x ∈ R, luego, es siempre cóncava.Con esta información podemos esbozar el gráco de la función y es el siguiente:

Figure 1.5.2: Gráco de f(x) = ln(1 + sinx)

30. Analice completamente la función f(x) = exp

(1

lnx

)indicando dominio, paridad, asíntotas , crecimiento, concavi-

dad, gráco y recorrido.

Solución: Dominio: Dom(f) : x > 0 , además, la función no posee ni paridad ni imparidad. Es necesarioanalizar que para si x = 1, ya que ahí se produce un cambio en el signo de lnx, notemos que

limx→1

f(x) =∞

ylimx→∞

f(x) = 1

entonces, estamos en presencia de una asíntota vertical y horizontal.

Primera derivada:

f ′(x) = exp

(1

lnx

)·(− ln−2(x)

x

)= exp

(1

lnx

)·(−1

x ln2 x

)Para f ′(x) > 0 , tenemos que exp

(1

ln x

)> 0 para cualquier número real dentro del dominio, además, vemos que el

signo sólo depende de x en la expresión ya que ln2 x > 0 entonces:

f ′(x) > 0 si y solo si − x > 0⇒ x < 0

Sin embargo, el resultado obtenido no se encuentra dentro del dominio, por lo tanto en el complemento relativo a eseconjunto, la primera derivada es negativa lo cual nos dice que la función para cualquier punto considerado dentro

Page 70: Apuntes Resueltos Todo Calculo 1

70 CAPÍTULO 1. PROBLEMAS RESUELTOS.

de su dominio es decreciente.

Segunda derivada

f ′′(x) =exp

(1

ln x

)·[1 + 2 lnx+ ln2 x

]x2 ln4 x

=exp

(1

ln x

)(1 + lnx)2

x2 ln4 x

Todos los términos de esta derivada son positivos, entonces f ′′(x) > 0,∀x ∈ Dom(f) lo que nos lleva a concluir quees convexa.

Figure 1.5.3: Gráco de f(x) = exp(

1ln x

)31. Dada la función y = (lnx)

x

a) Determine su dominio y analice la posibilidad de incluir el valor 1 en el dominio de la función tal que ésta seacontínua en x = 1

Solución: Tenemos que lnx > 0 , luego x > 1 , para incluir el 1 debemos calcular limx→1

(lnx)x = 01 = 0

, luego es posible denir:

g(x) =

(lnx)

xx > 1

0 x = 1

b) Calcule dydx y pruebe que si x > e , entonces la función es estríctamente creciente.

Solución: Para la derivada:

ln y = x ln(lnx)

1

yy′ = ln(lnx) +

1

lnx

dy

dx= (lnx)

x

[ln(lnx) +

1

lnx

]Como nos dan el dato que x > e ⇒ lnx > 1 ⇒ ln(lnx) > 0 , entonces, como (lnx)x > 0 se tiene que dy

dx > 0 loque prueba que es creciente.

32. Determine el punto de mínimo dado ai ∈ R para i = 1, 2, ..., n de la función:

f(x) =

n∑i=1

(ai − x)2

Page 71: Apuntes Resueltos Todo Calculo 1

1.5. DERIVADAS 71

Solución: Podemos escribir f(x) como sigue

f(x) = (a1 − x)2 + (a2 − x)2 + ...+ (an − x)2

Derivando término a término

f ′(x) = −2(a1 − x)− 2(a2 − x)− ...− 2(an − x)

= −2

n∑i=1

(ai − x)

= 2nx− 2

n∑i=1

ai

Para hallar puntos críticos, se tiene que f ′(x) = 0 entonces

2

(nx−

n∑i=1

ai

)= 0

x =1

n

n∑i=1

ai

Para corroborar, tenemos que calcular f ′′(x) , luego

f ′′(x) = 2n

f ′′(x) > 0

Por lo tanto, ∀x ∈ R , la segunda derivada es positiva lo que asegura la existencia de un mínimo.

33. Sea f : R→ R una función derivable, positiva y creciente , además, sea g : R→ R una función derivable, negativa ydecreciente. Demuestre que la función h : R→ R denida por:

h(x) =f(g(x))

g(x)

Es creciente.

Solución: Con la información del enunciado es posible deducir que f(x) > 0 , f ′(x) > 0 , g(x) < 0 , g′(x) < 0 ,derivando h(x)

h′(x) =f ′(g) · g′(x) · g(x)− f(g) · g′(x)

g2(x)

=g′(x) (f ′(g)g(x)− f(g))

g2(x)

Observemos que g′(x) < 0 , además , que el producto f(g)g(x) < 0, esto implica que la diferencia f ′(g)g(x)−f(g) < 0,

por lo tanto g′(x) (f ′(g)g(x)− f(g)) > 0, como g2(x) > 0 para cualquier valor de x , se tiene que h′(x) > 0 con lo

cual se deduce que h es una función creciente.

34. Un avión vuela a una altitud H cuando empieza a descender hacia una pista de aterrizaje que está horizontal en el

suelo a una distancia L del avión, como se ilustra en la gura. Supnga que la trayectoria de aterrizaje del avión es

la gráca de la función polinomial cúbica f(x) = ax3 + bx2 + cx+ d , donde f(−L) = H y f(0) = 0.

Page 72: Apuntes Resueltos Todo Calculo 1

72 CAPÍTULO 1. PROBLEMAS RESUELTOS.

Figure 1.5.4: Ruta del avión

a) ¾Qué es dfdx en x = 0?

b) ¾Qué es dfdx en x = −L?

c) Use lo anterior para demostrar que

f(x) = H

[2( xL

)3

+ 3( xL

)2]

Solución: Se tiene quef(x) = ax3 + bx2 + cx+ d

; f(−L) = H y f(0) = 0. En primer lugardf

dx

∣∣∣∣x=0

es un punto crítico, luego

f ′(0) = 0

. Para el otro caso se tiene lo mismo, es decir:

f ′(−L) = 0

Para encontrar explícitamente f derivamos quedando lo siguiente:

f ′(x) = 3ax2 + 2bx+ c

Evaluando en la primera y segunda derivada de acuerdo a los datos obtenidos queda el sistema:H = −aL3 + bL2 − cL+ d

0 = d0 = 3aL2 − 2bL+ c

0 = c

Los resultados de este sistema son los siguientes:a = 2H

L3

b = 3HL2

c = 0d = 0

Reemplazando el la función tenemos:

f(x) =2H

L3x3 +

3H

L2x2 = H

(2( xL

)3

+ 3( xL

)2)

Page 73: Apuntes Resueltos Todo Calculo 1

1.5. DERIVADAS 73

35. Se desea poner en movimiento un cuerpo arrastrándolo sobre una supercie mediante la aplicación de una fuerza F .se tiene que al aplicar una fuerza en un ángulo θ con respecto a la horizontal, se tiene que la expresión para estafuerza es

F (θ) =µmg

cos θ + µ sin θ

Donde µ es el coeciente de roce, m es la masa del cuerpo, g es la aceleración de gravedad. Calcule el valor deθ ∈ [0, π2 ] para los cuales la fuerza sea mínima.

Solución: Derivando, tenemos que

dF

dθ= − µmg

(cos θ + µ sin θ)2· (− sin θ + µ cos θ)

LuegodF

dθ= 0 si

µ cos θ = sin θ

µ = tan θ

θ = arctanµ

Analizando el signo de la primera derivada, vemos que si θ ∈ [0, arctanµ[ , dFdθ < 0 , con lo cual se aprecia que lafuerza en ese intervalo decrece, si θ ∈] arctanµ, π2 [ , la derivada cambia de signo con lo que se concluye que alli lafunción crece , por ende la fuerza en el punto encontrado es mínima.

36. Se dispone de un alambre de largo 3a con el cual se desea formar un trapecio isósceles, con tres lados iguales a a y labase más grande de largo x de modo de maximizar el área. Determine el valor de x que cumple con esta condiciónextremal. Justique su respuesta.

Solución: Se tiene que el área del trapecio esta dada por

A =1

2(a+ x)h

Donde h es la altura del trapecio. Se puede apreciar (hacer la gura) que la base del triángulo rectángulo formadopor la altura h y el lado a vale x−a

2 , luego, se cumple la relación

a2 = h2 +

(x− a

2

)2

h =

√a2 −

(x− a

2

)2

=1

2

√2ax− x2 + 3a2

Luego

A(x) =1

4(a+ x)

√2ax− x2 + 3a2

Derivando

A′(x) =1

4

(√2ax− x2 + 3a2 + (a+ x) · 2a− 2x

2√

2ax− x2 + 3a2

)=

1

4√

2ax− x2 + 3a2

(2ax− x2 + 3a2 + a2 − x2

)=

ax− x2 + 2a2

4√

2ax− x2 + 3a2

Page 74: Apuntes Resueltos Todo Calculo 1

74 CAPÍTULO 1. PROBLEMAS RESUELTOS.

Luego A′(x) = 0 si

ax− x2 + 2a2 = 0

(a+ x)(2a− x) = 0

Como x > 0, la única solución factible de esta ecuación es x = 2a, la cual es nuestro candidato a máximo. Noteseque el signo de la primera derivada sólo depende del factor (2a− x), donde A′(x) < 0 para x > 2a y A′(x) > 0 parax < 2a lo que conrma el hecho de que x = 2a es un máximo.

37. Se debe construir una pirámide regular de base cuadrada de modo que la supercie total de sus 4 caras laterales seaST = 4S, donde S es la supercie de cada cara lateral. Se pide encontrar el máximo volumen de esta pairámide enfunción de S (Vmax = V (S)) para lo cual proceda como sigue:

Figure 1.5.5: Pirámide del problema

a) Demuestre que el valor de la altura V T de la cara BDV de la pirámide, que es un triángulo isósceles esV T = 1

2

√4h2 + x2 , donde h es la altura de la pirámide y x es la longitud del lado de la base (ver gura).

Solución: De la gura

V T 2 =(x

2

)2

+ h2

V T =

√x2 + 4h2

4

=1

2

√4h2 + x2

b) Deduzca una relación entre x , h y S y encuentre una expresión V (x) para el volumen de la pirámide(V = 1

3 (BASE) · (ALTURA))

Solución: Sabemos del enunciado que ST = 4S, donde S es la supercie de cada cara lateral , luego

S =1

2· x · (V T )

S =x

4

√4h2 + x2

Page 75: Apuntes Resueltos Todo Calculo 1

1.5. DERIVADAS 75

Además, V =1

3x2h , luego debemos encontrar V = V (x) , por ende

16S2 = x2(4h2 + x2)

16S2 = 4h2x2 + x4

16S2 − x4

4x2= h2

1

4

(16

(S

x

)2

− x2

)= h2

h =1

2

√16

(S

x

)2

− x2

Reemplazando en el volumen

V (x) =1

6x2

√16

(S

x

)2

− x2

V (x) =1

6x√

16S2 − x4

c) Determine las dimensiones de x y h que producen el máximo volumen. Justique que es máximo y calcule Vmax

Solución: Derivando el volumen

V ′(x) =1

6

(√16S2 − x4 − 4x4

2√

16S2 − x4

)=

1

6

(√16S2 − x4 − 2x4

√16S2 − x4

)=

1

6

(16S2 − x4 − 2x4

√16S2 − x4

)El candidato a punto crítico se dará cuando V ′(x) = 0 , es decir,

V ′(x) = 0 ⇔ 16S2 − 3x4 = 0

⇔ (4S −√

3x2)(4S +√

3x2) = 0

⇔ (2√S − 4√

3x)(2√S +

4√

3x)(4S2 +√

3x2) = 0

x1 = 2

√S

4√3

x2 = − 2√S

4√3

Analizando signos de la primera derivada, vemos que V ′(x) > 0⇔ x ∈]− 2√S

4√3, 2√S

4√3[ allí el volumen es creciente,

entonces, V ′(x) < 0⇔ x ∈]−∞,− 2√S

4√3[∪] 2

√S

4√3,+∞[ allí decrece. Por lo tanto, como x > 0 y los datos anteriores,

podemos asegurar que x1 = 2√S

4√3es el punto de máximo buscado. para calcular h basta reemplazar este valor ,

por lo que se tiene

h =1

2

√16

(S

x

)2

− x2

h =1

2

√√√√√16

S2√S

4√3

2

(2√S

4√

3

)2

Page 76: Apuntes Resueltos Todo Calculo 1

76 CAPÍTULO 1. PROBLEMAS RESUELTOS.

La simplicación queda a cargo del lector, nalmente

Vmax =1

6x√

16S2 − x4

=1

6

(2√S

4√

3

)√16S2 − 16S2

3

=1

6

(2√S

4√

3

)4S√

2√3

=1

6· 8√

2S

334

38. A partir de un circulo de papel de radio R se desea construir un cono, recortando del círculo el sector curcular OABde ángulo central θ y juntando los trazos OA y OB de modo que coincidan. Se formará de esta manera un conorecto circular cuya base es un círculo de perímetro igual a la longitud del arco que queda después del corte, y sualtura h puede deducirse del esquema que se muestra

Figure 1.5.6: Cono formado por semicírculo

Para calcular el valor del ángulo θ de modo que el cono formado como se indicó tenga volumen máximo se pide:

a) Denuestre que el radio basal del cono es r = R

(1− θ

)Solución: Para encontrar la magnitud del radio r en función del ángulo θ debemos considerar que:

PBA

= 2πR−Rθ

PBA

= 2πR

(1− θ

)

De ahí :

2πr = 2πR

(1− θ

)r = R

(1− θ

)

a) Denuestre que la altura del cono es h = R

√1−

(1− θ

)2

Page 77: Apuntes Resueltos Todo Calculo 1

1.5. DERIVADAS 77

Solución: Del Teorema de Pitágoras:

h2 + r2 = R2

h2 = R2 − r2

h =√R2 − r2

h =

√R2 −R2

(1− θ

)2

h = R

√1−

(1− θ

)2

b) Verique que con la sustitución x = 1− θ2π el volumen del cono queda V (x) = 1

3πR3x2√

1− x2

Solución: Si hacemos el cambio x = 1− θ

2π, se tiene:

V (x) =1

3πR3x2

√1− x2

c) Analice la función V (x) indicando: dominio, ceros, signos, paridad, crecimiento y decrecimiento y deduzca elvalor de x que hace máximo a V (x)

Solución: Analizando V (x) se tiene que su dominio es x ∈[−1, 1] (esto sin considerar la situación geométricadel problema) , para los ceros:

f(x) = 0 ⇒ x2√

1− x2 = 0

En este caso, los ceros de f son: x1 = −1x2 = 0x3 = 1

Para el signo; tenemos que V (x) ≥ 0 , ∀x ∈ R

Para la primera derivada:

V ′(x) =1

3πR2

(2x√

1− x2 − 2x3

2√

1− x2

)V ′(x) =

1

3πR3

(2x(1− x2)− x3

√1− x2

)V ′(x) =

1

3πR3

(2x− 2x3 − x3

√1− x2

)V ′(x) =

1

3πR3

(2x− 3x3

√1− x2

)Para los puntos críticos:

V ′(x) = 0⇒ x(2− 3x2) = 0x1 = 0

x2 =√

23

x3 =√

23

Page 78: Apuntes Resueltos Todo Calculo 1

78 CAPÍTULO 1. PROBLEMAS RESUELTOS.

Analizaremos los signos de la primera derivada , en donde se obtiene que V (x) es creciente si x ∈]∞,−√

23 [∪]0,

√23 [

, y será decreciente si x ∈]−√

23 , 0[∪]

√23 ,∞[

Con ello tenemos entonces dos puntos de máximo los cuales son x1 =√

23

x2 = −√

23

d) Calcule el volumen máximo del cono y el ángulo θ que lo genera.

Solución: Volviendo a las variables originales tenemos que:

x = 1− θ

2π⇒x1 ⇒ θ1

x2 ⇒ θ2

θ1 = 2π(1− x1)⇒ θ1 = 2π

(1−

√2

3

)< 2π

θ2 = 2π(1− x2)⇒ θ2 = 2π

(1 +

√2

3

)> 2π

Como el ángulo θ2 escapa a las limitaciones geométricas del problema, entonces el ángulo máximo que generael volumen máximo del cono es θ1 y el punto de máximo en la variable x es x1. Calculando el volumen máximotenemos:

Vmax =2

9πR3

√1

3[u3]

39. Considere una caja de base cuadrada de lado a y altura h. Un insecto está localizado en el vértice A y debe llegaral vértice B caminando en línea recta desde A hasta P y de igual forma desde P hasta B. Determinar la posicióndel punto P de manera de minimizar la distancia total de la gura. Justique su respuesta.

Figura 1.5.7: Ruta de la hormiga

Solución: Sea x la distancia de vertice superior a B hasta P entonces, el segmento restante mide a− x . Luego, enla tapa superior, se forma un triángulo rectángulo, por lo tanto es válida la relación

AP =√a2 + (a− x)2

Page 79: Apuntes Resueltos Todo Calculo 1

1.5. DERIVADAS 79

En la cara frontal, también se forma un triángulo rectángulo, entonces

PB =√h2 + x2

Luego la distancia L(x) es

L = AP + PB

L(x) =√a2 + (a− x)2 +

√h2 + x2

Derivando

L′(x) =−(a− x)√a2 + (a− x)2

+x√

h2 + x2

Luego, L′(x) = 0, entonces

x√h2 + x2

=(a− x)√

a2 + (a− x)2

x2

h2 + x2=

(a− x)2

a2 + (a− x)2

x2a2 + x2(a− x)2 = h2(a− x)2 + x2(a− x)2

a2x2 = h2(a− x)2

ax = ± h(a− x)

Como x < a , entonces

ax = h(a− x)

x =ah

a+ h

Este punto es el buscado, la comprobación del mínimo se obtiene derivando denuevo, luego

L′′(x) =

√(a− x)2 + a2 − (a− x) · a−x√

(a−x)2+a2

(a− x)2 + a2+

√x2 + h2 − x2

√x2+h2

x2 + h2

=(a− x)2 + a2 − (a− x)2

((a− x)2 + a2)32

+x2 + h2 − x2

(x2 + h2)32

=a2

((a− x)2 + a2)32

+h2

(x2 + h2)32

Como L′′(x) > 0 , se comprueba que el punto obtenido, es un mínimo.

40. Dos postes, uno de 3[m] y el otro de 9[m] están colocados verticalmente y separados 16[m] entre si. Se desea colo-car unos tirantes, sujetos a una sola estaca que se ubica a nivel del suelo en una línea que une los dos postes. ¾Enqué lugar de esa línea debe colocarse la estaca para que la suma de las longitudes de los tirantes AC y BC sea mínima?

Solución: Se tiene la siguiente gura

Page 80: Apuntes Resueltos Todo Calculo 1

80 CAPÍTULO 1. PROBLEMAS RESUELTOS.

Figura 1.5.8: Postes del problema

De acuerdo a esto, se tiene que

L(x) = y(x) + z(x)

En donde

y(x) =√

9 + x2

z(x) =√

81 + (16− x)2

Reemplazando

L(x) =√

9 + x2 +√

81 + (16− x)2

Derivando

dL(x)

dx=

x√9 + x2

− 2(16− x)√81 + (16− x)2

Como dLdx = 0 , se tiene que

x√9 + x2

− 2(16− x)

2√

81 + (16− x)2= 0

x√

81 + (16− x)2 − (16− x)√

9 + x2

√9 + x2 ·

√81 + (16− x)2

= 0

x√

81 + (16− x)2 − (16− x)√

9 + x2 = 0

x√

81 + (16− x)2 = (16− x)√

9 + x2

Elevando al cuadrado

x2(81 + (16− x)2) = (16− x)2(9 + x2)

81x2 + x2(16− x)2 = 9(16− x)2 + x2(16− x)2

81x2 = 9(256− 32x+ x2)

8x2 + 32x− 256 = 0

x2 + 4x− 32 = 0

Ecuación cuyas soluciones son x1 = −8

x2 = 4

Como x > 0 , la única solución admisible para el problema es x = 4 el cual es el punto buscado . La vericación deque este punto es de mínimo se deja a cargo del lector.

Page 81: Apuntes Resueltos Todo Calculo 1

1.5. DERIVADAS 81

41. Una barra de metal tiene forma de cilindro circular recto. Cuando se calienta, su longitud L y su radio R aumentana razón de 0, 005[cm/min] y 0, 002[cm/min] respectivamente. ¾A qué razón aumenta el volumen de la barra en elmomento que el largo mide 40[cm] y su radio mide 1, 5[cm] ?

Solución: Los datos del problema son los siguientes

dL

dt= 0, 005[cm/min]

dr

dt= 0, 002[cm/min]

Sabemos que el volumen del cilindro está dado por:

V = πr2h

Derivando:

dV

dt= π

(2rL

dr

dt+ r2 dL

dt

)Como L = 40[cm] y r = 1, 5[cm] reemplazando todos los datos:

dV

dt= π

(2 · 1, 5 · 40 · 0, 002 + 1, 52 · 0, 005

)42. Una cámara de TV sigue desde el suelo el despegue vertical de un cohete, que se produce de acuerdo con la ecuación

s = 50t2(s es la altura medida desde el suelo en metros y t es el tiempo en segundos). La cámara está a 2000m dellugar de despegue. Halle cómo varía el ángulo de elevación de la cámara después de 10s del despegue del cohete.

Solución: Figura del problema

Figure 1.5.9: Diagrama del problema

Solución: De la gura, es posible deducir que

tan θ =s

2000

sec2 θdθ

dt=

1

2000

ds

dtdθ

dt=

cos2 θ

2000

ds

dt

Además cos θ =2000√

s2 + 20002, s = 50t2 ⇒ ds

dt= 100t , reemplazando

dt=

2000

(50t2)2

+ 20002· 100t

Page 82: Apuntes Resueltos Todo Calculo 1

82 CAPÍTULO 1. PROBLEMAS RESUELTOS.

Reemplazando en t = 10[s]

dt

∣∣∣∣t=10[s]

=2000

(50(10)2)2

+ 20002· 100 · 10

[rad

s

]La reducción de esto se deja a cargo del lector.

43. Un canal vacio empieza a llenarse a razón de 14[m3/min] . El canal mide 50 metros de largo y su sección transversales un trapecio isósceles con altura 1 metro y las dimensiones dadas de la gura.

Figure 1.5.10: Sección transversal del canal

Determine la rapidez con que crece la altura del agua transcurridos 3 minutos.

Solución: Sean h y x la altura y el ancho del agua acumulada después de t minutos. En el instante t , elvolumen es

V =1

2· 50 · h · (4 + x)

= 25h(4 + x)

De semejanza de triángulos, se tiene que

h12 (x− 4)

=1

12 (6− 4)

2h

x− 4= 1

x = 4 + 2h

Reemplazando

V = 50h(4 + h)

= 50h2 + 200h

Derivando y usando quedV

dt= 14 , se tiene que

dV

dt= 50

(dh

dt(4 + h) + h

dh

dt

)14 = 50(2h+ 4)

dh

dtdh

dt=

7

25(2h+ 4)

Para calcular h , usamos los datos del problema, luego

V (3) = 42

V = 50h+ 200h2

Page 83: Apuntes Resueltos Todo Calculo 1

1.5. DERIVADAS 83

Igualando

42 = 50h+ 200h2

Ecuación cuya solución válida es

h = 0, 2

Reemplazando en la expresión para la variación de altura se obtiene lo pedido. (se deja al lector).

44. Sean dos resistencias R1 y R2 conectadas en paralelo. De acuerdo con esto, la resistencia equivalente corrresponde a

1

R=

1

R1+

1

R2

Si R1 y R2 aumentan a una razón de 0, 01 y 0, 02 (Ω/s) respectivamente. ¾En cuánto varía la resistencia equivalentecuando R1 = 30[Ω] y R2 = 90[Ω] ?

Solución: Tenemos que

R =R1R2

R1 +R2

Derivando

dR

dt=

(dR1

dt ·R2 +R1 · dR2

dt

)(R1 +R2)−R1R2

(dR1

dt + dR2

dt

)(R1 +R2)2

=dR1

dt R2R1 +R21dR2

dt + dR1

dt R22 +R1R2

dR2

dt −R1R2dR1

dt −R1R2dR2

dt

(R1 +R2)2

=R2

1dR2

dt + dR1

dt R22

(R1 +R2)2

De los datos del problema, se tiene que dR1

dt = 0, 01[Ω/s] y que dR2

dt = 0, 02[Ω/s] , además R1 = 30[Ω] y R2 = 90[Ω], reemplazando

dR

dt=

900 · 2 · 10−2 + 8100 · 10−2

(120)2

El cálculo nal se deja al lector.

45. Un peso está suspendido por una cuerda y se le causa una vibración mediante una fuerza sinusoidal, su desplazamientof(t) en el tiempo t viene dado por:

f(t) =A

c2 − k2(sin(kt)− sin(ct))

donde A , c y k son constantes positivas , c 6= k. Determinar el valor del límite cuando c tiende a k.

Solución: Aplicando L´Hopital

limc→k

A

c2 − k2(sin(kt)− sin(ct)) = A lim

c→k

−t cos(ct)

2c

= −At cos(kt)

2k

Page 84: Apuntes Resueltos Todo Calculo 1

84 CAPÍTULO 1. PROBLEMAS RESUELTOS.

46. Calcule limx→0

x1

ln(exp(x)−1)

Solución: Sea v = x1

ln(exp(x)−1) , luego

ln v =1

ln(exp(x)− 1)lnx

limx→0

ln v = limx→0

(1

ln(exp(x)− 1)lnx

)ln(

limx→0

v)

= limx→0

(lnx

ln(exp(x)− 1)

)Aplicando L´Hopital al lado derecho

ln(

limx→0

v)

= limx→0

1x

exp(x)exp(x)−1

= lim

x→0

(exp(x)− 1

x· 1

exp(x)

)= 1

Exponenciando

ln(

limx→0

v)

= 1

limx→0

v = e

47. Calcule los siguientes límites

a) limx→∞

x1x

Solución: Aplicando ln() a la expresión L = limx→∞

x1x se tiene

lnL = limx→∞

1

xlnx

Luego, aplicando L´H:

lnL = limx→∞

1

x= 0

Por lo tanto:

lnL = 0⇒ L = 1

b) limx→0

(ax + bx

2

) 1x

Solución: Análogamente al procedimiento anterior:

lnL = limx→0

1

xln

(ax + bx

2

)lnL = lim

x→0

(2

2 (ax + bx)(ax ln a+ bx ln b)

)lnL =

ln a+ ln b

2

L =√ab

Page 85: Apuntes Resueltos Todo Calculo 1

1.5. DERIVADAS 85

c) limx→0+

x1

ln(sinh x)

Solución: De manera similar:

lnL = limx→0+

1

ln(sinhx)lnx

lnL = limx→0+

1x

cosh xsinh x

lnL = limx→0+

sinhx

x coshx

Aplicando nuevamente L´H:

lnL = limx→0+

coshx

coshx+ x sinhx

lnL = 1

L = e

48. Considere la curva dada paramétricamente mediante

x(t) = x− 1

t

y(t) = x+ 1t

a) Encontrar la ecuación de la recta tangente a la curva en el punto (x(2), y(2)).

Solución: En primer lugar, la ecuación de la tangente está dada por y − y(2) = y′(2)x′(2) (x− x(2)) , luego:

x(2) = 2− 1

2=

3

2

y(2) = 2 +1

2=

5

2

Además

x′(t) = 1 +1

t2

x′(2) = 1 +1

4=

5

4

y′(t) = 1− 1

t2

= 1− 1

4=

3

4

Entonces:

mT =y′(2)

x′(2)

=3

5

Luego

y − 5

2=

3

5

(x− 3

2

)y =

3

5x− 8

5

Page 86: Apuntes Resueltos Todo Calculo 1

86 CAPÍTULO 1. PROBLEMAS RESUELTOS.

b) Analice el crecimiento y decrecimiento de la curva

Solución: Notemos que

dy

dx=

y′(t)

x′(t)

=1− 1

t2

1 + 1t2

=t2 − 1

t2 + 1

Entonces, la función es creciente cuando

dy

dx> 0 ⇔ t2 − 1 > 0

⇔ (t− 1)(t+ 1) > 0

⇔ t ∈]−∞,−1[∪]1,+∞[

Por otro lado, la función es decreciente si

dy

dx< 0 ⇔ t2 − 1 < 0

⇔ t ∈]− 1, 1[

c) Calcule limt→0±

x(t) , limt→0±

y(t) , limt→±∞

x(t) y limt→±∞

y(t) . Deduzca como se comporta la curva.

Solución: Calculando lo pedido, se tiene

limt→0+

x(t) = limt→0+

(t− 1

t

)= −∞

limt→0−

x(t) = limt→0−

(t− 1

t

)= +∞

limt→0+

y(t) = limt→0+

(t+

1

t

)= +∞

limt→0−

y(t) = limt→0−

(t+

1

t

)= −∞

Sintetizando, si t→ 0− , desde muy lejos en el eje real x positivo, la componente y decrece indenidamente,por otro lado, si t→ 0+ se tiene que desde muy lejos en el eje real x negativo, la componente y viene desde+∞ decreciendo.

Page 87: Apuntes Resueltos Todo Calculo 1

1.5. DERIVADAS 87

Por otro lado

limt→−∞

x(t) = limt→−∞

(t− 1

t

)= −∞

limt→−∞

y(t) = limt→−∞

(t+

1

t

)= −∞

limt→+∞

x(t) = limt→+∞

(t− 1

t

)= +∞

limt→+∞

y(t) = limt→+∞

(t+

1

t

)= +∞

Luego, si t → −∞, tanto la componente x como y decrecen indenidamente, por otro lado si t → +∞ , tantox como y crecen indenidamente.

49. Dada la curva

x(t) = 1−t2

1+t2

y(t) = t−t31+t2

a) Calcule x′(t) e y′(t)

Solución: Determinaremos x′(t) e y′(t) , luego:

x′(t) =−2t(1 + t2)− 2t(1− t2)

(1 + t2)2=

−4t

(1 + t2)2

y′(t) =(1− 3t2)(1 + t2)− (t− t3) · 2t

(1 + t2)2=

1− 5t2 − t4

(1 + t2)2

b) Calcule la ecuación de la recta tangente a la curva en (x(1), y(1))

Solución: Calculando la ecuación de la recta tangente en (x(1), y(1)) , tenemos:

y − y(1) =y′(1)

x′(1)(x− x(1))

Donde:

y′(1) =1− 5− 1

22= −5

4

x′(1) =−4

22= −1

y(1) = 0

x(1) = 0

Reemplazando:

y =5

4x

a) Calcule limt→∞

x(t) , limt→∞

y(t). ¾Qué puede decir de la curva cuando el parámetro t crece indenidamente?

Page 88: Apuntes Resueltos Todo Calculo 1

88 CAPÍTULO 1. PROBLEMAS RESUELTOS.

Solución: Al calcular los límites pedidos queda:

limt→∞

x(t) = limt→∞

1− t2

1 + t2= −1

limt→∞

y(t) = limt→∞

t− t3

1 + t2=∞

Esto nos indica que estamos en presencia de una asíntota vertical en x = 1 para esta curva.

a) Determine los puntos en donde la tangente es paralela al eje X y al eje Y .

Solución: La tangente es paralela al eje X cuando y′(t) = 0 y esto ocurre cuando 1 − 5t2 − t4 = 0. Estaecuación bicuadrática tiene como soluciones:

t1 = −√

5+√

292

t2 =

√5+√

292

t3 = −√

5−√

292

t4 =

√5−√

292

Similarmente, la tangente será paralela al eje Y cuando x′(t) = 0 y esto ocurre cuando −4t = 0 y esto es cuandot = 0.

a) Determine la ecuación cartesiana de la curva.

Solución: Note que:

y = t · 1− t2

1 + t2︸ ︷︷ ︸x(t)

== tx

Por otro lado:

x =1− t2

1 + t2⇒ x(1 + t2) = 1− t2 ⇒ t2(1 + x) = 1− x⇒ t = ±

√1− x1 + x

Como y = tx tenemos nalmente que:

y = ±x√

1− x1 + x

50. Considere la curva denida mediante

x(t) = 2 cos3 t

y(t) = 2 sin3 t, t ∈ [0, 2π]

a) Determine el rango de valores que pueden tomar x e y . Deduzca la región del plano XY , donde se encuentrael gráco de la curva y calcule los puntos (x, y) correspondientes a t = 0, π2 , π,

3π2 , 2π.

Solución: Por acotamiento, sabemos que

−1 ≤ sin t ≤ 1

−1 ≤ cos t ≤ 1

Page 89: Apuntes Resueltos Todo Calculo 1

1.5. DERIVADAS 89

lo que implica,

x(t) ∈ [−2, 2]

y(t) ∈ [−2, 2]

Por lo tanto, la curva se encuentra dentro del rectángulo [−2, 2]× [−2, 2]. Para los puntos indicados:

t = 0 ⇒ (x (0) , y (0)) = (2, 0)

t =π

2⇒

(x(π

2

), y(π

2

))= (0, 2)

t = π ⇒ (x (π) , y (π)) = (−2, 0)

t =3π

2⇒

(x

(3π

2

), y

(3π

2

))= (0,−2)

t = 2π ⇒ (x (2π) , y (2π)) = (2, 0)

b) Determine los puntos del plano XY donde la tangente a la curva es paralela al eje X y al eje Y .

Solución: La tangente es paralela al eje X si

dy

dx= 0 ⇔ y′(t)

x′(t)= 0

⇔ 6 sin2 t · cos t

−6 cos2 t · sin t= 0

⇔ − tan t = 0

⇔ t = 0, π, 2π

Por otro lado, la tangente es paralela al eje Y si

dx

dy= 0 ⇔ −6 cos2 t · sin t

6 sin2 t · cos t= 0

⇔ cot t = 0

⇔ t =

π

2,

2

c) Analice el crecimiento de la curva en el plano XY cuando t ∈ [0, π2 ].

Solución: Del item anterior, se sabe que

dy

dx= − tan t

En t ∈ [0, π2 ] , tan t > 0 , entonces

− tan t < 0dy

dx< 0

Por tanto para ese intervalo, la función es decreciente.

d) Demuestre que para t ∈ [0, π2 ] , la curva es convexa.

Page 90: Apuntes Resueltos Todo Calculo 1

90 CAPÍTULO 1. PROBLEMAS RESUELTOS.

Solución: Se tiene de la segunda derivada

d2y

dx2=

d

dx(− tan t)

= −ddt (− tan t)

dxdt

= − sec2 t

−6 cos2 t · sin t

=1

cos4 t · sin t

Analizando la expresión, vemos que para x ∈ [0, π2 ] , cos4 t > 0 y sin t > 0 , luegod2y

dx2> 0 por lo que la curva

en ese intervalo, es convexa.

e) Demuestre que 3√

(x(t))2 + 3√

(y(t))2 = 3√

4

Solución: Tenemos que

3√

(x(t))2 + 3√

(y(t))2 =3

√(2 cos3 t)

2+

3

√(2 sin3 t

)2=

3√

4(cos2 t+ sin2 t

)=

3√

4

1.6. Integrales: Métodos de Integración.

1. Calculeˆ

(x+ 1)dx√x2 + 2x+ 3

Solución: Haciendo el cambio u = x2 + 2x+ 3⇒ du = 2(x+ 1)dx⇔ du

2= (x+ 1)dx

ˆ(x+ 1)dx√x2 + 2x+ 3

=1

2

ˆdu√u

=√u+ C

=√x2 + 2x+ 3 + C

2. Calculeˆ

cos(2x) ·√

4− sin(2x)dx

Solución: Sea t = 4− sin(2x)⇒ dt = −2 cos(2x)dx⇔ −dt2

= cos(2x)dx , entonces

ˆcos(2x) ·

√4− sin(2x)dx = −1

2

ˆ √tdt

= −1

2· 2

3t32 + C

= −1

3(4− sin(2x))

32 + C

3. Calculeˆ

xdx√1 + x2 +

√(1 + x2)3

Page 91: Apuntes Resueltos Todo Calculo 1

1.6. INTEGRALES: MÉTODOS DE INTEGRACIÓN. 91

Solución: Primero realizaremos la sustitución u = 1 + x2 ⇒ du

2= xdx, entonces

ˆxdx√

1 + x2 +√

(1 + x2)3

=1

2

ˆdu√

u+√u3

=1

2

ˆdu√

u(1 +√u)

=1

2

ˆdu

√u ·√

1 +√u

Ahora sea p = 1 +√u⇒ dp =

1

2√udu , luego

1

2

ˆdu

√u ·√

1 +√u

=

ˆdp√p

= 2√p+ C

= 2

√1 +√u+ C

= 2

√1 +

√1 + x2 + C

4. Calculeˆx−√

arctan 3x

1 + 9x2dx

Solución: Separando la integral:

I =

ˆx−√

arctan 3x

1 + 9x2dx =

ˆxdx

1 + 9x2︸ ︷︷ ︸J

−ˆ √

arctan 3x

1 + 9x2dx︸ ︷︷ ︸

K

Para J =

ˆxdx

1 + 9x2sea u = 1 + 9x2 ⇒ du = 18xdx , luego:

J =1

18

ˆdu

u=

1

18ln |u|+ CJ =

1

18ln |1 + 9x2|+ CJ

Para K =

ˆ √arctan 3x

1 + 9x2sea u = arctan 3x ⇒ du =

3dx

1 + 9x2, luego:

K =1

3

ˆ √udu =

2

9u

32 + CK =

2

9(arctan 3x)

32 + CK

Finalmente:

I =1

18ln |1 + 9x2| − 2

9(arctan 3x)

32 + C

5. Calculeˆepx sin(qx)dx

Solución: Usando integración por partes se tiene:

u = sin(qx)⇒ du = q cos(qx)dx

v =1

pepx ⇒ dv = epxdx

I =1

pepx sin(qx)− q

p

ˆepx cos(qx)dx

Page 92: Apuntes Resueltos Todo Calculo 1

92 CAPÍTULO 1. PROBLEMAS RESUELTOS.

Para la integral que surgió, realizamos nuevamente integración por partes quedando:

u = cos(qx)⇒ du = −q sin(qx)dx

v =1

pepx ⇒ dv = epxdx

Entonces:

I =1

pepx sin(qx)− q

p

1

pepx cos(qx) +

q

p

ˆepx cos(qx)dx︸ ︷︷ ︸

I

I =

1

pepx sin(qx)− q

p2epx cos(qx)− q2

p2· I

I

(p2 + q2

p2

)=pepx sin(qx)− qepx cos(qx)

p2

I =pepx sin(qx)− qepx cos(qx)

p2 + q2+ C

6. Sea In =

ˆsinn xdx , donde n ∈ N . Demuestre que In = − sinn−1 x cosx

n+n− 1

nIn−2

Solución: La integral se puede escribir como

In =

ˆsinn−1 x · sinxdx

Integrando por partes

u = sinn−1 x ⇒ du = (n− 1) sinn−2 x cosxdx

v = − cosx ⇒ dv = sinxdx

Luego

In = − cosx sinn−1 x+ (n− 1)

ˆcos2 x sinn−2 xdx

In = − cosx sinn−1 x− (n− 1)

ˆ(1− sin2 x) sinn−2 xdx

In = − cosx sinn−1 x− (n− 1) (In−2 − In)

nIn = − cosx sinn−1 x− (n− 1)In

In = − sinn−1 x cosx

n+n− 1

nIn−2

7. Sea F (m,n) =

ˆxm(1 + x)ndx , donde , m > 0 , n > 0 . Demuestre que

(m+ 1)F (m,n) + nF (m+ 1, n− 1) = xm+1(1 + x)n

Solución: Integrando por partes

u = (1 + x)n ⇒ du = n(1 + x)n−1dx

v =xm+1

m+ 1⇒ dv = xm

Page 93: Apuntes Resueltos Todo Calculo 1

1.6. INTEGRALES: MÉTODOS DE INTEGRACIÓN. 93

Luego

F (m,n) =xm+1(1 + x)n

m+ 1− n

m+ 1

ˆxm+1(1 + x)n−1dx

F (m,n) =xm+1(1 + x)n

m+ 1− n

m+ 1F (m+ 1, n− 1)

(m+ 1)F (m,n) = xm+1(1 + x)n − nF (m+ 1, n− 1)

Finalmente

(m+ 1)F (m,n) + nF (m+ 1, n− 1) = xm+1(1 + x)n

8. Calculeˆ

x2dx

(x2 + 64)32

Solución: realizaremos la sustitución trigonométrica x = 8 tan θ ⇒ dx = 8 sec2 θdθ , por lo tanto la integralqueda

ˆx2dx

(x2 + 64)32

=

ˆ83 · tan2 θ · sec2 θdθ

(82 tan2 θ + 82)32

=

ˆtan2 θ

sec θdθ

=

ˆsec2 θ − 1

sec θdθ

=

ˆsec θdθ −

ˆ1

sec θdθ

= ln | sec θ + tan θ| − sin θ + C

Para volver a las variables originales consideremos el triángulo siguiente:

Figure 1.6.1: Triángulo para sustitución trigonométrica

Aquí θ, se encuentra en el angulo agudo de la parte inferior del triángulo. Podemos apreciar que:

sec θ =

√x2 + 64

8

tan θ =x

8

sin θ =x√

x2 + 64

Page 94: Apuntes Resueltos Todo Calculo 1

94 CAPÍTULO 1. PROBLEMAS RESUELTOS.

Reemplazando, tenemos nalmente:

I = ln

∣∣∣∣∣√x2 + 64

8+x

8

∣∣∣∣∣− x√x2 + 64

+ C

9. Calcule I =

ˆx4 − 2x3 + 3x2 − x+ 3

x3 − 2x2 + 3xdx

Solución: Realizando la división de polinomios

(x4 − 2x3 + 3x2 − x+ 3)÷ (x3 − 2x2 + 3x) = x

(−x+ 3)

Por lo tanto, la expresión del integrando queda

x4 − 2x3 + 3x2 − x+ 3

x3 − 2x2 + 3x= x+

3− xx3 − 2x2 + 3x

Integrando

I =

ˆx4 − 2x3 + 3x2 − x+ 3

x3 − 2x2 + 3xdx =

ˆxdx+

ˆ3− x

x3 − 2x2 + 3xdx

I =

ˆx4 − 2x3 + 3x2 − x+ 3

x3 − 2x2 + 3xdx =

1

2x2 +

ˆ3− x

x3 − 2x2 + 3xdx︸ ︷︷ ︸

J

Para J =

ˆ3− x

x3 − 2x2 + 3xdx =

ˆ3− x

x(x2 − 2x+ 3)dx usando fracciones parciales tenemos que la expresión del

integrando se puede dejar de la forma

3− xx(x2 − 2x+ 3)

=A

x+

Bx+ C

x2 − 2x+ 3

3− xx(x2 − 2x+ 3)

=Ax2 − 2Ax+ 3A+Bx2 + Cx

x(x2 − 2x+ 3)

3− x = x2(A+B) + x(C − 2A) + 3A

Nos queda el sistema A+B = 0C − 2A = −1

3A = 3

Cuyas soluciones son A = 1B = −1C = 1

Por lo tanto, la expresión separada queda

3− xx(x2 − 2x+ 3)

=1

x+

1− xx2 − 2x+ 3

Integrando ˆ3− x

x(x2 − 2x+ 3)dx =

ˆ1

xdx+

ˆ1− x

x2 − 2x+ 3dx︸ ︷︷ ︸

K

J = ln |x|+K + CJ

Page 95: Apuntes Resueltos Todo Calculo 1

1.6. INTEGRALES: MÉTODOS DE INTEGRACIÓN. 95

Para la integral K hacemos la sustitución u = x2 − 2x+ 3⇒ −du2 = (1− x)dx, luego

K =

ˆ1− x

x2 − 2x+ 3dx = −1

2

ˆdu

u= −1

2ln |u|+ CK = −1

2ln |x2 − 2x+ 3|+ CK

Por lo tanto

J = ln |x|+K + CJ = ln |x| − 1

2ln |x2 − 2x+ 3|+ CK + CJ

Finalmente

I =1

2x2 + ln |x| − 1

2ln |x2 − 2x+ 3|+ C

10. Use la sustitución z = x2 para calcular: I =

ˆx3dx

x6 + 1

Solución: Si hacemos z = x2 ⇒ zdz = 2xdx⇒ zdz2 = xdx. Note que x3 = x2 · x, luego

I =1

2

ˆzdz

z3 + 1=

1

2

ˆzdz

(z + 1)(z2 − z + 1)

Separando el integrando en fracciones parciales

z

(z + 1)(z2 − z + 1)=

B

z + 1+

Cz +D

z2 − z + 1=Bz2 −Bz +B + Cz2 +Dz + Cz +D

(z + 1)(z2 − z + 1)=z2(B + C) + z(C +D −B) +B +D

(z + 1)(z2 − z + 1)

Esto nos conduce al sistema C +B = 0C +D −B = 1B +D = 0

Cuyas soluciones B = − 13

C = 13

D = 13

Entonces, la expresión separada

z

(z + 1)(z2 − z + 1)=

1

3

(z + 1

z2 − z + 1

)− 1

3(z + 1)

Integrando

I =

ˆzdz

(z + 1)(z2 − z + 1)=

1

3

[ˆz + 1

z2 − z + 1dz −

ˆdz

z + 1

]=

1

3

ˆ z + 1

z2 − z + 1dz︸ ︷︷ ︸

J

− ln |z + 1|

Para la integral J realizaremos completación de cuadrados, así

J =

ˆz + 1

z2 − z + 1dz =

ˆz + 1

z2 − z + 1 + 14 −

14

dz =

ˆz + 1

z2 − z + 14 + 3

4

dz =

ˆz + 1(

z − 12

)2+(√

32

)2 dz

Sea z − 12 =

√3

2 tan θ ⇒ dz =√

32 sec2 θdθ, luego

J =

ˆ (√3

2 tan θ + 32

) √3

2 sec2 θdθ

34

(1 + tan2 θ

) =

√3

2· 4

3

ˆ (√3

2tan θ +

3

2

)dθ

J =2√3

(−√

3

2ln | cos θ|+ 3

)+ CJ

Page 96: Apuntes Resueltos Todo Calculo 1

96 CAPÍTULO 1. PROBLEMAS RESUELTOS.

Para volver a las variables originales debemos considerar un triángulo similar al del ejercicio (8) (hágalo usted) conello se llega a

cos θ =

√3

2

√((z − 1

2

)2+(√

32

)2)

z − 1

2=

√3

2tan θ ⇒ θ = arctan

(2(z − 1

2

)√

3

)Reemplazando

J =2√3

−√

3

2ln

∣∣∣∣∣∣∣∣√

3

2

√(z − 1

2

)2+(√

32

)2

∣∣∣∣∣∣∣∣+3

2arctan

(2(z − 1

2

)√

3

)+ CJ

Entonces, la integral I queda:

I =1

3

− ln

∣∣∣∣∣∣∣∣√

3

2

√(z − 1

2

)2+(√

32

)2

∣∣∣∣∣∣∣∣+√

3 arctan

(2(z − 1

2

)√

3

)− ln |z + 1|

+ C

Pero como z = x2 nalmente:

I =1

3

− ln

∣∣∣∣∣∣∣∣√

3

2

√(x2 − 1

2

)2+(√

32

)2

∣∣∣∣∣∣∣∣+√

3 arctan

(2(x2 − 1

2

)√

3

)− ln |x2 + 1|

+ C

11. Encuentre una función f(x) tal que su pendiente es m(x) = sin2(√x) para todo x y que pase por el punto

(π2

16 , 0)

Solución: Sabemos que m(x) =df(x)

dxesto implica que f(x) =

ˆm(x)dx , luego

f(x) =

ˆsin2(

√x)dx

Sea u =√x⇒ u2 = x⇒ dx = 2udu , reemplazando

f(x) = 2

ˆu sin2 udu

= 2

ˆu

(1− cos(2u)

2

)du

=

ˆudu−

ˆu cos(2u)du

=u2

2−ˆu cos(2u)du

Integrando por partes la integral que quedó

p = u ⇒ dp = du

q =sin(2u)

2⇒ dq = cos(2u)du

Page 97: Apuntes Resueltos Todo Calculo 1

1.6. INTEGRALES: MÉTODOS DE INTEGRACIÓN. 97

Luego

f(x) =u2

2− u sin(2u)

2+

1

2

ˆsin(2u)du

=u2

2− u sin(2u)

2− 1

4cos(2u) + C

f(x) =x

2−√x sin(2

√x)

2− 1

4cos(2

√x) + C

Ahora si pasa por el punto

(π2

16, 0

), entonces

f

(π2

16

)= 0

π2

32− 1

2· π

4sin(π

2

)− 1

4cos(π

2

)+ C = 0

π2

32+ C = −π

8

C = −π8− π2

32

Por lo tanto

f(x) =x

2−√x sin(2

√x)

2− 1

4cos(2

√x)− π

8− π2

32

Page 98: Apuntes Resueltos Todo Calculo 1

98 CAPÍTULO 1. PROBLEMAS RESUELTOS.

Page 99: Apuntes Resueltos Todo Calculo 1

Capítulo 2

Problemas Propuestos.

2.1. Inecuaciones

1. Resolver las siguientes inecuaciones en R, e indique el supremo e ínmo si es que existen

a)3x+ 2

x+ 16

1

x+ 1

b) 2x2 − 12x+ 5 6 x(x− 8)

c)√x+ 14 > x+ 2

d)√x2 − x− 2 6 x

e)(3− 2x)(x+ 5)

(x2 − 25)(x+ 1)> 0

f )(x2 + 11x+ 24)

√x− 6

(x2 + 7)(x+ 1)> 0

g)√x2 − 2x− 1 <

√x2 + x+ 1

2. Resolver las siguientes inecuaciones en valor absoluto e indique el supremo e ínmo si es que existen:

a) |x− 3| > 7

b) |x− 5| 6 −4

c)

∣∣∣∣2x+ 4

x− 3

∣∣∣∣ < x+ 2

d) |x+ 1| − 2|x− 3| > xe) |x|+ |x+ 5| < |x+ 9|f ) |2x− 3|+ |6− 4x| 6 |x2 + 2|g) |x+ 1|+ ||x− 1|+ 3| ≤ |x+ 2|+ 8

h)||x+ 3| − 2|||x| − 1|

≥ 2

i)√|2x− 1| − 4 < x− 1

j )(x− 3)2(

√1− |x− 3| −

√4− |x|)

x2 + x+ 1≤ 0

3. ¾Para qué valores de m ∈ R la expresión mx2 + (m− 1)x+ (m− 1) < 0, ∀x∈ R?

99

Page 100: Apuntes Resueltos Todo Calculo 1

100 CAPÍTULO 2. PROBLEMAS PROPUESTOS.

4. Si a , b ∈ R+ , demuestre que (1

a+

1

b

)(a+ b) ≥ 4

5. Verique que ∀α > 0 , x > 0:x2

α+ 1− (x+ 1)2

α+ 1> 0

6. Demuestre que si a+ b+ c = 1, entonces:(1

a− 1

)(1

b− 1

)(1

c− 1

)≥ 8

7. Demuestre quea+ b+ c ≥

√ab+

√bc+

√ac

8. Demuestre que ∀x > 0 , x2 +2

x≥ 3. (Indicación: Analice el producto (x− 1)2(x+ 2)).

9. Considere los conjuntos:

a) M =x ∈ R : |x|4 − 3|x|3 − 10|x|2 ≥ 0

b) F =

x ∈ R :

1

x+ |x| ≥ 8

c) L =

x ∈ R :

x3 − 3x− 2

x|x|≤ 0

d) C =

x ∈ R :

x2 − 18x+ 80

|x− 3|+ 4≤ 0

1) Demuestre que 2 ∈ L2) Determine C ∪M ∪ L ∪ F3) Determine (M ∩ L) ∪ (F ∩ C)

10. Calcule el conjunto solucion de inecuacion

√(x− 2)

4 (x2 − 3

)|x− 1|2 (x2 + x+ 1)

≤ 0 , para x ∈ R

11. Considere a ∈ R. Entonces:

a) Demuestre que 2− a+ |a| < 0 Para cualquier a

b) Para que valores de a la ecuacion1

x2= |2 + |a| − a| − 2|a|a tiene solucion(es) real(es)

12. Considere el conjunto D =

x ∈ R |

√4x+ 1−

√x+ 6√

x− 1< 1 ∧ x

√x2 + x+ 1

(x+ 4)> 0

a) Determine explicitamente el conjunto D. (tambien llamado sistema de inecuaciones)

b) Analice acotamiento de D.

Page 101: Apuntes Resueltos Todo Calculo 1

2.2. FUNCIONES 101

2.2. Funciones

1. Determine el Dominio y Recorrido en las siguientes funciones:

a) f(x) =√

3x−√

7

b) f(x) =x2 − 6x+ 9

x2 + 9

c) f(x) =

√1− 2x

8 + 3x

2. Si denimos f(x) =1√

3|x| − |x− 2|. Determine el dominio de la función f

3. Dena las siguientes funciones de manera que no contengan los valores absolutos.

a) f(x) = |x|+ |x− 1| (graque esta función)

b) g(x) = |2x− 3|+ |3x− 4| − |3x− 1| (graque esta función)

c) h(x) =1√

3|x| − |x− 2|

4. Dada f(x), gracar: f(x), |f(x)|, maxf(x), 0 y max−f(x), 0

a) f(x) = 3x− 2

b) f(x) = x3

c) f(x) = x2 − 3x− 1

d) f(x) =

3x− 2, x 6 11− 4x, x > 1

5. Dada f :]− 2, 1]→M tal que f(x) =

√1− xx+ 2

a) ¾Es f una función inyectiva?. Justique

b) Determine el conjunto M tal que la función sea epiyectiva

c) A partir de lo anterior, deduzca que f es invertible y encuentre la función inversa de f .

6. Para las siguientes funciones determine: dominio, ceros, signo y la región del plano en donde se encuentra la curva:

a) f(x) =x

x2 − 9

b) f(x) = x√

3− x

c) f(x) =x2 − 4

x2 − 9

7. Sean las funciones f(x) = x−1x2+1 y g(x) = 2x−1

3x+1 . Determine (fog)(x) y (gof)(x), además, determine el dominio deambas composiciones

8. Dadas las funciones f(x) =

√6− |2x+ 1|√|x+ 3| − 4

y g(x) =√

5− x2

a) Encuentre el dominio de f .

b) Calcule, en caso que exista, (f + g)(2)

c) Determine en caso que exista, las preimagenes de 1 bajo la función g

Page 102: Apuntes Resueltos Todo Calculo 1

102 CAPÍTULO 2. PROBLEMAS PROPUESTOS.

9. Dada la función f , denida como:

f : [0,+∞[ → B

x 7→ f(x) = 1−√x+ 1

Determine B de modo que f sea una función biyectiva.

10. Se dene la funciónf : [0,+∞[→ R según f(x) =x

x2 + 1

a) Encuentre Rec(f)

b) ¾Es f inyectiva? . Justique

11. Considere la función f(x) =2x

1− |x|

a) Determine el dominio, ceros signo y paridad

b) Demuestre que ∀y > 0 , existe un x ∈ [0, 1] tal que y = f(x). Use este resultado para deducir que en ]0, 1[ f esepiyectiva.

c) Determine la región del plano en donde se encuentra la curva.

12. Considere la función f denida por f(x) =x

x2 − |x|.

a) Determine el dominio, ceros, signo y paridad.

b) Estudie el crecimiento y decrecimiento de f

c) Para la función g :]1,+∞[→ g(]1,+∞[) tal que g(x) = f(x) . Demuestre que g admite inversa y calcule g−1(x).

13. Dadas las funciones f(x) =x

x− 1y g(x) = x2 − 1

a) Determine una expresión para ((f + g) g)(x)

b) Determine el dominio de dicha expresión

14. Sean f(x) =1

x2 + 1y g(x) =

x

1 + x, dos funciones . Sea A(h) denida por:

A(h) =f(x+ h)− (f g)(x+ h)

(f · g)(x+ h)− f(x+h)g(x+h)

a) Calcule A(h) en términos de h y x.

b) Calcule A(1) y A(−1).

15. Sea f(x) =

√∣∣∣∣x− 1

x− 2

∣∣∣∣− 1. Determine Dom(f)

16. Sea f(x) =

x+ 1

x− 1, x < 0

x− 1

x+ 1, x ≥ 0

a) Demuestre que f es par y encuentre Dom(f)

b) Calcule f(g(x)) con g(x) = x2 − 5x+ 6

Page 103: Apuntes Resueltos Todo Calculo 1

2.2. FUNCIONES 103

17. Considere f(x) =

x− 5 x < 5

−x− 5 x ≥ 5Determine (f f)(x) y su dominio.

18. Considere las funciones f : R→ R y g : R−1 → R denidas por: f(x) =

2x+ 3 , 9 < x

x− |x| , −9 ≤ x ≤ 9

x− 4 , x < −9

, g(x) =x+ 1

x− 1

a) Determine g−1of

b) Determine f(g(x))

19. Si tan(θ) = x , calcule sin(θ) y cos(2θ) en términos de x .

20. Sean f(x) = sin(x) y g(x) = 2 arctan(x) . Muestre que (f g)(x) =2x

x2 + 1(indicación: Use el ejercicio 19)

21. Considere la función f(x) = 1√2+√

2+√

2+2 cos(8x)

a) Demuestre que f puede ser escrita como f(x) = sec x2

b) Determine el dominio de f y muestre que f no tiene ceros.

22. Considere la función f(x) = 1√cos x

a) Determine el dominio de f

b) Si g(x) = f(x) · cosx , resuelva la ecuación (g(x))2 = − sinx

c) Graque la función h(x) = | sinx|

23. Se tiene la función f(x) = cos(2x) +√

3 sin(2x) . Escriba la función como f(x) = A sin(wx + φ) y determine losparámetros A , w y φ.

24. Se tiene la función f(x) = 2√

2 +√

2 sin(3x) + 2√

2−√

2 cos(3x). Escriba la función como f(x) = A sin(wx+ φ) ydetermine los parámetros A , w y φ.

25. Considere la sinusoide f(x) = cos(π − 2x)−√

3 cos(2x+ π

2

).

a) Encuentre su amplitud, periodo y el ángulo de fase de f .

b) Determine el número de intersecciones de su gráca con el eje OX sobre el intervalo[0, 49π

12

].

26. Un rectángulo de altura x se inscribe en un triángulo ABC de base b y altura h . Exprese el perímetro P y el áreaS del triángulo en función de x.

27. Un cono de radio x se inscribe en una esfera de radio r . Exprese el volumen del cono en función de x.

28. Un triángulo isósceles de perímetro 2p = 12 dado gira alrededor de su base. Exprese la función V (x) , donde V (x)es el volumen del sólido de revolución así obtenido y x es la longitud del lado isósceles del triángulo.

29. En un cuadrado de lado 2a se inscriben don circunferencias de radios r1 y r2 centradas en la diagonal del cuadrado,tangentes entre si y ambas tangentes al cuadrado, como se muestra en la gura

Page 104: Apuntes Resueltos Todo Calculo 1

104 CAPÍTULO 2. PROBLEMAS PROPUESTOS.

Figure 2.2.1: Esquema del problema

a) Demuestre que r1 + r2 =4a

2 +√

2

b) Determine una expresión que modele la suma de las áreas de los círculos en términos de r1.

2.3. Sucesiones:

1. Calcule los siguientes límites

a) limn→∞

5n4 + 3n2 − 4

3n3 − 1

b) limn→∞

(n+ 2)(2n2 − 1)2

(5n+ 9)3(2n− 1)(3n+ 4)

c) limn→∞

1− n√n

d) limn→∞

5n+ 3√n2 + 1

e) limn→∞

αn − βn

αn + βn(separe casos en que α > β , α = β y α < β)

f ) limn→∞

3 · 2n+1 − 2(−3)n+1

5 · 2n−1 − 4(−3)n

g) limn→∞

(√n+ 2−

√n− 1)

h) limn→∞

(√n2 − 3n+ 7−

√n2 − 8n+ 1)

i) limn→∞

( 3√n+ 1− 3

√n− 1)

j ) limn→∞

n∑k=1

7 · 1

4k

k) limn→∞

n∑k=0

(−4

3

)kl) lim

n→∞

(1 +

1

25+

1

625+ ...+

1

52n

)

Page 105: Apuntes Resueltos Todo Calculo 1

2.3. SUCESIONES: 105

m) limn→∞

n∑k=1

1

(k + 1)(k + 2)

n) limn→∞

n∑k=1

1

(k + 1)(k + 3)

ñ) limn→∞

(1− 1

n

)no) lim

n→∞

(1 +

1

7n

)np) lim

n→∞

(n+ 1

n+ 3

)nq) lim

n→∞

(2n+ 5

2n+ 7

)3n

r) limn→∞

√(n2 + 5n+ 6

n2 + 7n+ 12

)ns) lim

n→∞

3 + 6 + 9 + ...+ 3n

8 + 16 + 24 + ...+ 8n

2. Usando acotamiento calcule los siguientes límites:

a) limn→∞

(1

n4 + 1+

8

n4 + 2+ ...+

n3

n4 + n

)b) lim

n→∞

n∑k=1

1√n2 + k

c) limn→∞

2n + 3√

ncos(nn

n!

)+ 2n+1

3−2n

2n

n! + (−1)n

n + 11− n!

nn

3. Determine los valores de φ ∈ R tal que limn→∞

(n3 − 4

2n2 + 1− 1− φn2

4n+ 5

)∈ R

4. Calcular el límite:

limn→∞

(1 + n)n(1 + 2n)2n(n∑i=1

12i2

)n

5. Considere la sucesión:

a1 = 2

5

an+1 = 2+4an5

a) Calcule a2,a3 y a4

b) Pruebe usando inducción que la sucesión es creciente

c) Pruebe que la sucesión es acotada superiormente por 3

d) Analice la existencia del límite y calcúlelo.

6. Considere:

a1 = 2

an+1 =√

3 + an. Pruebe que es creciente, además, demuestre que es acotada superiormente por 8 y

calcule su límite.

7. Considere la sucesion (sn) denida por: s1 = 1 y sn+1 =

√9 + s2

n

2, ∀n ≥ 1.

Page 106: Apuntes Resueltos Todo Calculo 1

106 CAPÍTULO 2. PROBLEMAS PROPUESTOS.

a) Pruebe que (sn) es acotada superiormente por 3.

b) Verique que (sn) es creciente.

c) Deduzca que (sn) converge y calcule a qué valor lo hace.

8. Considere la sucesión (xn)n≥1 tal que x0 ∈ [0, 1] y xn+1 = xn − x2n + x3

n − x4n , ∀n ≥ 0

a) Pruebe que esta sucesión es convergente.

b) Calcule limn→∞

xn.

9. Considere la sucesión (xn)n∈N denida por 0 < x1 < 1 y xn+1 = 1−√

1− xn para n = 1, 2, ....

a) Pruebe que (xn)n∈N es decreciente y que limn→∞

xn = 0

b) Demuestre que limn→∞

xn+1

xn=

1

2

10. Use el hecho de que limn→∞

an+1

an= limn→∞

n√an = L para calcular: lim

n→∞n

√33n(n!)3

(3n)!

11. Considere la sucesión (an)n≥1 tal que Sn =

n∑k=1

ak =3n2 + 9n

2

a) Pruebe que (an)n≥1 es una progresión aritmética.

b) Calcule limn→∞

1

nan

n∑k=1

ak

12. Analice la convergenia de la sucesión an : n ≥ 1 denida por an =b− an

n∑k=1

f

(a+

k(b− a)

n

)en donde f(x) = x2

13. Se deja caer una pelota desde a metros de altura sobre una supercie plana. Cada vez que la pelota toca la supercie,después de caer una distancia h, rebota hasta una distancia rh, donde r es positiva, pero menor que 1. Determinela distancia total que viaja la pelota hacia arriba y hacia abajo.

14. Dentro de un círculo de radio r se inscribe un cuadrado, en este cuadrado se inscribe un circulo y así sucesivamente.Calcule la suma de las áreas de los n cuadrados y de los n circulos. además, determine el límite de cada una de ellas.

15. Este ejercicio se reere al triángulo equilátero con lados de longitud 2b en la gura siguiente. Del triángulo originalse quitan los triángulos equiláteros que están de cabeza, como sugiere la sucesión de guras. La suma de las áreaseliminadas del triángulo original forma una serie innita. Determine esta serie innita, es decir, Aeliminada = lim

n→∞Sn

Page 107: Apuntes Resueltos Todo Calculo 1

2.4. LÍMITE Y CONTINUIDAD 107

Figure 2.3.1: Esquema del problema

16. Considere una circunferencia de radio r en donde hay un polígono inscrito en ella de n lados como se muestra en lagura

Figura 2.3.2: Situación geométrica

a) Pruebe que el área del polígono An = nr2 cos(πn

)sin(πn

)b) Demuestre que lım

n→∞An = πr2

2.4. Límite y Continuidad

1. Calcule los siguientes límites

a) limx→1

x3 − 1

x− 1

b) limx→−2

x3 + 8

x+ 2

c) limx→2

5x2 − 13x+ 6

4x2 − 9x+ 2

Page 108: Apuntes Resueltos Todo Calculo 1

108 CAPÍTULO 2. PROBLEMAS PROPUESTOS.

d) limx→1

√x+ 3− 2√x− 1

e) limx→0

√1− x2 − 1

x2

f ) limx→1

3√x− 1√x− 1

g) limx→a

x2 − x(a+ 1) + a

x3 − a3

h) limx→+∞

3√

1 + x2 − 3√

1 + x

x− x3

i) limx→1

x2 − x− 6

x2 − 4

j ) limx→0

x5 − 2x3 − 2x2

x4 − 5x2

k) limx→1

x8 − 1

x5 − 1

l) limx→−2

(1

x2 − x− 6+

1

x2 + 2x

)m) lim

x→1

3√x− 1

4√x− 1

n) limx→1

xn − 1

xm − 1, m, n ∈ N

ñ) limx→∞

5e3x − ex + 1

2e3x − e2x + 3

o) limx→0

34x − 1

x

p) limx→0

ex − e−x

x

q) limx→0

e3x − e2x

e6x − e4x

r) limx→π

4

sinx− cosx

1− tanx

s) limx→π

2

1− sinxπ2 − x

t) limx→π

2

√sin2 x− 2 sinx+ 6−

√sin2 x+ 2 sinx+ 2

sin2 x− 4 sinx+ 3

u) limx→π

sinx

x− π

v) limx→0

sin(8x)

e3x − e7x

w) limx→0

√2−√

1 + cosx

sin2 x

x ) limx→0

√1 + x sinx−

√cos 2x

tan2(x2

)y) lim

x→0

sin(a+ x) + sin(a− x)− 2 sin a

x2

Page 109: Apuntes Resueltos Todo Calculo 1

2.4. LÍMITE Y CONTINUIDAD 109

z ) limx→1

lnx

ex−1 − 1

2. Calcular los límites:

a) limx→∞

(2x+ 1) sin

(1

x

)b) lim

x→3+(7− 2x)

1x−3

3. Calcular limx→3

√9 + |x− 3| − 3

x3 − 27

4. Dada la función f(x) =3|x−1|+2

√|x|−1

x2−3x−4 . Analice la existencia de

a) lımx→+∞

f(x)

b) lımx→−∞

f(x)

c) lımx→1

f(x)

d) lımx→−1

f(x)

5. Dado n ∈ N y x ∈ R se dene: f(x) = limn→∞

1

1 + x2n+1

a) Calcule f(1) , f( 12 ) , f(− 1

2 ) , f(2) , f(−2)

b) Determine el dominio de f

c) Calcule limx→1

f(x) y limx→−1

f(x)

6. Determine Dom(f) . De acuerdo a ello analice la continuidad y la existencia de asíntotas horizontales y vertivalespara las siguientes funciones:

a) f(x) = 5sin(6x)

b) g(x) =√

4cos x

c) h(x) = 1cos x−sin x

7. Determine las asíntotas horizontales, verticales y oblicuas de:

a) f(x) =x3 − 3x2 − 10x√

x6 + 1

b) f(x) =x2

√x2 + 1

8. Calcule todas las asíntotas de la función: f(x) =

arctanx x ≤ 0

sin xx(x−1) 0 < x < 1

1 x = 12+x+x2

1−x2 e−1x2 x > 1

9. Estudiar si existe limx→1

f(x) con f(x) =

2−√x+3

x−1 , x > 12x2−3x2+3 , x < 1

Page 110: Apuntes Resueltos Todo Calculo 1

110 CAPÍTULO 2. PROBLEMAS PROPUESTOS.

10. Sea f : R→ R denida por f(x) =

(1 +Ax2) sin

(3πx

2

)x ≤ 1

1− 3√

2x−11−√

2x−1x > 1

.Calcule el valor de A tal que f sea contínua en

x = 1.

11. Considere la función f(x) =

a|x|+ sin(πx)

xx 6= 0

b x = 0. Calcule los límites laterales lim

x→0+f(x) , lim

x→0−f(x) y deter-

mine los valores de a y b para los cuales f es contínua en x = 0.

12. En los siguientes casos, analice la continuidad en todo punto del dominio de cada una de las funciones siguientes:

a) f(x) =

√x+5−

√5

x , x 6= 0√5

2 , x = 0

b) f(x) =

√2+ 3√t−2t−8 , t 6= 8

1, t = 8

13. Determine los valores de A y B de modo que las funciones presentadas sean continuas en todo R

a) f(x) =

x2 x < −1Ax+B −1 ≤ x < 41− 2x x ≥ 4

b) f(x) =

Ax− 1 x < −1Ax2 +B −1 ≤ x < 13x− 2B x ≥ 1

14. Si f(x) =

arctanL(x+1)

x+1 x < −1

L−Mx2 |x| ≤ 1eM(x−1)−1

(x−1) x > 1

, determine los valores de M y L para que f sea una función contínua.

15. Determine los valores de c ∈ R de modo que f sea contínua en x = 0, donde:

f(x) =

x3−πx

x+sin(3x) x 6= 0

c x = 0

16. Muestre que limx→π

2

f(x) no existe si f(x) =

secx− tanx, x < π

2π−2x

sin(x−π2 )x > π

2

17. Sea f(x) =

a(1− x) tan

(πx2

)0 < x < 1

2b x = 1x−13√x−1

x > 1

. Determine a, b ∈ R tal que f sea contínua.

18. Determine los valores de A y B de modo que f sea contínua en −π2 y π2 , donde

f(x) =

cos( x2 )+sin( x2 )

cos(x) −π < x < −π2A sin(x) +B −π2 < x < π

2(√2−√

1+cos(x−π2 ))·(x−π2 )

sin3(x−π2 )π2 < x < π

19. Sea g(x) =(π4 − x

)tan(2x)

a) Calcule, si existe, limx→π

2

g(x)

Page 111: Apuntes Resueltos Todo Calculo 1

2.4. LÍMITE Y CONTINUIDAD 111

b) Calcule , si existe, limx→π

4

g(x)

c) Es claro que π4 no está en el dominio de g . ¾Es posible redenir g en dicho punto de modo que sea contínua?

. Justique.

20. Considere una circunferencia de radio r y centro O en donde ]TOQ = x , la recta PT es tangente a la circunferenciay TQ es perpendicular al diámetró AA′ como se muestra en la gura_

Figure 2.4.1: Descripción gráca

Al respecto:

a) Calcule limx→0

QA

AP

b) Calcule limx→0

AP

(QT )2

21. Sea F centro de una circunferencia de radio r. M ABC isósceles, AC = BC inscrito en la circunferencia como semuestra en la gura:

Figure 2.4.2: Triángulo inscrito

a) Calcule limAB→0

Área M AFC

Área sector circular AFB

b) Calcule limAB→0

Área M ABC

Área sector circular AFB

22. Se tiene un semicírculo de radio AB descansa sobre la base de un triángulo isósceles ABC donde los lados igualesmiden a, para así formar la región de la gura. Si A(θ) es el área del semicírculo y B(θ) es el área del triángulo, se

pide calcular limθ→0+

A(θ)

θ ·B(θ)

Page 112: Apuntes Resueltos Todo Calculo 1

112 CAPÍTULO 2. PROBLEMAS PROPUESTOS.

Figure 2.4.3: Descripción gráca

Page 113: Apuntes Resueltos Todo Calculo 1

2.5. DERIVADAS 113

2.5. Derivadas

1. Usando el hecho quedy

dx= lımh→0

f(x+ h)− f(x)

h. Demuestre que:

a) Si y = lnx ⇒dy

dx=

1

x

b) Si y = xn ⇒dy

dx= nxn−1, n∈ N

2. Demuestre que si g(x) = f(cx) donde c ∈ R⇒ g′(x) = cf ′(cx)

3. Considere la ecuación que describe la posición en función del tiempo como: x(t) = x0 + v0t+ 12at

2 ; donde x0, v0 ya son constantes.

a) Calcule la velocidad media vm sabiendo que vm = ∆x∆t = x(t)−x(t0)

t−t0 y calcule lımt→t0

vm = v(t0)

b) Calcule la aceleración media am sabiendo que am = ∆v∆t = v(t)−v(t0)

t−t0 y calcule lımt→t0

am = a(t0)

4. Analice la derivabilidad en x = 0 de las siguientes funciones:

a) f(x) =

x2 sin

(1x

), x 6= 0

0 , x = 0

b) f(x) =

1−cos x

x , x 6= 0

0 , x = 0

5. Considere la función denida como f(x) =

x arctan

(1x

)x 6= 0

0 x = 0

a) Analice la continuidad de f en x = 0

b) Estudie la existencia de f ′(x) en x = 0

6. Sea f : R→ R denida por f(x) =

x2 − π x ≤ πx2 − π cosx x > π

a) Calcule f ′(x) en todos los puntos donde ella exista.

b) Analice la existencia de f ′′(π).

7. Muestre que f(x) =1

1 + |x|es contínua en x = 0 pero no derivable en ese punto.

8. Calcule la derivada de las siguientes funciones:

a) f(x) = x2 + 3x+ 4− (x− 1)(x− 3)

b) f(x) = 2x+33x+2

c) f(x) = (x+ 1)(x+ 4)− x2+13x4+2

d) f(x) = lnx · cosx+ tanx · arcsinx

e) f(x) = 4x tan xln x

f ) f(x) = sec x log3 x1+exp x + cosx · log x

g) f(x) =√

1 + 2x3

Page 114: Apuntes Resueltos Todo Calculo 1

114 CAPÍTULO 2. PROBLEMAS PROPUESTOS.

h) f(x) = 4√

1 + x5 · (1 + x)2

i) f(x) = ln (1 + sinx)− arctan(1 +√x)

j ) f(x) =√

ln (sin(1 + (2x3 + 1)(3x+ 2)))

k) f(x) =√

1+ln x3+exp(3(x−1)2) − ln(sin

√x)

l) f(x) = βφ arctan

(xφ

)exp

(1− x2

β

)+ 3x5β2φ√

β2x3−φ, β ∧ φ ∈ R

m) f(x) = (1 + ln(ax)) arcsin(1− cos(x2))−√

1 +3√x2

n) f(x) = 1+x√x+1√

1+ln(1+e2x2 )− arctan(5 +

√2x+ 6x5) · cos

3√x7

9. Determinedy

dxen las siguientes funciones implítitas ó por el método logarítmico según corresponda:

a) y + x2 − 2xy = 8

b) (xy + 2)2 − (5x+ 4y) = 2x2y2

c) 3x− 4y = 1−√x 3√y2 − 1

d) 4 sin(y) cos(2x) = 1 + sin(xy + 2y)

e) ln(1 + 3eπy) + tan(√xy) = sin(y2 − 3x2)

f ) y = xln x

g) y = (2x+ 4)sin x

h) y =(exp

(2(1− x2)

))arctan x

i) y = (cosx)1+ln(x2)

+ (1 + sinx)exp(3x)

j ) y = (3x2+1)tan x

(x2+sin x)log x

10. Hallar las rectas tangentes y normales em P0 para las siguientes curvas:

a) x2 + xy + y2 = 1, P0 = (2, 3)

b) x2 + y2 = 25, P0 = (3,−4)

c)x− yx− 2y

= 2, P0 = (3, 1)

d) y − x2 = 2x+ 4, P0 = (6, 2)

e) x3y + arcsin(y − x) = 1 en el punto (1, 1)

f ) y2 + x2y2 + cos(y2) = x− π2 + sinx en el punto

(π2 , 0)

g) f(x) = arcsin(

ln x1+ex

)en el punto (1, f(1))

11. Determine en qué puntos las funciones presentadas tienen una tangente horizontal.

a) f(x) = x2 + 4x− 1

b) f(x) = x3 − 3x

c) f(x) = sin(3x+ π

4

)12. ¾Existen puntos en la curva y =

x

2+

1

2x− 4donde la pendiente sea −3

2? . Si es así, encuéntrelos.

13. Determine los puntos en la curva f(x) = 2x3 − 3x2 − 12x+ 20 donde la tangente es

Page 115: Apuntes Resueltos Todo Calculo 1

2.5. DERIVADAS 115

a) Paralela al eje X

b) Perpendicular a la recta y = 1− x

24c) Paralela a la recta y = −12x

14. Demuestre que las tangentes a la curva f(x) =π sinx

x, en x = −π y en x = π se intersectan en ángulos rectos

15. ¾Para qué valores de c, la curva f(x) =c

x+ 1es tangente a la recta que pasa lor los puntos (0, 3) y (5,−2)?

16. Sea a > 0 , considere la ecuación√x +√y =√a , sean p y q los puntos donde se intersecta la recta tangente a la

curva con el eje OX y OY respectivamente. Demostrar que p+ q es constante.

17. Demuestre que la tangente trazada en un punto cualquiera P0(x0, y0) de la hipérbola xy = 1 , determina en el ejeX un punto A talque el triángulo OAP0 es isósceles.

18. ¾En qué puntos la curva y = 2x3 + 13x2 + 5x+ 9 , tiene tangentes que pasan por el origen?

19. Demuestre que las curvas y = x2 e x = y2 se cortan ortogonalmente

20. Determine f ′(0) si f(x) =u(x) · (1 + tanx)

v(x)donde u y v funciones tales que

du

dx=

1√1 + x2

;dv

dx= v(x) , u(0) = 0

y v(0) = 1

21. Determine los dos puntos donde la curva x2 + xy + y2 = 7 cruza el eje X y demuestre que las tangentes a la curvaen estos puntos son paralelas. ¾Cuál es la pendiente común a tales tangentes?.

22. Demuestre que las funciones presentadas:

a) f(x) = sinx

b) f(x) = cosx

c) f(x) = a sinx+ b cosx

satisfacen la ecuación:f ′′ + f = 0

¾Son las dos primeras funciones un caso particular de la tercera?. Justique.

23. Dada la siguiente función: f(x) = 1 + x+ x2 + x3 + ...+ xn. Pruebe que:

df

dx=nxn+1 − (n+ 1)xn + 1

(x− 1)2

24. Una partícula de masa constante m se desplaza a lo largo del eje X. Su velocidad v y su posición x satisfacen larelación:

1

2m(v2 − v2

0) =1

2k(x2

0 − x2)

donde k, v0 y x0 son costantes. Demuestre que siempre que v 6= 0:

mdv

dt= −kx

25. Considere la función f(x) = sin(kx)x , k 6= 0, demuestre que f satisface la relación:

f ′′ +2

xf ′ = −k2f

Page 116: Apuntes Resueltos Todo Calculo 1

116 CAPÍTULO 2. PROBLEMAS PROPUESTOS.

26. Se tiene la función f(x) = a

(1

b− x+

1

b− 2x+ ...+

1

b− kx

), a, b, k ∈ R+. Calcule f (n)(x).

27. Sea f : R→R, contínua y diferenciable. Demuestre que:

a) Si f es una función impar, entonces,df

dxes una función par.

b) Si f es una función par, entonces,df

dxes una función impar.

28. Se tiene la función f(x) = a cos(wx) + b sin(wx), entonces:

a) Demuestre que el máximo valor que puede alcanzar f es√a2 + b2

b) Demuestre que: 0 =d2y

dx2+ w2y

29. Sea g : R → R dos veces derivable con g′(x) 6= 0,∀x y f : R → R denida por: f(x) = cos(kg(x)). Demuestre queesta función satisface la ecuación diferencial:

f ′′ − f ′ g′′

g′+ (kg′)2f = 0

30. Sea f(x) =1

x− a1+

1

x− a2+

1

x− a3+ ...+

1x− an

, ai > 0 con i = 1, 2, 3, ..., n. Demuestre que f ′′′(x) < 0, ∀x ∈ R

31. Calcule f (n)(x) , ∀n ∈ N si f(x) = ln(ax+ b) , con a y b constantes reales jas.

32. Dada la función y = arcsinx+ arcsin2 x. Determine φ ∈ R de modo que se cumpla la relación:

(1− x2)y′′ − xy′ = φ

33. Si y = f(u) y u = f(x). Demuestre que:

a)d2y

dx2=dy

du

d2u

dx2+d2y

du2

(du

dx

)2

b)d3y

dx3=dy

du

d3u

dx3+ 3

d2y

du2

d2u

dx2

du

dx+d3y

du3

(du

dx

)3

34. En un triángulo ABC se cumple que a =√b2 + c2 − 2bc cosϕ . Si consideramos que b y c son constantes y ϕ variable

, demuestre queda

dϕ= hϕ , en que hϕ es la altura del triángulo correspondiente a la base a

35. Sean a, b ∈ R tal que 0 < a < b. Demuestre que

a) b−ab < ln

(ba

)< b−a

a (indicación: Considere f(x) = lnx y aplique el T.V.M.)

b) Usando un procedimiento similar pruebe que: b−acos2(a) ≤ tan(b)− tan(a) ≤ b−a

cos2(b)

36. Sea f [0,∞[→ R una función que verica las siguientes propiedades:

f es contínua ∀x ≥ 0f ′existe ∀x > 0f(0) = 0f ′ es estrictamente creciente

Sea g :]0,∞[→ R denida por g(x) = f(x)x

Page 117: Apuntes Resueltos Todo Calculo 1

2.5. DERIVADAS 117

a) Demuestre aplicando el T.V.M. en [0, x] , que f ′(x) > g(x)

b) Deduzca que g es estrictamente creciente.

37. Dada la función: f(x) =x2

x2 + 1

a) Determine su dominio, analice la existencia de ceros y estudie el signo de f

b) Determine si es que existen, asíntotas horizontales y verticales

c) Determine su crecimiento, sus puntos de máximo, mínimo y su recorrido

d) Analice la concavidad y sus puntos de inexión.

e) Esboce el gráco de f

38. Dada la función f(x) =x+ 1√x2 + 1

a) Analice la existencia de asíntotas horizontales y verticales

b) Analice el crecimiento de la curva y la existencia de máximos y mínimos

c) Analice la curvatura y la existencia de puntos de inexión

d) Determine el recorrido y esboce el gráco de f .

39. Considere la curva denida como f(x) =−2(x− 1)3

(x− 2)2

a) Determine Dom(f), ceros y signo.

b) Determine sus asíntotas.

c) Determine crecimiento, decrecimiento asi como la posible existencia de máximos y mínimos.

d) Analice la curvatura y la existencia de puntos de inexión

e) Graque.

40. Dada la función f(x) =1

4cos4 x

a) Determine su dominio, analice su paridad, continuidad y muestre que f tiene periodo π.

b) Determine sus ceros y analice su signo.

c) Analice la existencia de asíntotas horizontales y verticales.

d) Analice el crecimiento y la existencia de máximos y mínimos.

e) Analice la curvatura y puntos de inexión.

f ) Graque.

41. Considere la función f(x) =1

3√

2πexp

(−1

2

(x− 2

3

)2)

a) Verique que f ′(x) = −1

9(x− 2)f(x) y f ′′(x) = −1

9

(1− 1

9(x− 2)2

)f(x)

b) Determine máximos, mínimos y puntos de inexión

c) Analice el crecimiento, decrecimiento y curvatura

d) Determine las asíntotas de la función y graque.

42. Se tiene f(x) = cos2 x− cosx:

Page 118: Apuntes Resueltos Todo Calculo 1

118 CAPÍTULO 2. PROBLEMAS PROPUESTOS.

a) Determine su dominio, ceros, pruebe que f(x+ 2π) = f(x), justique porqué f es contínua y analice su signo.

b) Pruebe que Rec(f) ⊆ [−2, 2] usando solo acotamiento.

c) Analice la existencia de asíntotas horizontales y verticales.

d) Analice su crecimiento y la existencia de máximos y minimos si es que existen.

43. Sea f una función contínua con segunda derivada contínua en R tal que f(x) > 0 y f ′(x) = −x · f(x) , ∀x ∈ R .Encontrar:

a) Intervalos de crecimiento y decrecimiento.

b) Intvalos de concavidad y convexidad

c) Puntos de Inexión

44. Sea f : R→ R dos veces derivable. Demostrar que si ∀x ∈ R , se cumple que f(x) > 0 y que f(x)f ′′(x) > (f(x))2 ,entonces, la función g(x) = ln(f(x)) es convexa.

45. Sean g y h funciones denidas en [−a, a] tales que g es derivable en [−a, a], h tres veces derivable en [−a, a] y

h(0) = h′(0) = 0 , g(0) 6= 0 y h′′(0) 6= 0 . Si f(x) =x2g(x)

h(x), Calcule lım

x→0f(x)

46. La corriente I(t) que circula en un cierto circuito eléctrico en el tiempo t viene dada por I(t) =E

R

(1− e−RtL

),

donde E , R , L son números positivos. Determine limR→0+

I(t)

47. Calcule los siguientes límites:

a) limx→2

ln(x2 − 3)

x2 + 3x− 10

b) limx→0

tanx− xx− sinx

c) limx→∞

2 + 2x+ sin(2x)

(2x+ sin(2x))esin x

d) limx→1

(1

lnx− 1

x− 1

)e) lim

x→0(cos(2x))

nx2

f ) limx→0+

x1

ln x

g) limx→+∞

x

(π − 2 arcsin

(x√

x2 + 1

))h) lim

x→0

ln(1 + sinx)− ln(1 + x)

x− tanx

i) lımx→1

1− x ln(x)

1 + cos(πx)

j ) lımx→0

(1

sin2(x)− 1

x2

)48. Encuentre los valores de c tal que:

limx→∞

(x+ c

x− c

)x= 4

Page 119: Apuntes Resueltos Todo Calculo 1

2.5. DERIVADAS 119

49. Considere f(x) =

x ln(x)x−1 , x > 0 y x 6= 1

α x = 1. Determine α ∈ R para que f sea contínua en R+ y analice la existencia

de f ′(x) para x > 0 , en caso de existir, calcúlela. (De ser necesario, puede usar la regla de L'Höpital. )

50. Hallar la longitud de la escalera, de longitud mínima, que puede pasar por la esquina de un corredor cuyas dimensionesse indican en la gura, se supone que la escalera de transporta parlela al suelo

Figure 2.5.1: Diagrama de la escalera

51. Hallar las dimensiones del cono recto circular de máximo volumen, que puede ser inscrito en una esfera de radio a.

52. Hallar la altura y el radio de la base de un cono recto circular, de volumen mínimo, que se circunscribe una esferade radio r. ¾Cuál es este volumen mínimo?

53. Encuentre la altura del cilindro recto inscrito a una esfera de radio R que tenga la supercie lateral máxima.

54. La resistencia de una viga de sección rectangular el diréctamente proporcional a la anchura y al cubo de la altura.Hallar el ancho de la viga de máxima resistencia que podría ser obtenida de un tronco de madera de 16[cm] dediámetro.

55. Se quiere construir una caja de una hoja de papel cuadrangular de 12 cm. de lado, cortando cuadrados en lasesquinas y doblando los lados. Hallar la longitud del lado del cuadrado que se debe cortar para que el volumen seamáximo.

56. Las ecuaciones del movimiento de un proyectil están dadas por x = v0 cos θ · t ; y = v0 sin θ · t − 16t2 , con v0 lavelocidad inicial, θ el ángulo de elevación del cañon , t el tiempo en segundos y x e y las coordenadas del proyectil.Encuentre la altura máxima del proyectil y muestre que el mayor alzance se obtiene cuando el ángulo de elevaciónes de 45°

57. Un alambre de longitud L se corta en dos partes, una se dobla para que forme un círculo y la otra para que forme uncuadrado. ¾Cómo se debe cortar el alambre para que la suma de las áreas encerradas por las dos partes sea máxima?

58. Considere la circunferencia de radio R dado. Se inscribe en ella un triángulo isósceles ABC.

Page 120: Apuntes Resueltos Todo Calculo 1

120 CAPÍTULO 2. PROBLEMAS PROPUESTOS.

Figure 2.5.2: Triángulo inscrito

a) Calcule el perímetro del triángulo en términos del ángulo θ

b) Encuentre el triángulo cuyo perímetro es máximo.

59. Cálcule el área del máximo rectángulo que es posible inscribir, con sus lados paralelos a los ejes coordenados, lagura encerrada por las curvas 3y = 12− x2 y 6y = x2 − 12.

60. Doble una hoja de papel rectangular haciendo coincidir el vértice C con un punto del lado AD como se muestra enla gura:

Figure 2.5.3: Hoja doblada

a) Demuestre que l2 = x2 +ax2

2x− a

b) Determine el valor de x que minimiza el largo l

c) Calcule el valor mínimo de l

61. En un cuadrado de lado 2a se inscriben don circunferencias de radios r1 y r2 centradas en la diagonal del cuadrado,tangentes entre si y ambas tangentes al cuadrado, como se muestra en la gura

Page 121: Apuntes Resueltos Todo Calculo 1

2.5. DERIVADAS 121

Figure 2.5.4: Esquema geométrico del problema

a) Demuestre que r1 + r2 =4a

2 +√

2

b) Determine los valores de r1 y r2de modo que la suma de las áreas de los círculos sea mínima. Justique.

62. Considere un cuadrado de lado 1[m]. En tres vértices consecutivos de él se toman los centros de tres circunferenciasde forma que los radios de dos de éstas sumen 1[m] (ver gura)

Figure 2.5.5: Situación gráca

a) Hallar los radios de las circunferencias para que la suma de sus áreas sea máxima.

b) Calcule dichas áreas.

63. En procesos de transferencia de calor, se sabe que una condición necesaria para que exista este fenómeno es queexista un ujo de calor Q en el sistema, además, para calcular el ujo de calor por conducción en una esfera endonde hay un uido que genera energía con temperatura TA y coeciente convectivo constante hA, un metal con unacierto radio , con una conductividad térmica kA y un aislante que posee un radio r3 , conductividad térmica kB y elambiente con una temperatura constante TB y coeciente convectivo constante hB se tiene la expresión siguiente:

Q(r3) =4π(TA − TB)

1r21hA

+1r1− 1r2

kA+

1r2− 1r3

kB+ 1

r23hB

Se llama radio crítico rc al radio de aislante optimo para producir las menores perdidas de calor al ambiente.

Page 122: Apuntes Resueltos Todo Calculo 1

122 CAPÍTULO 2. PROBLEMAS PROPUESTOS.

Demuestre que el radio crítico para la situación descrita es:

rc =2kBhB

64. Para sistemas con reacción química, es posible deducir una expresión que describe la velocidad de una reacciónautocatalitica A+R R+R , la cual es

(−rA) = k1cA(c0 − cA)− k2c2A

Donde (−rA) es la velocidad de reacción, k1, k2 constantes de velocidad, c0 concentración inicial constante y cAconcentración del reactivo A, en donde además cA = cA(t)

a) Demuestre que:

d(−rA)

dt= (k1c0 − 2k1cA + 2k2c0 − 2k2cA)

dcAdt

b) Dado que para que exista cinética debe de existir una variación de concentración de A (cA) en términos deltiempo, deduzca que la concentración óptima para la velocidad de reacción está dada por:

cA =c0(k1 + 2k2)

2(k1 + k2)

65. En escurrimiento de uidos, particularmente en el caso de un gas que uye a través de un tubo de diámetro D, delargo L, es posible modelar una expresión que determina la dependencia de la densidad del ujo másico del gas Gen términos de la presión de salida p2, mediante la expresión:

G2 =M(p2

1 − p22)

2RT(

2f(LD

)+ ln

(p1p2

))Demuestre que esta expresión determina un ujo máximo GMAX y que este ujo es

GMAX = p2c

√M

RT

Donde p2c es la presión que determina el ujo máximo.

66. Una estatua está colocada sobre un pedestal, como se muestra en la gura.¾A qué distancia del pedestal debe pararsela persona para maximizar el ángulo visual θ? (indicación: recuerde la identidad tan(θ2−θ1) . También es sucientemaximizar tan θ en vez de θ ¾por qué? )

Figure 2.5.6: Estátua

Page 123: Apuntes Resueltos Todo Calculo 1

2.5. DERIVADAS 123

67. Dos bobinas que condicen corrientes iguales producen un campo magnético en un punto Q del eje x de intensidad

B =1

2µ0r

20I

[r20 +

(x+

r0

2

)2]− 3

2

+

[r20 +

(x− r0

2

)2]− 3

2

Donde µ0 , r0 e I son constantes (vea gura). Demuestre que el valor mínimo del campo magnético ocurre en x = 0

Figure 2.5.7: Dos bobinas

68. El principio de Fermat establece que: la luz sigue, entre dos puntos A y B. la trayectoria que corresponde al tiempomínimo de recorrido.

Sean A y B dos puntos pertenecientes a medios de propagación diferentes separados por una interfase. Se de-sea determinar cual es la trayectoria que sigue un rayo de luz para ir desde el punto A al punto B , siendo v1y v2

las velocidades de los medios I y II

Figure 2.5.8: Situación física

Demuestre que

sinφ1

sinφ2=

v1

v2

Indicación: calcule el tiempo total recorrido en términos de x (que representa la distancia entre el origen y el puntoA) , siendo la distancia d (largo horizontal entre el punto A y el punto B ) conocida y recuerde que el rayo de luzse propaga con movimiento rectilineo uniforme.

69. Las dimensiones de un rectángulo varían de modo que su área permanece constante. ¾Cuál es la rapidez con quedecrece la altura del rectángulo en el momento que la base y la altura son iguales?. Suponga que la base crece conrapidez de 5(m/s)

Page 124: Apuntes Resueltos Todo Calculo 1

124 CAPÍTULO 2. PROBLEMAS PROPUESTOS.

70. Una persona de 2(m) de altura camina a una rapidez constante de ,3(m/s) alejándose de un poste de alumbrado de6(m) de altura. ¾Con qué rapidez se alarga la longitud de la sombra?

71. Dos barcos, A y B, parten del mismo punto O según drecciones que forman un ángulo de 120º. El barco A navegaa 20(km/h) y el barco B a 30(km/h). ¾Con qué rapidez está variando la distancia entre ellos en el instante queOA = 8(km) y OB = 6(km) ?

72. Un depósito de agua tiene la forma de un cono circular recto con su vértice hacia abajo. Su altura es de 10(m) y elradio de la base es de 15(m). Al mismo tiempo, se vierte agua en el depósito a una razón de A(m3/s) y el agua salepor el fondo a una razón de 1(m3/s). Calcule el valor de A de modo que el nivel de agua ascienda a una razón de4(m3/s) en el instante que el agua alcanza una altura de 8(m)

73. Una mancha con forma de cilindro recto circular se ha formado al derramarse en el mar 100[m3] de petróleo. Calculecon qué rapidez aumenta el radio de la mancha cuando ese radio es de 50[m] si el espesor disminuye a razón de10[ cmh ].

74. Un terreno circular de radio R se ilumina con un foco colocado en el punto A como indica la gura adjunta. Unmóvil recorre el segmento BC con movimiento rectilíneo uniforme de velocidad u, mientras su sombra S proyectadasobre el muro perimetral describe un movimiento circular de velocidad v. En un instante t cualquiera el móvil seencuentra en un punto P , siendo x la distancia BP y s la longitud de arco BS.

Figura 2.5.9: Descripción geométrica del problema

a) Hallar la relación entre θ y ϕ y calule θ en términos de x

b) Encuentre la expresión de v en términos de x

c) Tomando t = 0 cuando el móvil pasa por el punto B , determine en que posiciones del móvil la velocidad de lasombra es máxima y mínima para x ∈ [0, 2R]

d) Calcule la velocidad de la sombra cuando el móvil pasa por el punto medio del segmento BO e indique cuál esel porcentaje de esa velocidad con respecto a la velocidad máxima.

75. Cae agua dentro de un depósito hemisférico de 10[m] de radio a razón de 12 [ m

3

min ]. El agua escapa por la base del

depósito a razón de 14 [ m

3

min ] . Se puede demostrar que en cualquier instante el volumen de agua en el depósito es

V = 10πh2 − πh3

3

¾Cómo cambia la profundidad del algua cuando el h = 5? (ver gura)

Page 125: Apuntes Resueltos Todo Calculo 1

2.5. DERIVADAS 125

Figura 2.5.10: Recipiente hemisférico

76. Por el vértice de un estanque con agua con forma de pirámide invertida escapa agua a razón de 72[m3

dia ] . Si el estanquetiene 20[m] de profundidad y una base cuadrada cuyo lado es de 30[m]. ¾A qué velocidad desciende el nivel del aguacuando la altura del agua es de 12[m] ?

77. Un puente está construido perpendicularmente a la dirección de un río recto y a una altura de 5[m] sobre el niveldel mismo. En cierto momento un auto pasa por el centro C del puente a una velocidad de 12[ms ]. En ese mismoinstante una lancha L que se acerca al puente a una velocidad de 20[ms ] dista a 100[m] del punto P situado sobre elagua y exactamente bajo el centro del puente. Si la carretera continua perpendicular al rio , ¾cuál es la velocidad lacual se están separando y el auto 8[s] despues de que aquélla pasó por el punto P?

Figure 2.5.11: Puente y río

78. Considerar la curva

x(t) = a cos t

y(t) = a sin t

a) Demuestre que la curva se trata de una circunferencia de radio a y centrada en el origen.

b) Calculardy

dx

79. Calcule dydx y d2y

dx2 si

x(t) = t4 − 8t3 − 9

y(t) = t3 − t2

80. Sea la curva

x(t) = t3 + t

y(t) = 3t4

4 + t2

2 + 1

Page 126: Apuntes Resueltos Todo Calculo 1

126 CAPÍTULO 2. PROBLEMAS PROPUESTOS.

a) Determine dydx y d2y

dx2

b) Indique para qué valores de t ∈ R la curva es simultáneamente creciente y convexa.

c) Demuestre que los puntos (x, y) satisfacen la relación

d2y

dx2·

(1 + 3

(dy

dx

)2)

= 1

81. Dada la curva

x(t) = t ln t

y(t) = 2 ln tt

a) Determine los valores de t para los cuales la curva se encuentra en el primer cuadrante. En particular, indoqueel valor de t para el cual la curva pasa por el origen.

b) Determine los puntos (x, y) para los que la recta tangente a la curva es paralela al eje X y donde es paralela aeje Y .

82. Considere la curva

x(t) = cos t

y(t) = sin(2t), t ∈ [0, π]

a) Determine la región del plano donde se encuentra la curva.

b) Calcule los puntos (x, y) donde t = 0, π4 ,π2 ,

3π4 , π

c) Encuentre los puntos donde la tangente es paralela al eje X y donde es paralela al ele Y .

d) Analice el crecimiento de la curva

e) Demuestre que d2ydx2 =

−2 cos(t)·(2 sin2 t+1)sin3 t

y analice la concavidad de la curva.

2.6. Integrales

1. Usando el método de sustitución simple, calcule las siguientes integrales:

a)ˆ

eln x

x2 + 7dx

b)ˆ

x2

x3 + 8dx

c)ˆ

(x+ 2) sin(x2 + 4x− 6)dx

d)ˆx sin(1− x2)dx

e)ˆx3√

1 + x4dx

f )ˆ

arcsinx√1− x2

dx

g)ˆ

arctan(x2

)4 + x2

dx

h)ˆ

tan(lnx)

xdx

i)ˆetan 2x sec2(2x)dx

Page 127: Apuntes Resueltos Todo Calculo 1

2.6. INTEGRALES 127

2. Usando integración por partes, resuelva las siguientes integrales:

a)ˆx cosxdx

b)ˆx2 sinxdx

c)ˆ

lnxdx

d)ˆ

ln(x+√x2 − 1

)dx

e)ˆ

arctanxdx

f )ˆ

arcsin(√x)√

xdx

g)ˆx sin2 xdx

h)ˆ

sinn xdx

i)ˆ

tann xdx

j )ˆx sinx cosxdx

k)ˆ

cos (2 lnx) dx

3. Use el método de sustitución trigonométrica para resolver las siguientes integrales:

a)ˆ

dx√(4− x2)

3

b)ˆ √

25− x2

xdx

c)ˆ

dx

x√x2 − 1

d)ˆ

x2

(x2 + 82)32

dx

e)ˆ

dx

x2 − a2

f )ˆ

dx√5− 4x2

4. Usando el método de fracciones parciales calcule las siguientes integrales:

a)ˆ

dx

x2 − 5x− 6

b)ˆ

xdx

x2 − 4x+ 4

c)ˆ

dx

x3 − 2x2 + x

Page 128: Apuntes Resueltos Todo Calculo 1

128 CAPÍTULO 2. PROBLEMAS PROPUESTOS.

d)ˆx4 − 6x3 + 12x2 + 6

x3 − 6x2 + 12x− 8

e)ˆx4 − 2x3 + 3x2 − x+ 3

x3 − 2x2 + 3xdx

f )ˆ

(3x+ 7)dx

(x− 1)(x− 2)(x− 3)

g)ˆ

(x2 − 1)dx

(x2 + 1)(x− 2)

h)ˆ

dx

(x+ 1)(x2 + 1)

i)ˆ

etdt

e2t + 3et + 2

j )ˆ

sin θdθ

cos2 θ + cos θ − 2

5. Completando cuadrados resuelva las siguientes integrales:

a)ˆ

dx

x2 + 2x+ 5

b)ˆ

(2x− 3)dx

x2 + 2x+ 2

c)ˆ

xdx√x2 − 2x+ 5

d)ˆ

(x+ 2)dx

x2 + 2x+ 2

e)ˆ

(2x− 3)dx

x2 + 6x+ 15

6. Las siguientes integrales se realizan por más de un método conocido de integración, calcúlelas:

a) Obtenga una fórmula de reducción para In =

ˆxn sin(2x)dx

b) Calcular I =

ˆx2

1 + x4dx

c) Calculeˆ

secx tanx

1 + sec3 xdx

d) Determine la fórmula de reducción de Ip =

ˆdx

(x2 + z2)p, p ∈ R.

e) Calculeˆx2 arctanx

1 + x2dx

f ) Calculeˆ

(lnx− 1)dx

x(1 + ln2 x)

g) Calculeˆ

18 tan2 x sec2 xdx

(2 + tan3 x)2

h) Calculeˆ

sin(√θ)dθ√

θ cos3(√θ)

Page 129: Apuntes Resueltos Todo Calculo 1

2.6. INTEGRALES 129

7. Calculeˆ

x2 − 1

x4 + x2 + 1(indicación: x4 + x2 + 1 = (x2 + x+ 1)(x2 − x+ 1) )

8. Sean n y r numeros naturales tales que In,r =

ˆxn(x + a)rdx. Obtenga una fórmula de reducción para In,r y con

ello calculeˆx3(x+ 3)6dx

9. Sea f : R→ R+ derivable y g : R→ R contínua tales que f ′(x) + g(x)f(x) = 0 . Muestre queˆg(x)dx = − ln (f(x)) + C

10. Demuestre que para n ∈ NIn =

ˆ(lnx)ndx = x(lnx)n − nIn−1

11. Demuestre que para n ∈ N

In =

ˆ(x+ 1)n sin(2x)dx = −1

2(x+ 1)n cos(2x) +

n

4(x+ 1)n−1 sin(2x)− n(n− 1)

4In−2

12. Demuestre que para n ∈ N

In =

ˆxm lnn udu =

um+1

m+ 1· lnn u− n

m+ 1In−1

13. Demuestre que si m , n ∈ Z+ , entonces

ˆsinm(x) cosn(x)dx =

sinm+1(x) cosn−1(x)

m+ n+

n− 1

m+ n

ˆsinm(x) cosn−2(x)dx